71 the hon. r. richardson, - parliament of victoria … · 71 the hon. r. richardson, ... if i had...

72
71 The Hon. R. Richardson, 22nd August, 1894. have been inquiring into this matter the inhabitants of Poowong have waited on me, and have offered to pay the interest on the money that will be expended in making the line if the line should be from N yora to Poowong. The people from Gembrook have also made the same offer, and they have asked, if Government arc uot prepared to do it., that we will get the sanction of Parliament for them to do It themselves. 1147. The secretary to the Committee will read a letter from the Honorable W. A. Zeal, which is an important one, ami we would like to luwe your opinion of it.-[ The secretar!f read tlw same] ?-Those are quotations from the Transactions of the Institute of Civil Engineers. Two of those illustrations used here I handed in this morning on the document I gave you which I copied from volume 96. I agree with this, that this is the kind of thing that we want to serve our outlying districts. The questiOn of break of gauge has been brought before this Committee I see, but, as I stated in the Hou!'c, I do not regard this as a break of gauge-it is a different gauge for n different purpose. If I had to choose between the two gauges, the wide gauge and the narrow, where there was plenty of traffic and plen1y of money to expend, such as they have been having at the Hailways, putting their hands in an accessible manner into the loan fund, I should go for the broad gangc-I would not waut two p-anges; the varied wants of the country render it necessary, I think absolutely that there should be a cheap line made, ami you can only m•tke that line cheap enough to carry the tratnc by having rt narrow-gauge line. You are aware that they have three or four gauges in India. They have had some of those narrow gauges carryiug an immense traffic, and they have replacetl them with wirle-gauge lines and removed the narrow gauges into other parts of the country; but those lines are specially for military and strategic purposes. The same thing occurs here where we have people living in the mountains n distmwo away, where there is not traffic enough to pay for a wide-gauge lino of traffic, but enough to make a narrow-gauge line pay, so I do not regard it as a break of gauge, only a change of gauge; awl motlcrn appliances have been adopt ell ami so improved that the transhipmr,nt of loading done witb little difficulty and very little delay. I think I forwardetl to you some plans where it shown how that can be done. One plan shows the narrow-gauge trucks with fom· parts on end on a skeleton carriage or trnck. These can be loaded with 2 tons of wheat in each or 3 tons, us the case ma.y be, and they can be easily transhipped to the ordinary trucks that we are running without nnloading anti with very little difficulty. It is found on the continent of Europe that the 6 tems of roots tbat they carry in hnlk there can be transhipped from the narrow to the wille or the wide to the narrow gauges in fifteen minutes l1y two men, so I do not regard that as an objection that is worthy of consideration. Another statement has been made in this evidence that the >vind would blow narrow-gauge trucks over. I forwarded to your secretary a tabulated statement showing what force it would requfrc to blow a truck over, and giving an illnstmtion by diagram of where the line of gravitation would be when a truck was blown over. Y on will sec from that that it requires 29 lbs. of pressure to the square foot to overturn nn empty truck. \Yhen you get 29 lbs. to the square foot that becomes a hurricane--it is about ·76 miles nn hour; and to blow a loaded truck over it woulrl require a wind running at about 116 miles an hour-that is not within the experience of man ; but for any one to suppose that the wind could blow one of those trucks over is almost to suppose an impossibility, unless it iR under peculiar because if 29 Jl>s. pressure will blow a truck over on the narrow gauge it will blow a truck over on the wide gauge. 11·18. Are you referring to trucks in motion? -No, st11tioHary. Trucks in motion are not likely to be afiected in the same degree. I think there arc only one or two cases that arc recorded of trucks or carriages lmving been capsized by the wind, and that was under peculiar circumstances. I think I remember one case occurring in America, and latterly there was a ease in New Zealand; it was a peculiar place where they were, and the peculiar way in which the wind was coming, and the way in which it caught the trucks. It is one of the contingencies that ma,y happen with a wide gauge as readily as with the narrow, so I do not attach any importance to statements of that kind ; they arc made without fully going into the matter, and fully considering it. ] 149. Your evidence is altogether in favour of our recommending Parliament that the narrow gauge be adopted, and that narrow gauge should he the 2-ft. one ?-2 feet or 2ft. fiin. 1150. Have you quite made up yonr mind which would be the better ?-I have always favoured the 2ft. 6in., but I Reo from the of the Transactions of the Institute of Civil Engineer;;, and by some letters which I have from experts in connexiou with this question, that the experience of engineers in connexion with the narrow gauge is that the 2-ft. gauge is the most solid. There is a French engineer gave evidence recently to that effect, and Mr. V r.n de Velde showed men statement of tbe evidence that had been given to that effect. So far as that is concerned, I have seen a tramway on a 14-in. gauge, and running as solid and mfe as it could be. 1151. Yon referred to the fact that in India they have, for military purposes, taken np the metre gauge an<l Willene<l it to the standad gaugc-wonhl it not be almost impracticable in our hilly country here to widen the gauge as traffic developed ?-Not at all. 1152. Except at a very large additional cost the cost would not be more here than it would be in any other place. As to the bogie machinery which, I think, the Engineer-in-Chief referred to when he gave his evidence, if you have bogie engines, ami the Fairlie principle generally adopted in the rolling- stock, you could go ronud very short carveR or curves with a short radius. If you followed the line of the narrow gange you would have to adopt that kind of rolling-stock. 115;3. In carrying a 2-ft. line over diflicult. country you might be able to carry it where it wouhl be almost impossible to carry a wider line ?-I do not see that. 1154. Except at an excessive :uHl tunnelling ?-The narrow-gauge lines are taken round the hills. No doubt the length is incrc.tsed, bnt they are taken round the hills-they are not taken through them, and if you could get rolling-stock of a larger type that would go round the same curves, then you could follow the narrow-gauge lines. ] 155. You said there wonld be a great saving in earthworks and cnttings ; would there be any difference in the cost of the rolling-stock ?-A very great difference. The 6-ton locomotives for the narrow gange cost £1,000, ami f()r other sizes from ()up to 15 tons the cost is increased, bnt it is not increased in t.hc same ratio that larger engines on the wide gnnge are increased. The locomotive engines on the wide gauge cost several thousands each, so that the rofling-stock is about one-fouTth. I do not

Upload: hoangcong

Post on 06-Mar-2018

216 views

Category:

Documents


2 download

TRANSCRIPT

71 The Hon. R. Richardson, 22nd August, 1894.

have been inquiring into this matter the inhabitants of Poowong have waited on me, and have offered to pay the interest on the money that will be expended in making the line if the line should be from N yora to Poowong. The people from Gembrook have also made the same offer, and they have asked, if t~e Government arc uot prepared to do it., that we will get the sanction of Parliament for them to do It themselves.

1147. The secretary to the Committee will read a letter from the Honorable W. A. Zeal, which is an important one, ami we would like to luwe your opinion of it.-[ The secretar!f read tlw same] ?-Those are quotations from the Transactions of the Institute of Civil Engineers. Two of those illustrations used here I handed in this morning on the document I gave you which I copied from volume 96. I perfec~ly agree with this, that this is the kind of thing that we want to serve our outlying districts. The questiOn of break of gauge has been brought before this Committee I see, but, as I stated in the Hou!'c, I do not regard this as a break of gauge-it is a different gauge for n different purpose. If I had to choose between the two gauges, the wide gauge and the narrow, where there was plenty of traffic and plen1y of money to expend, such as they have been having at the Hailways, putting their hands in an accessible manner into the loan fund, I should go for the broad gangc-I would not waut two p-anges; the varied wants of the country render it necessary, I think absolutely ~o, that there should be a cheap line made, ami you can only m•tke that line cheap enough to carry the tratnc by having rt narrow-gauge line. You are aware that they have three or four gauges in India. They have had some of those narrow gauges carryiug an immense traffic, and they have replacetl them with wirle-gauge lines and removed the narrow gauges into other parts of the country; but those lines are specially for military and strategic purposes. The same thing occurs here where we have people living in the mountains n distmwo away, where there is not traffic enough to pay for a wide-gauge lino of traffic, but enough to make a narrow-gauge line pay, so I do not regard it as a break of gauge, only a change of gauge; awl motlcrn appliances have been adopt ell ami so improved that the transhipmr,nt of loading i~ done witb little difficulty and very little delay. I think I forwardetl to you some plans where it i~ shown how that can be done. One plan shows the narrow-gauge trucks with fom· parts on end on a skeleton carriage or trnck. These can be loaded with 2 tons of wheat in each or 3 tons, us the case ma.y be, and they can be easily transhipped to the ordinary trucks that we are running without nnloading anti with very little difficulty. It is found on the continent of Europe that the 6 tems of roots tbat they carry in hnlk there can be transhipped from the narrow to the wille or the wide to the narrow gauges in fifteen minutes l1y two men, so I do not regard that as an objection that is worthy of consideration. Another statement has been made in this evidence that the >vind would blow narrow-gauge trucks over. I forwarded to your secretary a tabulated statement showing what force it would requfrc to blow a truck over, and giving an illnstmtion by diagram of where the line of gravitation would be when a truck was blown over. Y on will sec from that that it requires 29 lbs. of pressure to the square foot to overturn nn empty truck. \Yhen you get 29 lbs. to the square foot that becomes a hurricane--it is about ·76 miles nn hour; and to blow a loaded truck over it woulrl require a wind running at about 116 miles an hour-that is not within the experience of man ; but for any one to suppose that the wind could blow one of those trucks over is almost to suppose an impossibility, unless it iR under peculiar circnmstance~, because if 29 Jl>s. pressure will blow a truck over on the narrow gauge it will blow a truck over on the wide gauge.

11·18. Are you referring to trucks in motion? -No, st11tioHary. Trucks in motion are not likely to be afiected in the same degree. I think there arc only one or two cases that arc recorded of trucks or carriages lmving been capsized by the wind, and that was under peculiar circumstances. I think I remember one case occurring in America, and latterly there was a ease in New Zealand; it was a peculiar place where they were, and the peculiar way in which the wind was coming, and the way in which it caught the trucks. It is one of the contingencies that ma,y happen with a wide gauge as readily as with the narrow, so I do not attach any importance to statements of that kind ; they arc made without fully going into the matter, and fully considering it.

] 149. Your evidence is altogether in favour of our recommending Parliament that the narrow gauge be adopted, and that narrow gauge should he the 2-ft. one ?-2 feet or 2ft. fiin.

1150. Have you quite made up yonr mind which would be the better ?-I have always favoured the 2ft. 6in., but I Reo from the dbcu~sion of the Transactions of the Institute of Civil Engineer;;, and by some letters which I have from experts in connexiou with this question, that the experience of engineers in connexion with the narrow gauge is that the 2-ft. gauge is the most solid. There is a French engineer gave evidence recently to that effect, and Mr. V r.n de Velde showed men statement of tbe evidence that had been given to that effect. So far as that is concerned, I have seen a tramway on a 14-in. gauge, and running as solid and mfe as it could be.

1151. Yon referred to the fact that in India they have, for military purposes, taken np the metre gauge an<l Willene<l it to the standad gaugc-wonhl it not be almost impracticable in our hilly country here to widen the gauge as traffic developed ?-Not at all.

1152. Except at a very large additional cost ?~~No; the cost would not be more here than it would be in any other place. As to the bogie machinery which, I think, the Engineer-in-Chief referred to when he gave his evidence, if you have bogie engines, ami the Fairlie principle generally adopted in the rolling­stock, you could go ronud very short carveR or curves with a short radius. If you followed the line of the narrow gange you would have to adopt that kind of rolling-stock.

115;3. In carrying a 2-ft. line over diflicult. country you might be able to carry it where it wouhl be almost impossible to carry a wider line ?-I do not see that.

1154. Except at an excessive co~t :uHl tunnelling ?-The narrow-gauge lines are taken round the hills. No doubt the length is incrc.tsed, bnt they are taken round the hills-they are not taken through them, and if you could get rolling-stock of a larger type that would go round the same curves, then you could follow the narrow-gauge lines.

] 155. You said there wonld be a great saving in earthworks and cnttings ; would there be any difference in the cost of the rolling-stock ?-A very great difference. The 6- ton locomotives for the narrow gange cost £1,000, ami f()r other sizes from ()up to 15 tons the cost is increased, bnt it is not increased in t.hc same ratio that larger engines on the wide gnnge are increased. The locomotive engines on the wide gauge cost several thousands each, so that the rofling-stock is about one-fouTth. I do not

The Hon. R.Richardson, ll2nd August, 1894. 72

think it is quite one-fourth; and then the rolling-stock, except the engines, is made oflighter material, and yet they carry more in proportion to the weight of the carriages.

1156. Is it not a fact that although they might be built of lighter material, the mechanism has to be compressed into so small a compass that unless the material and workmanship are better than the ordinary ones, breakages would be oftener occurring ?-No. You cannot better workmanship than is put into ordinary locomotives. No engineer who has had experience in constructing locomotives will tell you that better work than that can be done.

1157. The difference in the cost of the maintenance of a line wonld be lc~s ?-Very much less, and the working expenses are less; you do not want the number of men, and yon do not want the stations. If yon do without stations in both cases, of course there would be no difference.

ll58. You know the principle of the Decauvillc steel sleeper, riveted on to the rails ?-Yes. 1159. It would not be necessary to use those steel sleepers here where wood is so abundant and

cheap ?-I would act upon the principle of taking whatever is the cheapest. On the narrow-gauge lines (Fowler's in England, the Decauvillc, and many German manufacturers who do nothing else but make ~arrow-gauge lines), the rolling-stock is on the snme principle ; they have the sleepers either bolted on or nveted on. The Decauville is peculiar in that it has the whole thing riveted instead of bolted.

1160. Those 2-ft. lines could in many instances be carried on the onlinary roads of the colony?­Yes.

1161. And saYing a large amount of expenditure ?-Yes; at present we do not make a line unless we have the land given to us. There is necessarily a great expense connected with it. In this Wycheproof line we are laying the line on the main road for eleven miles.

1162. By the Hon. E. Morey.-How wide is that road ?-A 3-chain road, but there is plenty of room ou the l §-chain ro!ltls. The lloort to Quamlmtook line is carrie(l along the side of the road-·just a strip taken off the laud. If I bad been fortunate enough to go there before it was carriml along the land I should have carried it along the road. I did suggest it before I went, but the engineers have an objection to depart from auy custom; they do not like to go on to the road; hut there wns plentv of room, There is very little traffic along those roads. •

1163. One or two of our witnesses suggested that in the event of the State not undertaking any of those branch lines they might be undertaken by private enterprise; would you approve of that ?-Where the State is not prepared to make a line to any place I would certainly allow private companies or private individuals to do it. There is a grave objection to it, bnt you cannot shut people out who are twenty miles away from a city by refusing to mako them a line. I would like to guard myself against any such arrange~ ment as has come out of the Katamati.te Tramway or the Koondrook Tramway. The Railway Department is working those two lines now, and working them at a loss. The two municipal bodies are very anxious that the Railway Department should take them over.

1164. By Jf1·. Burton.-Take the responsibility over ?-Yes. 1165. By tlte Cha-irman.-At the commencement of yonr statement you laid great stress upon the

fact that lines should be built according to traffic, and that the traffic should first be determined before the line was fixed upon; has not that occurred all through ?-No.

1166. The bulk of our line~ have peen constructed on that principle ?-No, there is not any line that has been constructed since the late Mr. Higinbotham canstrncted the North-Eastern line where that has been the ruling principle. I have found in my experience that the line that was most likely to pay best was the most difficult to obtain.

1167. You told us you had read the bulk of the evidence that has been given before this Committee? -Not the bulk of it; I have read some of it. I have read Messrs. Rennick's and Norman's.

1168. You said the question of what we call the break of gauge is not an engineer's question simply, except for the cost-engineers arc not business men ?-I was referring then to the place where and the kind of line that should be made. When I stated they were not business men, a break of gauge is a,

mechanical operation as a rule, and one that engineers would not be in a position to give valuable information about.

1169. What we call a break of gauge you term a change of gauge-what do you term a break of gauge ?-I regard as a break of gauge tho question that was before Parliament in 1870, when it was proposed to make the North-Eastern line on a different gauge and running out of one station to different parts of the country-having two gauges for it when the North-Eastern was likely to have more traffic on it than any other line. I think it is quite a sensible thing to do if you have a large amount of traffic to make such a line as will carry that traffic and carry it, most efficiently, but where you have only a limited quantity of traffic you ought to limit your expenditure of money so that the traffic will pay for it.

1170 . .By tlte lion. D. Mdville.-Are there any places where you could safely say the broad gauge could be safely constructed in this country ?-No, there is no place that I could recommend with the statement that it was likely to pay.

1171. As to the experiments that have been made with those new lines, do you think they arc now built on paying lines ?-Do you refer to the extensions in the mallee?

1172. Yes ?-I have said before that the Wycheproof line will have 70,000 acres of land under cultivation this year. I have given the data upou which I base my statement regarding that line.

1173 . .By 1Wr. Young.-That is a line you are making yourselves?-Yes. 117 4. Without the consent of Parliament?-Yes. 117 5. By the lion. D. 1vfelnille.-Would these lines that have been finished pay were you not

charging the rate that was made l>y the Railways Committee, called the local rate ?-Certainly not. The bulk of that traffic would be carried at a !d. per ton per mile. The first 100 miles from Melbourne or from the port of transhipment is charged Id. a mile ; the seconcl 100 miles is charged 1d. ; the next lOO and anything beyond is charged -!d. per ton per mile-id. per ton per mile will not pay.

1176. You have ascertained that definitely ?-I have had it carefully worked out by the Commissioners; they said that with the greatest possible care, after all the facts were before them, that will not pay. I have a Bill before Parliament in which I propose reducing those special rate~. that is, altering the rates to the ordinary rates. I think those people np there were suffering unfair treatment ; they were charged more than other pe()ple, and when the great pres;;ure came upon the Hailway Department about

73 The Hon. R. Riclmrdsou. !2nd August, 1894.

the freights and fares, that was the only thing I could see tMt was likely to be of any service or that it would be fair to do. I have been opposed to the reduction of freights and fares.

1177. You say they will not pay if you take away those special rates. The Railway Committee concluded, from the evidence and the willingne;<s of parties to pay, that it wonld be well to give it a trial. Do you think if yon abolish those rates it will pay ?-Xo, it will not pay.

1178. Jt is not generally known that we enrry so cheaply. Is that not ''omething new ?-It is not new; but the mallee farmers have made snch a noise that they were being charged so nmch, awl, with tho nssistanee of the press, they eomplaiued of what the Railway Department wac> doing in charging them so much more than other people, until they got it into their heatlc they WCl'O ovorclmrged.

1179. It is a piece of iguornnee on the part of some one. If \YO arc C[lrrying nt a farthiug, is there any use going lower than that ?-No, the avorag<> is taken of the an•rage distance carried, aml then it is the average price the~t they qnote. There is n diflh-mH'O bctwoou thew heat rate,; and the l\!!i'icultnral rates ; the agricultuml rates arc higher. lu the b~t cln«siikaJion cftccte<l by the CommLsioners they did reduce certain thing~ in connoxion with a;zrieultnre, they rod!IC<'(1 lwy nud st,raw-th0y reduced that with the object of getting that t.raffic. I tltit1k it was a mi"htb:n they nevm· \vill got it; bnl that was tJwlr object in doing it. \Vith thnt exception the agricultnral mtcs are higher than the whent rateti ; bnt the wheat rates are what I have told you. If the w ht>nt is enrriod only 200 miles, the first 100 is 1 d., and the other is four-fifths of a penny pc;· mile, so haye to average 'th::t. Then when ir gets beyond it is charged fd. per ton per mile. That has to l>ronght into it, ami that rotlncc~ the aYorage.

1180. \Ve have to determine whotlJcr even the narrow gauge will pay, aml we have to consider the question of rates. Suppose the Committee t1o rocornmeud tlir~ survey of n llf!JTow-gaugo lim;, have you any place that yon could quickly get sneh a snrvey mn<le of?-vVo have u :nnnber of places. I received the resolntions of the Committee, and }'[r. Hitohie lu:s been instructou io reply to it, sayiug that as won !Hl

this vote of \Vant of Confitleneo h_, oyer, step,; will he taken to oiiect to the wi~l1es of the Committee. Thoro are several places where liuos have been surveyed on the wide gauge, an<l we have the quantities and the estimate~ n~ to their cost, r.ntl I conld very oa,;ily have a survey nmdc, or I could examine tho survey that has been macle, and see whether il1at snrvey could be used for the narrow gauge. I do not see any reason why it should not, because all the difierence there wonl<l be wonl<l he in the cross­sections, that is in the witlrh of the cutting, or in the width of the fonnation, an<l that would reduce the quantity very much, and would make the two lincB exactly on tilO mme lmAis. Th:n >YnH what Mr. Woodroffe was euggesting.

lli'il. There nre 80mo that would tak3 a totally t!iiTerent track-they would go right round the hills ?-\Vhere there wore deep cuttings. The same survey would do to a certain extent in that ca~e.

1182. By the Hon. E. JJlort'?J·- \Vhat: weight of rail wonltl you rccommo!l<l ?-I woultt recommend for the narrow gauge, where the traffic wns su!iioiem for the rail, n :W-11>. steel mil; ·whC're rho traffic was less and would not warrant it, I would a;lopt a 20-lh. steel rail. It is most oxtraonlinary how it has turned out ill connexion with the mnintonanco o!' thoPe lines in lutllu. It ha~ been provo<l that for seYeu years a steel rail of 20 lbs. has been used, and yet lhere has hoen apparently 110 wear ami tear upon that rail. The weight is not much, the speed is not high, ai!tl oYOrything goc~ in fayonr of ~aving· the mil.

1 183. There cannot he m:my cnrvos apon that lino ?-If it i~ hcrcvy co11ntry, hilly country, you want a he:tvy rail, :tnd the CJU'vos make a difference to it wo.

1184. Has the Government not got a great number of light rnil~ lying about that they are not using ?-\Ye have a few miles of light 50-lb. rails lying down f<t Kilcunda.

1185. Are there not many other~< ?-Kot a large llllmbor. 1186. Do not the tram mil;.; wear :~way going round the st.reets ?-Yes, hut they are very short

curves. 1187. That line at llw gas works lu;s 24-lb. rails ; that Foems very light ?-I think it is 20 lbs. The

rail depend;; upon the traffic you will have over it. 'With rnuch trailic you want a heavier rail. I take it that this qnoRtion is beyond the oxperime!;tul stngos~-they lmvo beeu adopted for thousands of miles.

1188. They have n0t beenndoptod iu this i'olony '?~No. 1189. By Jf1·. Young.-As to the qnostion:, ubont the milwayr; that have been made in the past,

you stated that in oounexion witl1 the hvo tramways you "'ere saddled with a linl,ility in taking those OYer. Seeing that that is so, ana seeiug that you have broug:ht fonYanl the two lines which the Railways Standing Committee recommen•1ed-the Donahl to Birehip, and the Wurraeknabeal to Boulnh-and that, though those lines are paying nt the present time, you iutew1 to haw; tl,ose special comlitious raken oiT, and after that they will not pay; do you think it is polic:y to ;my what those t\I'O councils have done, and also to say that you are foreed to take ofl' tlw special conditions of tlwso tiYO linos ?-The t,wo lines, the Katamntite and Koonilrook, are not taken over yet; we nre workiug them, but 1 purpose submitting the other proposal to Parliament, aml Jetting Parlian!eiJt do a8 it think;< be~l--therc is n matter of policy in it. \Vhen 1 was at neuluh some time ago, I :<aw the people there ; tl:ey were lontling their wheat, and I saw them at Hopotonu loading their wheat, and they were paying more for their whent from there to Warracknabeal than the people from ~Jl1i!l wore to Dimboob. I thonght it wn~ hardly fair to expect them to do that, aml UJiles~ some other rhiJ1~· w''" done to 1~10ot the exigencies of the case, I thought it was only a reasonable thing to give them, alrbou~rh .I pointe<l ont to them at the time that I did Hot promise it to them. 1 pointed out it was part of a bargain-that the Standing Committee lw<l suggested it should be done-that t;,ey lmd eutere<l i11to tlw compact, and until they got the railway they were quite willing to pay it, hnt after J.;::ot the mil IY~ty comphiucd. There WIH a. great deal of pressure brought to bear upon the Government to make lhnt r:lter:1tion. 1 ntthcr thb1k it wa~ the weight of the preBsure that was brought to bear that cause<l ir to be dm;e.

1190. What tlo you mean b:· "W('ight of pressure "?-Political influence. Jl91. Ry the lion. D. Jfe!vi!le.-Hero i" what \YO di~;coYerC'd :--''A large a.monnt of oYidence was

received as to the very high prieos that JimnPr~ and othrr< had to pay for ca.rtage to the nearest railway station. SeYeral witne;;soc slated thor had to pay as mneh a,~ ltl. per hag per mile for carting wheat, and evidenee was given in all lHH'ls of the er,foll.v that the em;t of carting hy road was geuerally from ten to twn1ty times as rnueh as the rai!\l';:y wou],1 l•c, a.nd that the carliug eoul(l, as a rul~e, be done in the summer months only." The Commitwo dwn asked them \\"Oul.l they have this special rare, which did not amount to very mueh, and if ,1·on cnmwt: mnintnin the eharge of 3d. per ton per mile until it is evident that

the Hon. R. Richardson, 22nd August, 1894. 74

the railways pay, it is rather a slight upon the late Committee helping you if you are going to abandon that charge ?-I have no hesitation in saying it will not pay, and, as a matter of policy, I am opposed to working lines without making them pay. The Chairman of the Acting Commissioners went to the opening of the Hopetoun to Beulah Hailway, and he came back under the pressure of t,he influence that had been exerci~ed upon him and sai<l the people would be satisfied if they got this concession, and, as a matter of pohcy, the Government conceded it.

1192. You have not got it through ?-No; but it will be submitted to Parliament, and I will be bound to state what I think about it when it comes before Parliament.

1193. By j'J,fr. Younrf.-Yotl said you have travelled over many parts of the colony, and there is not a single portion of it that you would be prepared to recommend the present gauge in ?-No, not with the statement that it might be expected to pay.

1194. Is that under the old construction or the now ?-There are places where there was a very high estimate given of what the line would cost.

1195. They wore all alike ?-There are places that would eost very much less than the estimate, but I could not expect the lines to be made a~ cheaply as the Sea Lake line is made ; everything there is in its favour. If there is any pnrticular place that you want an opinion about, I would be glad to give you my opinion.

1196. Take the line from Donald to Birchip, or \Yarracknabeal to Beulah ?-The 'Varracknabeal to Beulah could be made as cheaply ns the one to Sea Lake. It is the engineers who run the cost up.

1197. There is no one to check them ?-No. I think the engineers of the Department are wishful to make the lines as cheaply as they can be made, but they want to make them good, and there are certain things we have been in the habit of supplying. I said, "\V e will not supply them," and they are not supplied, and that reduces the cost.

1198. Do you not think the statement abont the information you got in connexion with the \Yyche­proof line, where you are going to get such an immense traffic, is similftr to some of the statements made to the Standing Committee in relation to some of the other lines ?-No, I have most implicit confidence in myself and my own eyes. I saw the evidence given before the Standing Committee at Kewlyn and Allan­dalc-the evidence given before the Committee at those places was not evidence at all-they were simply matters of opinion; but I went to the \Yycheproof' line and wont along the whole line, and saw how it was inhabited. I have some i(lea of the extent of country that was cultivated, and I have had that corroborated by getting the occupation of seven miles on c~wh side of the line, and that which is under cultivation this year is 70,000 and some odd hundred acres of land. That is an estimate based upon facts.

1199. There was evidence of that kind upon tho Natimuk to Goroke line ?-I do not know how they could get that evidence upon that line; going over it I do not see how any one could see that it would pay. Mr. Madden still thinks it will pay, but we are proposing to run only one train a day there.

1200. By the Chairman.-One or two witnesses said there was a necessity for new surveys, even in the fiat country, for a 2-ft. or 2ft. 6in. line as well as the 5ft. 3in. line. I have in my hand a letter from Messrs. Hargrave and Ellis, engineers, saying they recognise the necessity for a survey for the narrow and broad gauge lines in tbe same country-do you agree with that ?-I had a letter from thoRe gentlemen, but when you come to know as much about engineers as I do you will not take much notice of a letter like that.

The witness witltdrew.

Thomas Hale W oodroffe, recalled and further examined.

1201. By the Chairman.-Did you hear the letter that Jl.1r. Watson, the Clerk of the Committee, read from the Hon. W. A. Zeal ?-Yes.

1202. Do you agree with what is stated in that letter ?-I have no doubt that the facts stated therein are correct. Of course, the lon<l which is spoken of as having been drawn up this grade is a very small one; it is only 10 tons, that means the vehicle as well.

1203. By the Hon. D. 1llelville.-What grade is that ?-One in 115; that rules the roart. 1204. Your average on the spurs is only 25 tons ?-Yes. 1205. If you have only 25 tons, with such appliances ag these upon a 2-ft. line, and with the

cheapness of a 2-ft. line, would you still say we should not try it ?-The argument cuts the other way just as well ; if the broad-gauge line can be made to carry that same tonnage at the same cost or a little more I say you should have the broad gauge.

1206. From the evidence you have read it looks absurd to make such a proposition. You say ii the broad gauge can be made at the same cost-you say that even the broad gauge can be reduced enormously ; but you do not say there is any doubt that the narrow gauge cannot be made at about one-fourth the cost?­\Vhat I contend is this, thnt given tho same capacity and the same traffic on both lines, if the broad gauge can he made as cheap as the narrow, which ~Ir. Rennick, who has studied the question, says it can, and cheaper, then I say we shouhl make the broad gauge.

1207. You are not tied to the views of Mr. Renuick-I want your individual opinion upon that?­The opinion must be formed upon the estimate of the cost and. the character of the two lines. I have not estimated the cost of the two lines. It wonld take me some time to take an example and grade it and estimate it under the two conditions, and no one could say until he had done that.

1208. Having ascert~tined the average load was 25 tons, will yon not abandon the brond gauge in cases where the average was so exceedingly low ?--If you will allow me I will take this example-here is a grade of 1 in 15, and the grefttcst load that an engine can take, and I have no doubt that is under favor­able circumstances, is 10 tons. If you had a daily traffic of 30 tons on that line that would mean you would have to run three trains a tlay to clear the traffic; but if that were on a broad gauge with an engine that would take three times that load, then you would only have one train.

1209. You have no experience of the broad gauge with 1 in 15 grades ?-No j but it is quite poseible. It is just ~ts possible to go up 1 in 15 on a broad gauge as on a narrow.

1210. You still hesitate about those 2-ft. lines. Do you know Carrajuug ?-I know the country; I have not been at that particular place.

75 T. H. Woodroll'e, 22nd August, 1894.

1211. Is there any place that you could think of where thet·e is a possibility of a 2-ft. gauge?-J do not think it is a matter of locality.

1212. Do you remember any place where a 2-ft. gauge would be better ?-No. 1213. Do you know Walhalla ?-No. 1214. Do you know Gem brook to Pakenham ?-Yes. 1215. Whnt do you think of that liue; there would be very light traffic there-would that be a

suitable line for a trial ?-Yes, I think it would, from what I know of it. 1216. Do you know the Warburton to Croydonline?-Yes, I have been over that country. That

is suitable country for steep grades and sharp cnrves. 1217. \:V ould those he fair illustrations of the difficulties that we, as a Committee, are contending

with ?-Yes, I think they would be. 1218. Would it require special snrveys ?-I am rather inclined to think it would, because those

surveys, I imagine, have been made on a certain system, that is a ruling gradient of 1 in 40, and a curve of 10 or 12 chains radius. If yon adopt another basis with sharper curves or steeper grades that might take you another line altogether.

1219. Do you know of any place now, if we dared construct a broud-gauge line to it, that would pay in the next four or five years ?-I do not know of any.

1220. On the old 5ft. 3in. gunge you think railway constmction is as dead as Julius Cresar ?-I am only speaking of comparisons. I do not know of any place where it would pny to make railways at the cost they have hitherto attained.

1221. By the Chairrnan.-You had this evidence sent to you last week ?-Yes. I read Mr. Rennick's and JYfr. Van de Velde's evidence.

1222. Between his opinion and your opinion there is a wonderful difference as to the power of an engine. Mr. Van de V cl de, speaking of the Decauville system, said a Mallet engine could ascend gradients of J in 15 or 1 in 20 with paying loads of 20 to :30 to us. According to you that must be incorrect ?-That would be quite possible with a gross load of :30 tons; I have not denied that.

The witness withdrew.

At:fiourned to }Vednesday next, at clet•cn o'clock.

[PROCEEDINGS OF THE SECOND COMMITTEE TERMINATED BY THE DISSOLUTION OF

PARUAMENT ON THE 4TH SEPTEMBER, 1894.]

77

MINUTES OF EVIDENCE

TAKEN BY THE

THIRD C 01\IMITTEE.

THURSDAY, 14TH MARCH, 1895.

iJlentbers present:

MR. CA~fERON, in the Chair ;

The Hon. J. Buchanan, M.L.C., The Hon. D. ~:lelville, M.L.C., The Hon. E. Morey, M.L.C.

Mr. Burton, Mr. Craven, 1Ir. J. S. White.

Clement Van de Velde, sworn and examined. 1223. JJy the Chairman.-You ga,vc evidence before the former Railways Committee ?-Yes. 1224. You are a civil engineer ?-I am. 1225. What company do you reprebent ?-I am the representative in Australia of the Decauville

Uompany. I have had very short notice of this meeting, but I have prepared some notes, which I will read.-[ Tlte witness rwd the following paper:-J

I have been requested, through the secretll.ry of your Committee, to come and repeat before you, for the benefit of those who have not already heard them, my arguments in favour of the adoption of the 2ft.-gauge for the new railways of sec?ndary importance to be built in this colony, and also to explain the principal features of the Deeauville system, for w hrch I am agent.

As an engineer, I highly appreciate the compliment, that is paid to me by your Committee in being asked to appear here again.

I propose to-day using some new, and, I believe, very strong arguments, in favour of my views, and will present the whole thing in a somewhat <lifferent way, so that those among you gentlemen who have already heard me may perhaps take a new interest in what I am about to say.

I will divide my new evidence into two chapterB. In the first I will simply speak as an engineer, and in the second as an engineer and agent for the Decauville Company combined.

What is wanted in this country, as in any country on the globe, is not so much cheap railways as railways that will afford cheap carriage. To ohtnin cheap carrittgc three things therefore must be steadily kept in view. lst. The interest on the capital, and, consequently, the capital itself. or, in other words, the cost of construction of a railway and its equipment must be as reduced as possible. 2nd. The <~ost of maintenance of the permanent plant and rolling-stock must also be kept down to a minimum ; and 3rd, and perhaps above all, the daily general expenses must be kept as low as pos­sible. A railway, on these points, does not differ from auy other industrial or commercial undertaking.

Thus, in all eases, the best railway will be the one that costs the least, •Yhose cost of maintenance is most reduced, and which will cost the least to work. From what preercles, we may conclude, without fear of error, that the contemplated traffic must be our only guide in the construction of a railway, and as the amount of t,raffic differs, so'the railways must differ, and this will be true as long as we are satisfied that railways must be considered as commercia,! unacrtakimrs.

In u']y previous C\·idence 1 have m:. de the following statement :-"That any gauge milway must be built in view of the maximum traffic that is capable of," :1-nd l added-" Doing otherwise would be equivalent to building an hotel and closing half the rooms up." waste of money wnnld 8trike any one in this case, but in a railway, not built in view of the maximum trutlic gunge is capable of, the cmpt,y rooms or waste of money is not so apparent to tlie general public. The waste is there nevertheless.

There is, however, a gauge that escapes that condition, and that is the 2-ft. gauge. Why is that so? Because our present knowledge a,nd experience do not allow us to red nee that gauge any further in the ease of public railways, having in view the safety of !lJC passengers.

A :? ft. gauge r~ihnty. therefore, must not be built in view of the maximum traffic its gange is eapa!Jle of, but simpl~· in view of the maximum contemplated traflic, and for that reason gradients which are excusable on a :!-f't. railway are not so on broader gauge railn·nys, not even on a ~ft. 6in. rnilway.

The point which I have jnst tried to explain is of extreme import>mce, but, 1 admit, very delicate, and if I am not well understood, g-entlemen, ' wish ynn wonld int<·rrnpt me here, and 1 woul<l then try to illustrate the case. Presuming the above assertion to h<; eorrcet. h shows thtlt tlte ouly rational way to reduce the cost of a railway consists in reducing its ~auge, and rts the Clli>ibl inYcstc;l in the <'(lllstrncti,n• and equipment of :t railw:~y must be in proportion to the traffic, it follows that the traffic must rc;:ulate the The trnth of tlut principle in railw><r construction is to-day well e;;tahlishe<l, emJ,Cf[l>ently, .\ir. Chairman :md wiH•n we compare the cost of two railways of different gauges, to be fair we must thdr <:o,4 wh<·n lmilt. both •ie•v of their maximum traffic, ami if u11der sueh eonditions we compare the f'os:- pcrff'et. :Le ouc on i!Ic• fJft. 3In. gauge, and the nrher ,on the :!-H. gnuge, we will find thar the proportion of re::: tin: eost evi,Jcnce. viz., the to one, is a long wny too small. For in,.,tance, a surrey 'e, h.:i:lg ''""l''eled ''"" to Orbo;t. the cost of which is to be compare<l with the c•,sr of a 5it. :Jiu. line ('m>m'et tlH' stHl<e I rc-grc't that so far there is no oflicial information on the results, IJUt I hen) only of the wcicdll of rnils to be l'<?COlllll!endcd in each ca,;c,, in tlw :Jin. li!He rhe as low nq atHl for the 2-ft. it will he a 40-lh. rail. it is to saY that it •ril: he as nmeh '" P'"sibl.'. This is c•dtlcntly and it makes me fear that the forthcoming information •Yill HOt allow you (o thl' case on ih nwrits.

Besides. the wa.\· this hnsiuess hecn started a'JSunl ft'C>lll the beginniug. \Vben the construction of a branch rnih,~~iY i:-; contt·mp::lt('(l. ~lH; tiling to b:• tlont· is to lmt!(e an htqn!ry about: tile uature aud quantity of traffic, and the prohablP expan,io:J d tlmt trnilie in tlle ftHnre. This being· known, and if it is rlcsirc•d to make compariwn between the cost of a ~-ft. line and a 0 fl. :3in. line, shoal([ he made for the two lines. The narrow-gang"' engineer will then l.Je en a bled to rlw w0ight of tiw mil' to used, the gradients and cnrves "'hi eh it is wise to adopt in view of keeping down the working expenses of this railway in good proportion to the probable returu~, &c.; and the broad· gauge engineer will then also he able to determine the same points.

C. Van de Velde, 14th Ma.rcll, 1895. 78

Then, Mr. Chairman, we wilt be able to compare not only the cost of lines, but we will have a very fair idea of the cost of maintenance and working as well. And if the estimated traffic is small, the results of such inquiry will allow you to cmnP r<> a cnuclusion in five minutes. The results of the investigation made on the present basis, or rather on a want of b<:>L,, cu.n only lead to confusion.

At this point I beg once more to draw your attention that in my advocacy of the 2·ft. gauge, I only wish to recommend it for branch railways of secondary importance.

No advocate of this narrow-gauge would dream of recommending it for connecting large centres of population and commercE>. In that case a break of gauge must be considered as equivalent to a national calamity. Narrow-gauge railways of 2-ft. cannot compete witiJ broad-gauge milways when the traffic is important, but, vice versa, broad-gauge lines cannot compete with 2-ft. lint's when the traffic is feeble.

In my former evidence I have endeavoured to show why the 2-ft. gauge railways can be built so much cheaper than broad-gauge lines. To-day, however, I will simply ennmerate the reasons.

1. 'l'he rolling-stock: being considerably lighter, the weight of the rail can be reduced in the same propGrtion. 2. Much less ballast is required. 3. The sleepers being smaller are less expensive. 4. Bridges, viaducts, culverts, and tunnels are all reduced in importance. 5. The formation of a 2-ft. line being so much smaller, the volume of earth or rock to be removed in case of cuttings

or embankments is also much smaller. 6. The 2-ft. gauge permitting the use of curves as low as 66 feet radius, most of the works of art, such as tunnels,

bridges, viaducts, and also costly cuttings and embankments, can be avoided by running the lines round the hills on inexpensive side cuttings. The running of the lines round the hills has, however, the drawback: of lengthening the lines, but as a rule compensation for this is found in a reduction of gradients in the longer line, and, consequently, in its working expenses.

Such are the principal reasons ;.-hy a 2-ft. railway can be built so much cheaper than broader gauges. I have now, Mr. Chairman, an important statement to make, one that is the result of the deep studies I have made of

this gauge question for some time past. The discussions on the relative cost of 2-ft. and broad gauge, and all the surveys that may be made to prove their larger or their smaller difference in cost, according to the personal aspirations of the engineer who may be put in charge of such work, all this, in ruy humble opinion, is a waste oftime and money. The statement is as follows:-

Supposing the case of two points to be connected by a railway line, the contemplated traffic being small, if it could be proved that a broad-gauge line, of even perfect construction, does not cost more than a 2·ft. line, then still preference must be given to the narrow line.

This is a bold 8tatement to make, but I here challenge any railway engineer in the world to prove that I am wrong. Why is that so? Simply because the traffic being small, the 2-ft. g;mge line will answer all requirements, will cost less in maintenance, and the daily general expenses will be considerably reduced.

\Ve sometimes hear of the world going backwards, and therefore it may happen perhaps one day that some engineer may prove that it cost less to build a 5ft. 3in. rail way than a 2-ft. one. That is the only fear I have for the upsetting of the views I have just expressed. I will say no more, therefore, on the question of cost.

The second point which, as I have statcJ, must be kept in view to secure cheap carriage is that the cost of maintenance of the permanent road and rolling-stock should be k:ept down to a minimum. This means that the rolling-stock must be of the best workmanship, and that the line should be well laid down; that no false economy should be made in the quality of sleepers, in the quality and quantity of ballast ; that the drainage should be effective, &c. All things being equal, it is evident that it will cost less to maintain a narrow-gauge raihvay than a broad railway. This seems so clear that I do not think there is any demonstration required.

The third point it is to say the necessity of keeping down the general expense to obtain cheap carriage requires more attention.

Many engineers in Australia (I say engineers in Australia because that race of engineers is quite extinct in Europe) maintain that the general expenses per ton of goods and passengers are greater on a narrow-gauge than on a broad-gauge line, and such is one of their great arguments against the adoption of the narrow lines. I will explaiu where their error comes from. They compare figures without comparing circumstances, and at the example of the Englishman who, arriving in Paris and finding no soap on his washstand at the hotel, conclutles that Frenchmen don't wash, those engineers also draw wrong conclusions from what they see. ·

I,et us take a commercial example, which we will afterwards apply to the case of railways. Supposing two beet·SU!?ar factories (I take the example of beet-sugar factories, because this is the great coming

industry in this colony), the first is able to work: up every day say 250 tons of beet, and will produce 35 tons of sugar. The second is able to work 1,000 tons of beet, and will produce 140 tons of sugar. It is evident that the ton of sugar will be produced cheaper in the second mill than in the first, because the general expenses per ton of sugar will be llrnaller in the larger mill. Producing on a large scale is the secret for producing cheaply.

For exactly similar reasons the broad-gauge railway which transports 1,000 tons of beet per day ;vill be able to offer cheaper freights than the narrow-gauge railway transporting only 250 tons a day will be able to offer. So far, as you see, I agree with my opponents. But let us continue the reasoning, and suppose that the larger sugarfactory, capable of working up 1,000 tons of beet a day, is not able to find those 1,00\J a day, and can only lind a supply of 250 a day. In this case the larger factory will produce exactly the same quantity of sugar as the smaller one, but the cards will be turned, and it is the smaller factory that will produce the cheapest, for the simple reason that the capital account, the cost of maintenance, and general expenses will be greater in the larger mill when it produces only the limited qmtntity of sugar, viz., 35 tons a day.

For exactly similar reason~ the narrow-gauge railway, which trR.nsports 250 tons of beet per day, may earn profits, while the broad-gauge line, able to transport 1,000 tons a day, but receiving actually only a supply of 250, will probably lose money. Is that clear, gentlemen ?

\Vhether a railway line is built on the 2-ft. gauge or on the 5ft. 3in. gauge, it will make no difference in the amount of traffic it will draw.

In selecting their examples for comparison, engineers opposed to the narrow gauge ought to take, not the ease of two railways which are each built in right proportions to their traffic, but tlwy must tak:e the case of two railways where the amount of traffic in passengers and goods is the same, and, us pointed out several times, where the traffic is moderate.

And how could it be otherwise, as is it not eTiJcnt that if a 10-ton locomotive is sulficient to carry the traffic, it will consume less coal, less oil, and less of everything than a 30-ton locomotive would do? If a wheel or an axle of a small locomotive break, is it not evident that it will cost less to replace them than if a similar piece ef a larger engine break ?

You arc also aware that the rolling-stock of 2-ft. gauge railways is much lighter for the same carrying capacity than broad-gauge cars and waggons, and that therefore much less dead weight is to be hauled by the locomotives. This is a source Gf great economy in fuel. But all these tbings are now so widely known that it seems almost ridiculous to repe&t such statements. I hope, Mr. Chairman, J,hat I have proved to your satisf,tction that th9 cost of a narrow-gauge line is considerably smaller than a broad-g·auge line, and that wheu the traffic is moderate the cost of their maintenance and work­ing is much smaller per ton of goods and passengers than would he the case of a broad-gauge performing the same amount of traffic.

There is, however, another point wllich you might wish to he proved, anu that is that 2·ft. railways are capable of meeting the req nirements of th<; traffic. It is at this point, Mr. Cuairman, that I wish to mention for the first time the name of Mons. Decau vi lie, who may jnstly he called the promoter of the 2-ft. gauge railways. And in speaking now as agent for the Decauville Company, l will not ce:tse to ~peak :ts a professional engineer, considering that as a pro­fessional engineer [have no hesitation in t•ecommending the Decauville system as the very b8st system in existence, and I am sure that in this respect I will not be contradicted by any engineer who has had the go()d fortune of seeing the Decauville at ·work.

Before showiug the capabilities of these 2-ft. railways, I wonld wish to explain the principal features of the

systcmits great superiority resicle,s almost exclusively in the particular desi!,Pl of its metal lie sleepers and in the fact t,hat the rail is riveted on them by powerful machines under a pressure of 140 tons. It can almost be sairl that rails and sleepers make only one solid piece of steel.

[The witness d1·ew on tlte blackboard a diagram of tlze Peclwt's patent sleeper.]

79 C. Van de Velde, Uth March, 1895.

The riveting of the rails on sleepers with such powerful machinery gives to the system, it is almost obvious to say, an extraordinary strength, and when once laid down and properly ballasted the line requires very little looking after, and the cost of maintenance is reducefl to its extreme limit.

Referr;ing now to the construction of a properly laid down railway of ll.ny gauge on wooden sleepers, the least that can ?e done IS to raise the ballast to the top of the sleeper. I know, of coarse, that in some places in this colony they are makmg milways without ballast~simply placing the sleepers on the surface.

1226. By iJJ.r. Craven.·- They use the stuff taken from the cuttings for ballast?-Yes, and the first heavy ruin washes it away.

In the Decauville system, owing to the little height of the sleepers, much less ballast is required. Through the shape of the sleepers, the ballast is, so to speak, caught under it, lateral displacement becomes impossible, and a much greater stability of the line is thereby secured. I hope we will soon have a line of this description in Victoria, and you will be agreeably surprised with the smoothn<:ss with which the cars ruo over it. You will undcrst<l.nd without further explanation that this must have a beneficial effect on the maintenanee of the locomotives and other rolling-stock in good order.

Such are the principal features of the Decauville permanent line. The advantages claimed for the system eau be resumed as follows :-Greater stability, greater effectiveness, greater dur.tbility, and reduced cost of maintenance of road and rolling-stock.

I now propose, Mr. Chairman and gentlemen, to give you an idett oi' the capabilities of these 2-ft. railways, and this brings me first to a description of the locomotives.

Iu my last el'idence I said we had three types of locomotives, but a fourth type has just been added latel_y. The first type only weighs 5 tons empty, or 6.J: tons in working order; it has four coupled wheels ; the hind wheels are

driven by the connecting rods, and the fore wheels by the coupling rods. The wheels are 60 c., or about 2 feet in diameter, made of iron with steel tires. The fumace is of red copper, the boiler of plate iron. The steam cylinders are 6 inches diameter and 12 inches stroke, the steam pressure 12 atmospheres. The water tanks are placed on the side of the eugiue and hold llO gallons, and it carries 6 cwt. of fuel. A locomotive of this description can travel with its provision of water and fuel a distance of from 8 to l 0 miles, >tn<l haul a load of from 20 to 25 tons on gradients of I in 66. Two of these loco­motives a.re in use at the gas works, in .Melbourne, and a number of them on the Queensland and Fiji sugar plantations. During a recent visit in Queensland I have seen one of these little locomotives haultl train composed of 60 trucks and about 120 tons of sugar cane.

The next type is the" Mallet" patent articulrLted compound locomotive. This weighs 9 tons empty, 12 tons in working order. The principal characteristic of the engine is that it iB made on the compound system to save fuel, and is designed for curves of 66 feet radius ; it has four steam cylinders and eight driving wheels, the second two coupled axles are fixe cl to the body of the locomotive, but the fore ones form an independent bogie. Direct steam m ay be sent into the four cylinders, and this gives the engine spare power when necessity sometimes arises, when heavy loads are to be hauled on long or steep gradients. The engine can tmYel a distance of from !6 to 20 miles and haul lOO tons on gradients of 1 in 40. Such au engine would be quite sufficient for the traffic actually carried on most of the Victorian branch rail­ways.

The third type is the duplex engine, improvetl by Messrs. Pechot and Bourdon, weighing JO tons empty, 12 taus in working order. It is characterized by the junction of two boilers with one central furnace. lt is placed on two bogies, and direct steam is applied in the four cylinders. This engine is especially designed for very hilly country. It will ascend gradients of 1 in 12~ with loads of.l5 tons, and travel mund curves of 50 feet radius at slow speed. On flat country it can haul a load of 230 tons at a speed of 14 miles, and can travel a distance of 20 miles with its provision of water and fuel.

During my former evidence I have made the suggestion that a comparison be made between the capabilities of the two latter engines and the traffic carried on some Victorian branch railways iu boom times. I believe that your $Ccretary, 1Ir. \Vatson, has prepared such comparisons, and they will no doubt inter<:st you and facilitate your conclusions.

1226A. When ascending n gradient of 1 in 12~ do you use a smooth rail or a rack rail ?-A smooth rail. Of eonrse we would uever use such a gradient as that, but the engine could do it.

1227. For wlutt distttnce ?-That is simply a question of the quantity of fnel and water on the engine. It will run as long as it has fuel and water. W c would never design such a railway, of course.

1228. Then is it not useless to put t.hat in ?-No, I think not. It sho1Ys the power of the engine. The new type of locomotive lately de~igned by ~rons. Decauvillc is to he used 011 gradients that ordinary locomo­

tives cannot ascend. It wei1;hs 12 tons 12 cwt. empty, and 15 tons 6 cwt. in working order. It has four cylinders and central cog wheel. Up till lately, many engineers maintained that this system eouhl not be applietl to the 2-ft. gauge. The cout.rary is proved to-day. !t has two coupled axles and one mdial axle. It is fitted with screw brakes, acting on the bearing wheels, friction brakes acting on the cog wheels, and compressed air brakes formed by the cylinders. The feeding of the boilers is assured by two injectors.

I will not take up yom time, gcnU"men. in making a {}escriptiou of the goods trucks and passenger cars. I will only say that the comfort of the latter is as perfect. as can be-l would even superior to what you now get on some of your branch Iincs. Carriages of all descriptions and all sizes are ruarlc meet different traffic requirements, and the climates of different countries. As far as the g-oorl~ trucks are conceTned, I will say they are made of the same carrying capacity as those of big railways, if requir0d. However. as these railways are int0ndcd for bmneh lines, where traffic is modemte, there has not been, so I:u, necessity for building trucks of above lO·ton carr:ving capacity. Jt is only in France, where this system has been adopted for the military railways, that tru~ks have been built for the carriage of heavy gnus of 48 tons. There is no weight that cannot be carried on a 2-ft. line as well as on any other gauge line. The only secret is to multiply the number of axles of the truck. There is no difficulty either in the transport of cattle or horses. Th·e loading into the special trucks designed for this purpose is even more eouvenient than on broau-gauge railways, where platforms are required. fn this case the truck is lmvered between the two bogies, ancl the animals walk into the trucks straight from the surf<tcc of the grounu.

1229. Colonial bnllocks could not be got into them as easily ns that ?-lVhy not ; what is tho dif:erence 1Jetwecm a French bullock and 1 he colonial bnlloek?

12:30. Here there i8 a race running from the yards; the door of tl1e truck is dropped and the bullocks are run along the race into the truck'?-You could do that in this case also.

1231. Do you think cattle can he carried on a narrow-gauge lhw at all ?-It is done in France. Mr. Dean, the Engineer-in-Chief of New South Wales, has seen it done, and is quite satisfied that it can be done.

1232. By the Chai?·man.-Horo we put nine ami ten bullocks in a truck-it would 1akc a long time to truck a mob of cattle if each truck only hehl four ?-Then i£ tho trade is lnrge enough you must builcl a broad-gauge rnilway; this railway is only int:enJe,1 for a small trnffic, &nd if there are 200 or 300 head of caltle to be carried every day, bllild n broad-gauge line.

The great ad vantage of a uarrow-gr.uge line is the facilities it to private persons to connect themselves with the railway. A 2-ft. gnnge railway costs very little to coustrnct. On private sidings wheriJ no locomotives are noel! the weight of the rail can be rednced to Lt lb~. per yard, and it will then COt't only about £300 per mile to put a line down. Yon caa run that line into stores much more couv<3niently than you eau a broad-gauge line, and with it you can go to the very spot where the goods are ]yino·, which you e:1nnot do with the broad-gallge, so in thn.t way you save one handling of the goods. Tak~ tbo example of where sleepers arc cno)n the forest for the big railways; they have to be carried on

0. Van de Velde, 14th Mareh, 1896. 80

carts to the railway, but with this system yon can take the line to the place where the sleepers are, by the use of a few portable sections, which two men can carry, and that}rcpresents an enormous saving. That is a great advantage of the 2-ft. line, uecause even when the gauge becomes 2ft. 6in. it is not so practicable, it is heavier, and if the line il:l 3-ft. it is impossible altogether.

One of the great arguments of the opponents of the 2-ft. gauge is that railways must not be built for to-day only, we must build them for the ft1ture. That is quite true. I do not advocate building railways for the present traffic only-in many caseb a bicycle would be sufficient for that. I have shown you by the power of these locomotives what is the extent of the traffic that they can do. The opponents also say-" We do not know what the traffic in a new district will be in the future." But I maintain that with the present state of civilization in Australia we do know what the futtlre of a railway will be. \Ve may perhaps make one mistake out of a large number of cases-if we build a narrow line somewhere, a Coolgardie may be discovered on that line, and we will :find that we have made a mistake; but that is only once, and it is far better to make one mistake like that than to have constant mistakecl made in every case in the building of broad-gauge railways for which there is not sufficient traffic. I have Leen shown a document, which I presume has also been laid before the Committee, referring to narrow-gauge linea in America. In that document it is ~:;tatcd that a large number of narrow-gauge railways, not 2-ft. but 3-ft. and 3ft. 6in. gauges, have been taken up and replaced by broad-gauge lines. I would advise you not to pay too much attention to what occurs in America. As you know, America is the country of liberty in the construction of railways-there is free competition. Between two places there may be six different lines of railway. It is evident that under those conditions the TJarrow-gau6'e line has no show-not that it cannot carry the traffic cheaper, because it is evident that, in America as elsewhere, a 2-ft. line can carry a moderate traffic much better and cheaper than a broad-gauge line; lmt if there are two railways connecting the same points the public will naturally patronise the ],ig line, so the little line will have no show, and they will have to take it up. You do not see that happen in countries where there is a State monopoly ; you do not aee it in Europe or Austrnlia. \Vhat takes plaee in America need not Le tt~ken into cpnsideration.

1233. \Vhy do yon favour a 2-ft. gauge as against a 2ft. 6in. or 3-ft. one ?-Because the 2-ft. gauge is the most elastic. It is proved that, as far as safety and mmying eapacity are concerned, the power of the 2-ft. gauge b quite equal to the 2ft. flin. or even a larger gauge, and that you can do with a 2-ft. gauge what you can do with a 2ft. 6in. On the other hand, in a mountainous country like Gippsland, you can run the 2-ft. gauge round a curve of 66 feet, but with a 2ft. 6in. gange that curve would have to be doubled, so the 2-ft. gauge gives you facilities that the 2ft. 6in. gauge does not give.

1234. Then why not adopt a 1ft. 6in. gauge ?-Because it is provcll that it would not be safe; 2-ft. is the lowest to which engineers have dared to go, taking into consideration the safety of the passengers.

1235. vVould it be ~afe to go to that extreme in a mountaiuons country ?-It is the safe extreme. In France we have 16-in. railways ; you will sec one at the Bois do Boulogne which has been working for years and years, earrying passengers from the outside of the grounds to the Jardiu des Plantes. It works very well, drawn by hon>r's, but that wonlJ not suit 11 State railway.

1236. By J}fr. Cravmt.-It is a tramway praetieally?-There is no tlifference between a tramway and a railway, except that the tramway runs ou the public road.

1237. By the Clwirman.-'l'here aro many mountainous districts in this colony where it is hope­less to think of constructing railways if they :>re to cost Ei,OOO, £6,000, and £7,000 a mile, but if a narrow­gauge line can be constructed for much less money there would he some hope of reaching those districts. vVe are anxious, therefore, to find ont the exact diiference in cost between a narrow gauge of say 2 feet and the broad gauge that we have in Victoria ?-I say the di1ference would be five to one when both are properly constructed.

1238. Do you say that a line that wonhi cost £5,000 a mile on the broad gauge would only cost £1,000 a mile on the mllTOW gauge ?-Yes.

1239. Have you ever seen in any conntry as nmch difference as that ?-I have seen a much greater difference. In France the broad-gauge lines cost £20,000 a mile, but most of them are double tracks, and land is dear, which raises the price, but the narrow-gauge lines only cost £1,800 a mile.

1240. \Ve have to conteml against the opinion tlmt there is yery little differenc~ in the cost of the two gauges ; that where a broad-gauge line co,<t,; £.5,000 a mile, a narrow-gauge line will cost £4,000, and we must be fortified with anthority Lefore we c:'d say we are satisfied the narrow-gauge will opJy cost one­fifth of the other. \Vhero wonld you make the s:tYing-is it in the curves or the gradient~ ?-The saving is in almost every item.

1241. That is a ~imple statement. Have yotl kuown au instance yourself where a narrow-gauge line has gone up a motEitain on a gradient of l in 15 or 1 in 12~ ?-\Vc would avoid that; it can be done, but it would be eommerciall v unsound.

1242. Yon >vonl~l avoid that by curving ?-Yes, which you c11nnot do with broad-gauge railways. 124:3. Yon woulrl make the distance ln many instances douhle the leugth between any two points ?

-Yes, uut it faciJil;ato~ the gradients and diminishes the working expen:;es. 1244. Yon would have to eou,;trnct two mile~ of narrow gauge for one mile of broad gauge ?-That

is not the proportion, hut in mourlt.ninous countries mtrrow-g~.ngc lines are always longer than broad-gauge lines. On the other hnud, illStend of lmving gradicllLH of 1 in 30, yon woulJ have them 1 in 80. In the Festiuiog ntilway in \Yale;;, which ruus through monntainons couutry, the steepest gradient is l in 80, which means an immem;e reduction in working expense~.

] 24.5. Ilow do you nceonut for the working expenses being so much ic'<s with the narrow gauge?­Because if you lmve a :30-ton locornotiYe on a broad gange pnlliug only one-tenth of what it can pull, there is an immense waste of moth·e power, while if the same weight is pulled by a locomotive that is only intended to pull that weight ail its motive powci' is utilize(l, The utilization o[ motive power is one of the great secrets of modern indnstry. If we, ou the CoHtincnt, beat yon Briti8h in many a.n industry, it is on account of that. [r, is nnt becnn~e oHr wageH are cheaper, as that is a very ;,mall item; it is in the utiliza­tion of motive power that we heat yon, arHl nothing else. That, combined with producing large quanti­ties, is the great secret of motlPm indnsLry.

124'). But a small eugiue on the wtrrow gauge will require as many men to work it as an engine on the broad g;tuge ?-To a certain extent that is so; no donbt the driver of the locomotive earns the same wages, but in France it is found that small railways really require much less labour than bigger ones,

81 0. Van de Velde, 14th March, 1896.

because they are protected by their small size. As soon as a big railway is made the people in the district want everything that a big railway can give. When the railway is first built they are satisfied, but after a while the Minister has to receive deputations asking for the construction of a station here and another there, and so it goes on until in a few years that railway will cost a lot of money. In France it is found that the small size of 2-ft. railways and their modest appearance is a protection in that respect.

1247. Have you been in Queensland ?-Yes. 1248. Have you seen the narrow-gauge lines working in the plantations there ?-Yes, a large

number of them are on the Decanville system. 1249. Are they a success ?-An immense success. 1250. That is level country there; but do they answer the purpose ?-They only cost about £800

a mile. 1251. JJy iffr. J. S. White.-Does that include rolling-stock ?-No, but those railways built for

industrial purposes are not the railways I would recommend for State railways. They are principally made for carrying goods, and if you have the misfortune to sit on a truck you will feel it ; they are very rough. There is not too. much ballast, and they are very badly laid down. That is the sort of railway I would compare with the broa(l-gauge line they are putting down at £2,000 a mile. I would put the same line down on the narrow gauge for £800 a mile.

1252. JJy tlu1 Cltai!J'man.-\Ve know as a matter of fact that in very mountainous districts now we can construct a broad-gauge line fo1· £5,000 n. mile, allowing a moderate amount of ballast. According to the proportion you have given us, a narrow-gauge line in the same kind of country would cost £1,000 a mile. That being so, how do you account for the fact that the railways on this level country in the plan­tations cost £800 a mile ?-I do not think any railway can be built in mountainous country for £1,000 a mile. I have already stated that when you compare the cost of two railways you must compare those two railways built perfectly, with the required ballast, and the scientific curves that those gauges allow, with the gradients in proportion to the traffic and all that, and you will then see that the proportion of 1 to 5 will hold good, but if you are going to make a cheap broad-gauge line and compare it with a per­fectly made 2-ft. line the difference will not be so great.

1253. A certain line was surveyed in the country, and the estimate of the cost was £18,000 a mile. Since then the price of everything has come down, and within the last few months the Engineer-in-Chief has given an estimate for that same line at £6,000 a mile, or one-third of the original estimate. How much do you think a decently constructed 2-ft. line could be constructed for in that broken country ?­I should say £2,000 a mile, but I contest that the broad gauge cannot be built in the same country for £5,000 or £6,000 a mile on the same principle.

1254. Then how do you come to the eoncluslon that it will only cost £2,000 a mile on the 2-ft. gauge ?-The most expensive line we built in France cost £2,000 a mile, including rolling-stock and everything, ao if I put the cost down at .£2,000 a mile in a mountainous country, where land is of no value, I have reached the maximum.

1255. Do yon include rolling-stock in that estimate ?-Yes; that is what it cost for the railway from Dives to Luc-sur-1\rfer.

1256. How much rolling-stock is provided for that sum ?-There were seven locomotives. 1257. :For how many miles ?-Twenty. There are ten trains a day in each direction. At the time

the line was constructed the rolling-stock consisted of one first class closed passenger carriage capable of seating 56 passengers; one open and eight closed second dass carriages; four close<l mixed carriages, com­prising first, second, antl third classes; and four third class closed carriages. There were two waggons for the transport of cattle, and 31 open goods waggon~, making a total of 51 vehicles and seven locomotives. That was at the start of the line. The line has since been extended to Caen, the capir.al of Normandy, the total length being now 40 miles, and I presume the rolling-stock has been increased considerably.

1258. By iffr. Cl'aven.-The line from Mount .M:oriac to Wensleydale is a little over 11 miles in length and cost nearly £40,000; the revenue is £812 per annum, working expenses £7,)5, interest on capital £1,600, approximate loss per annttm £2,000; Birrcgurra to :Forrest. 20 miles, c~st £147,000, revenue £1,400, working expenses £2,400, interest on capital £.'),900, approximate loss per annum £7,000 ; .Morwell to North .M:irboo 20 miles, capitnl cost £153,000. revenue £2,000, working expenses £3,000, interest on capital £6,000, approximate loss per annum £7,000. You see ifthose lines were built for nothing a lot of money would be lost in simply running them, because the working expenses are so much more than the revenue, and yet there are a mtmbor of people who demand tuat we should tap that clas~ of country with railways--what would you do in a case of that sort ?-It is evident that if I had money to invest I would not invest it in a railway like any one of those you have quotell, but it may be a matter of policy; the country may think it advisable to make a railway there eYen though it may result in a loss. In that case I should say that those districtK are extremely suitable for 2-ft. railways of the lightest doscription.

1269. The loss would be less on the narrow gauge than on the broad g-auge, and if you make a bad speculation, the less money you sink in the first instance the less you lose?-Y cs, necessarily so.

1260. You have said you do not think the smveys for the Orbost line were made as they ought to be ?-The surveys may be all right.

1261. Have you been over the route ?-No. I say that when a railway is going to be built the first thing to do is to inquire into the probable traffic and the probable expansion of that traffic. That being ascertained, if you wish to make a comparison between the two systems you must ask a narrow-gauge engineer to build a 2-ft. line to meet that traffic and no more.

1262. Are not our engineers as capable as any engineers in any part of the world to make a survey for a narrow-gauge line ?-I know that you h1ne good engineers in the service, but it requires train­ing to survey the route for a narrow-gauge line ; it is a different thing altogether from making a survey for a broad-gauge line, and therefore however capable your engineers may be in their present specialities, they might not be able to survey a line properly for a 2-ft. lino.

1263. lly the Chai:rman.-Can the steel sleepers be put Jown as cheaply n- \,. ·•1 n much cheaper, and they will last for years and years.

1264. JJy iffr. Crm:en.-The survey of that line has been fairly ('arried out by an experienee:1 engineer ?-I dare say the survey is all right, though I saw there was a viaduct provided on it, and when I asked if that could not be avoided, I was told it could by going round a. lake.

NARROW GA.UGE.

C. Van de V elde, 14th March, 1895. 82

1265. Can you give the figures as to what it costs per ton per mile for haulage on the narrow gauge? -I have no figures to give you, but it will differ in every country, according to the cost oflabour,eoal, and other things, and also according to the kind of traffic. You will find all that information available in the Department.

ably. 1266. Can you not give us an approximate idea of the cost ?-No, I cannot. It differs consider-

1267. You claim that the expense of maintenance is very much reduced on the narrow gauge­how many gangs of men would be required to attend to a length of thirteen miles of narrow gauge ?-One.

1268. How many men would there be in the gang '!-I suppose three men. 1269. Would you be surprised to know that we have railways in this colony on the 5ft. 3in. gauge

which are maintained at no greater expense than that '?-Are they kept in good order? 1270. They are ?-Then I congratulate the colony on the circumstance; but there is no traffic

perhaps. 1271. In cuttings you put your sleeper on the bottom and it remains there with the ballast, but

when you are on embankments do you not find a difficulty through settlement ?-Jnst the same as with an ordinary railway. Until the embankment is settled the difficult.y is the same with our sleeper as with the ordinary sleeper.

1272. Taking the line over the swamp near Port Fairy, you will find some of the metal 3 feet down. When the line has to be continually worked up to allow for that have you not a difficulty with the hollow sleeper ?-Not the least; you can hammer the ballast under it just the same.

1273. What is the primary cost of the sleeper ?-We sell the system at per running yard, sleeper and everything included.

1274. The diagrams you have produced show only one man on the engine; is it usual to run them with only one man ?-As a rule there is only one man.

1275. Is that safe?-We find it safe. On the Luc-sur-Mer line they have two men I fancy. 1276. They have the same as we have here-two men on the engine and the guard?-Yes. 1277. Y on maintain that the small engine is more economical than the large one, but suppose your

light engine is heavily loaded, does not the cost of fuel rise at a very much greater ratio then, and the lighter engine become an extremely expensive engine to work ?-We never work a locomotive beyond its power.

1278. If it is not of sufficient power to carry a little extra loading you have to run an additional train?-Yes, and so it is with the bigger locomotives on broad-gauge lines.

1279. But in running a large locomotive with a light train the fires are banked ?-That shows that your locomotive is not worked up to its full power, and then there is a loss of motive power.

1280. How much does that amount to ?-It is enormous. 1281. Do you know anything of the American system of dealing with a break of gauge, by which

the trucks are lifted by cranes from the narrow gauge and placed on new carriages at right angles to their former position on the other gauge ?-No, I do not know anything about it.

1282. Do you know the line from Hamley Bridge to Broken Hill in South Australia?-No; I know the principles upon which it was built.

12H3. That line developed a large mineral traffic?-Yes. 1284. In a case like that, where you knew you were going to develop a large mineral traffic, would

you keep to the narrow gauge or build a broad-gauge line right through ?-I would prefer to have the one gauge right through-it all depends upon the quantity of traffic.

1285. By the Hon. D. ~lfelville.-You are aware that we are now constructing lines in themallee to pay; there is the Dimboola to Jeparit line, 23 miles, cost £1,554 per mile; Boort to Quambatook, 22 miles, cost £1,7 ii2 per mile; Beulah to Hopetoun, 16 miles, cost £1,899 per mile, and so on-it is found that those railways pay. If you had had to constmct those lines, knowing thnt the traffic would be wheat, wonld you have broken the gauge, and started into the rnallee on the 2-ft. gauge ?-To answer that question I onght to know the amount of trnflic.

1286. The lines are paying now ?-It is not a question of whether the lines are paying, because they might pay better with the narrow gauge.

1287. Our engineers estimate, and we will accept their estimate for the sake of argument, that they can construct a narrow gauge for one-third less than a broad gm1ge. At 4 per cent. that woulJ amount upon 30 miles of railway at the prices I have quoted to £720 per annum in interest '?-I say if the traffic is large make a broad-gauge line, but if it is small and not likely to develop ·within a reasonable number of years, make the narrow-gauge line, even if it costs as much as the broad gauge, because you will save money in the working expenses and the cost of maintenance. That is the great statement I have made to-day.

1288. Those lines are fully occupied during the wheat season with our own surplus broad-gauge stock ?-Then the traffic must be very great.

1289. Seeing we have sufficient surplus rolling-stock just now to equip 1,000 miles of line, would it be wise, for the mere saving of £720 on every 30 miles of line, to break our gauge ?-If it were only for the saving in capital I would be of opinion that it would not be worth while, but there are other considerations to be kept in view. If you say the railway is fully occupied it shows there is a great traffic; in that case stick to your broad gauge.

1290. What I mean is, that the farmers were complaining that they could not get trucks fast enough to take away their stocks of wheat, so we must assume the lines are fully occupied during a certain season of the year ?-But can you give any idea of the quantity of wheat to he carried within a certain time?

1291. For three months in the year they arc worked night and day ?-What is the amount of tonnage carried ?

1292. Close on 500,000 bags of wheat come down the Hopctoun line; there are 300,000 hags from lVarracknabeal itself?-To be transported within what time?

1293. As fast as it can he got away-it generally takes from four to five months ?-Taking it at five months, that is 150 days for 50,000 tons, which is a little over 300 tons a day. I am convinced that a 2-ft. line would carry that quantity more economically than a broad-gauge line, and the traffic would be done just as well. You can put as many bags on the narrow-gauge trucks as on the broad-gauge trucks.

83 C. Van de Velde, 14th M!l.l'Ch, 1895.

1294. ·what wages would you pay-in this country you have a union driver and a union stoker, and things of that sort to contend with ?-The wages woultl be the same o£ course, but the cost o£ maintenance and working will be smaller.

1295. They burn mallec roots up there, antl we will leave the grease out and confine our calculation to the little engine running up there autl doing its utmost-with those fixed wages £or the driver antl stoker, can you do it any cheaper than it is done now ?-That is my impression ; I can only calculate it approxi­mately here, but on some sugar plantations they take 500 tons a day.

1296. What rate do they charge per ton per mile there ?-They do it on their own account. I cannot tell you what it costs on the spur of the moment, but I can make the calculation; there will not be much difficulty in arriving at a conclusion.

1297. \Ve are practically ignorant of what this little engine can do, and we would like to have some information on the subject ?-One of our locomotives can draw 300 tons, probably it will draw 330 tons, in which ca~e one trip per day would do, but without into figures carefully I cannot answer the question positively. If the haulage is cheaper on the broad-gauge line, it shows that there is an important traffic.

1298. The vulnerable point in your argument is, that the little engine requires a tlriver and a stoker, who will cost just as much as the driver and stoker on the broad-gauge line, whereas, on the broad-gauge there is a bigger engine employed which can take three times the quantity at one trip?­Yes, if there is three times the quantity to take, that is all right, but if not, it is all wrong. At Luc-sur-Mer they have ten trains per day each way, and in four months they transport 300,000 passengers ;•there are 2-ft. tramways or railways in France that transport about 150,000 passengers per month, and at the Paris Exhibition this 2-ft. railway transported 65,000 passengers in one day, and in six months it transported 6,.'500,000 passengers without one single accident.

1299. That might be so, and yet the line might utterly fail with a traffic such as we have in the mal lee ?-Why should it?

1300. That is what I think you have not shown us. We have to do this traffic at a cost of from id. to .Jtd. per ton per mile; can you do that on the narrow gauge with an engineer and a stoker, at the union rate of wage ?-I can go into the calculation if you like, but I have not the necessary information here to answer you. I would like to see how those railways are constructed, and what they will cost to main­tain in a few vears.

1301. 'Your system can Le constructed, taking the estimate of our own engineers, for one-third the cost of the Lroad gauge, but when constructed, the uifficulty arises aLont this small engine doing the work. If the smnll engine can do it for less than it is done now, viz., ~d. per ton per mile, then yonr argument is successful ?~Ilave those rail wnys Leen constructetl for any length of time ?

1302. Two or three years, ami the engineers have given a certificate that they pay ?-It mny be that those rnilways are eon~trncted on the right principle, but it simply shows that the amount of traffic is very great, and perhaps it would not pay to build them on a 2-ft. gauge. If, however, the traffic is only 300 tous per day, it seems small.

130:3. By the Hon. J. Buc!tanan.-What is the cost of the different types of those engines?­Nnrnher 1 type of engine weighs 5 tons and r.osts .£550-the 9-ton engine costs £1,000, and the duplex

tlw same; the last improved eugiue, weighing 12 tons 12 cwt., with a cog-wheeled arrangement for mountainous country, costs about £1,500.

1304. By Jlf't. Burton.-Taking a line from one of the present existing lines to Mildura, which would bo looked npou, if constructed, as a main line, would you be in favonr of such a line being constructed on the narrow gauge ?-Not if it is a main line; it is only branch lines that should be constructed on the 2-ft. gauge.

1305. By t!.e Chainnan.-IIave yon hatl any experience of a break of g·auge anywhere ?-The rail­way from Dives to Luc-snr-Mer is connected at each end with n broad-gauge railway.

1306. How do they manage to tranship the goods ?-They have two platforms, the trucks are placed on the same level, and they come \"ery close to each other, so that the goods can be easily tran­shipped.

1307. It has to be done by manual labour?-That depends upon the class of goodti--sometimes they use cranes.

1308. Can a broad-gauge truck be contrived so as to run upon a narrow gauge ?-That iloes not pay, you would lose all the benefit of the narrow gauge.

1309. In many parts of the colony the only produce the people haul is perishable fruit, and if that is shiftetl two or three times it will be destroyed ?-Of course, in ,;uch a case, extra precautions woulu have to be taken, aml perhaps new methods may be designed to meet the case. In any case, only one extra handling is required.

1310. You do not know of any better methotl than to remove the gootls from one truck to another? -That i~ how it is done in France. I believe at every terminal station they have a crane for transferring heavy goolls, but as a rnle it is done by hand, and no difficulty is experienced either in France or on the Continent. They have narrow gauge,; nll over the world; it is only iu Australia that there is any opposition to them, and I cannot make out why there should he that opposition. A B.ailway Congress has been instituted by Leopo1d, the Kiug of the which meets every year, aud engineers come to that Congress from all parts of the world, and connected with the constrnction of railways is con-sidered, but so far the Australian colonic~ hn.ve never tnken the trouble to send any one to repre~ent them at that Congress and listen to what i::~ said. That is why they arc backward. Mr. Eddy has gone to

New South "\Vales recently for the first time. I have never seen an engineer yet who had spent some time in Europe and studied the question who has not been converted to the necessity of building railways according to the traffic.

1311. By the Hon. D. ~klelville.-Can you tell me what are the wages usually paid iu France to the driver and stoker, aml what are their hours of work ?-1 have not that information, but I believe the engineer gets 4s. per day.

1312. By .1!1-r. Craven.-Do you admit that Wellington is an authority on railway construction?­I do not !mow of him.

GZ

a. Tan de Velde, 14th Marcll, 189~. 84

UH3. On page 7 51 of his work he says-" The irresistible logic of events has practically settled the question, and the belief in the narrow gauge as an expedient and defensible system of construction, which was from the beginning founded chiefly on illusion and delusion, is rapidly passing away, and all but gone. We may therefore merely summarize briefly the leading points of the question. But the question then arises : vVhat is saved thereby ? If it be to increase the hauling capacity of engines, a very slight addi­tional curve compensation will neutralize the extra resistance of the wider gauge, and we have already seen (par. 290) that any radius which is likely to be desired is readily practicable for properly designed standard­gauge engines. If it be to save the extra wear and tear and loss of power, a small reduction in an item, the whole of which is so small, is not worth any considerable sacrifice, nor can it be taken for granted (nor is it probable) that there is any such reduction. As respects rolling-stock, there cannot be a question that there is absolutely no practical advantage in the narrower gauge. Any reputable locomotive builder will contract to build engines of the same weight and power, for either gauge, which will traverse the same curves, for the same price. The standard-gauge engine, in faet, will or can have enough shorter wheel base, because of its greater width, to make it take curves a little better" ?-I do not agree with that, be­cause when you adopt the broad gauge you cannot go down in the weight of the locomotive below a certain limit, and that limit may be far too big for the traffic, while in the other case you can go down to 3 tons or 5 tons, and build an engine to exactly suit the traffic.

1314. He goes on to say-" A very important point which narrow-gauge advocates, and opponents alike, have almost wholly lost sight of. There remains, therefore, as the net gain from the narrow gauge, only the slight saving in grading and ties, which may amount to from l to 4 per cent. of the total cost of the line. On the other hand. thet·e are several very serions losses. The one ·which is alone of decisive importance is the great loss from not being nble to excllange traffic in bulk, bnt having to tranship a.ll freight and passengers. The loss from this is far more than its direct cost. The resulting inconvenience, delay, and damage to freight drives away much traffic. The cost of maintaining track to a given standard of excellence is likewise greater, the cost for track labour being in about inYerse proportion to the length of the ties. The less bearing area of the ties on the ballast increases this disadvantage materially" ?-I do not understand that. I would require to study those statements before I could give an opinion.

1315. And further-" The maintenance of rolling-stock is decidedly more costly in proportion to work done, and the train resistance higher, because of the smaller wheels "?-Yes, but they start from a bad point, they start from a big traffic, and that is where they make the mistake; that is quite right when the traffic becomes more moderate.

Sm,

The witness witltdrew.

Adjourned.

[Letter from C. Van de Velde, C.E., relative to zoorking expenses on narrow-gauge lines.]

Sydney, 3rd June, 1895. For the guidance of your Committee, I would vrish to present some new remarks on the gauge question. While the compll.l'ative merits of the 2-ft. and the 5ft. 3in. railways are being inquirell into, I think that the relative

cost of construction is only of secondary importance, because the amount of working expenses, added to the cost of main· tenance, considerably exceed the amount of interest on the capital sunk in the construction of a railway. There may be examples among the non-remunerative lines of Victoria where thttt is not so, anrl where the amount of interest may be greater than the working expenses and cost of maintenance. This would proYe there is no traffic, and that such line ought never to have been constructed at all.

I think, therefore, that the relative cost of working and maintaining 2-ft. aml 5ft. 3in. railways ought to be investi­gated at least as carefully as their relative cost of construction, which, so far, has attracted most attention. I must recognise, however, that questions to elucidate these points have been put to me by several members of your Committee, while I was under examination before, but I have only been able to give general information in view of proving why, when the traffic is moderate, working and maintenance expenses are smaller in the cases of narrow-gauge lines. In fact I could not, and no expert could, give any other information, because what wanted to pl''Jve the point is not the cost of working and maintenance of an existing 2-ft. railway, but their compamtin: amount with what it would be were the same traffic carried over a 5ft. 3iu. railway.

Comparing the cost of maintenance and working an<1 interest ller ton of traffic of an existing 2-ft. line with that of an existing 5ft. 3in. line somewhere else where the traffic is larger wonl<l lead to absurd conclusions. anc1 that is precisely what the engineers who believe exclusively in broad-gauge railways do; and this explains how they come to the conclusion that-" The working expenses for equal volumes of traffic arc greater on narrow-gauge tlmn on broad-gauge Iincs." That is true where the traffic is large, and it is the reverse where traffic is moderate.

I think, Sir, that your Committee could settle this question if the following suggcstiom were carried out:-1. Select among the existing Victorian railways a line which has left a clear loss sim:e its constrnetiou. 2. Get a survey made (I mean a proper surrey) of a 2-ft. line connecting the two same points, and establish t,he

cost of construction. 3. The traffic being known (this is the great point), weight of rnil, weight of locomotives, cost and deRcription

of trucks required, cost of haulage, cost of maintenance, interest on capit~l-in fact, every information will become available to settle the matter in what I believe to be the most practical manner.

If such inquiry is made by competent engineers, the result will prove thnt the selected line would have left either an annual profit or a considerably reduced loss to the colony had it been built ou the 2-ft. gauge.

I would be pleased if this letter could be printell as evidence.

I have the honour to be, Sir,

Your obedient servant,

C. VAN DE VELDE.

85

WEDNESDAY, 7TH AUGUST, 1895.

The Hon. J. Buchanan, .1\LL.C., The Hon. D. Melville, :M:.L.C., The Hon. E. Morey, 1\:LL.C.

Members pre1ent :

Mr. Burton, Mr. Craven, Mr. Harris, Mr. Trenwith, Mr. J. S. White.

In the absence of Mr. Camoron, Chairman, the Hon. D. Melville took the Chair.

Robert A. Robertson, sworn and examined.

! .n6. By the Vice-Chai·rman.-What are you ?-A saw-miller. ; :~17, How long have you been saw-milling in Victoria ?-Fifteen years. I was also in the timber

line in ~·•· tland and America. 1;; \ -. Have you ha(l any experience in tramways or narrow-gauge railways in eonnexion with saw-

milling ?--:-,res, I liuilt one. 13! c'. Ha-ve you had any experience of them in any other country?-Yes, in America. 13 :J. What gauge were they ?-Three feet six. 13:H. What was the smallest ?-That was the smallest I have seen working. 1322. What is the nature of the line that you have built ?-It is heavy grades, and for timber

traffic. It is on the Plenty Ranges. It starts from W andong, going due east. 1323. How long is it since you started the construction of that line ?-In 1884. 1324. How manv miles were constructed in 188·1 and 1885 ?-Eleven miles. 1325. How long' did you work the eleven miles ?-It is still working. I have added to it since. 1326. What would be the averagn cost during tbe year,l884-5 .for the line you constructed, without

rolling-stock ?-~At that time it was all wood. The sleepers were 6ft. 6in. by Bin. by 4in.; the rails were 4in. by 5in., wedged in, worked by horses, and corduroyed. 'rhe grades were heavy, and I had a cable rope half-way. lnelmling the cost of that and the heavy bridges it was under £500 per mile.

1327. What has been the cost of maintenance of such a road as that, including bridges, over the whole period of ten years ?-Nearly £1,000 a year for the eleven miles, with cable rope work.

1328. Is that for maintenance alone?-Yes, and cable work. 1329. By 1lfr. Trenwitlt.-The cost of maintenance is much higher for wooden rails than for iron

rails ?-Yes. I commenced to extend it in 1886, and am still doing so. It is not finished extending yet. I have extended it for nine miles up to the present.

1330 . .By the Vice-Chairman.-Has that extension been about the same cost as it was for the original construction in 1884--5 ?-No, it was cheaper; I did the wooden rails for £350 a mile, including bridges.

1331. What is the nature of the trafllc that you have ?-I have averaged 40 tons a day; my trucks carry 5 tons each.

1332. Have yon made any estimate of the cost per mile of your traffic downwards to the railways ? -Threepence per ton per mile, including maintenance. It is cheaper on the other nine miles, where we are carting logs; there we do it for twopence.

1333. I believe yon have leased some iron rails from the Government ?-About two years ago I leased those iron rails, ~nd we have been substituting them for the wooden ones.

1334. What is the steepest grade in your line? -One in 10. 1:135. Is that where the wire rope is ?-No ; the cable rope is 1 in 4, the first portion. 1;)36. Have you formed an estimate of the cost of a 2-ft. gauge for the whole twenty miles on a

grade of 1 in 40?-No. l 337. What would it cost to convert that 3ft. 6in. line into a real railway with a l in 40 grade ?­

If I had been allowed elm·en year~ ago to go the way we are uow going along the range, which would have been fifteen miles instead of eleven miles, I could have got grades of 1 in 25, and 1 could have constructed the whole line, with wooden bridges and all, for £350 a mile, the Department letting me have 50-lb. iron rails at £3 10~. a ton-that is £320 a mile. In constructing a tramway with iron rails I would have cheapened the construction and brought it down, without the iron rails, to under £300 a mile.

13:38. By 1lfr. Trenwith.-Wonld corduroy do for a railway ?-No; I would add £100 a mile for hallasting, and make it £700 a mile.

1339. \Vhat is the difference in the cost of maintenance between a railway with iron rails and a wooden tramway ?-I went to Mr. Speight before he left the Railways, and showed a loss in six years as between wooden rails and iron rails of £6,000.

1340. The cost of maintenance for that eleven miles was about £1,000 a year-how do you make out that you would save £1,000 a year when your present expense is only £1,000 a year ?-I find there would he need for less horses, and there would he heavier loads. I could maintain that lino with iron rails for £150 a year for the whole distance, with no cable.

1341. You think that a 3ft. 6in. railway could be constructed substantially in that district to carry engines and a reasonable traffic for £700 a mile?-Yes; that is the minimum.

1:142. And it could be maintained for £20 a mile ?-If you want to carry passenger traffic you must a little more for maintenance; you would then have to allow about £25 a mile for maintenance.

l3f3. You think that a railway could he made in that district for £700 a. mile, and maintained for £25 a mile ?-In my opinion it could.

1344 . .By M-r. J. 8. White.-Do you work the whole of the service with horses ?-We have had a locomotive since the bt•ginning of the year, which works portion of it. The heaviest grade is 1 in 14, and the locomotive pushes up that grade of 1 in 14 with a 70-ft. curve from 10 to 15 tons on iron rails.

134.). By .h:fr. Trcnwith.-What is the power of the engine ?-About lO horse-power nominal. 1346. By 11-'f.r. J. S. White.-What is the weight of the engine?-With the water tank filled it weighs

7f tons.

R. A. Robertson, 7th August, 1895. 86

1347. What could she take up Gn a grade of 1 in 25 ?-I think she would take up 30 tons, but I do not know that as a matter of fact.

1348. By 11-fr. Harris.-'\Vhat is the most weight you have drawn ?-I have never put on an extra load, but I think she could pull 50 tons on easy grades.

1349. By 1Jfr. Trenwitli.-If the engine were in front could you carry any larger load?-Yes, you wouh~ have to use sand on wet mornings. It is a Baldwin engine, not on the bogie principle; the wheel base Is 5 feet.

1350. By .!Yb-. Harris.-That engine is not the only motive power on the line ?-No, it is chiefly horses. We have horses, an engine, and a cable rope half-a-mile long.

1351. By 1rlr. Trenwith.-Assuming you had the same load to carry both ways, do you think there would be any economy in coming down by gravitation; that is, would the saving in coming down more than compensate the extra cost of going up. If the line were level would it take more coal to haul a load both ways than to haul it up and let it run down by gravitation ?-I cannot say; the engineers in America say they save money by having the loading to go part of the way by gravitation.

1352. By Mr. J. S. Wkite.-If you had equal tonnage going e!l.Ch way what would it cost you?­I could do three times my present loading and bring the cost down to 1~d. per ton per mile if I had the quantity to take. If I had loading both ways I could bring the cost down to 1 d. per ton "per mile; the traffic is limited now, but if you treble the output from the mill, it would only cost a small amount more to carry the increa.sed traffic, and if heavy traffic going down to the railway it would bring the cost down to 1d. per ton per m1le.

1353. By the Vice-Chairnwm.-More tmffic both ways would lessen the cost?-Yes, my men and horses could do the work.

1354. By the Hon. E. 1Jforey.-What do yon consider the most suitable gauge for taking traffic up an incline ?-When I commenced in that eountry, I took my mill into the mountains and I had to find my way out. I studied the necessities of the timber traf!ic, and the 2ft. 6in. gauge appeared to be too narrow for log work. I satisfied myself that we could work the log and timber traffic with a 3ft. 6in. gauge more economically than with any other. I was satisfied that the cost of construction would be only about half of a broad gauge round the hills.

1355. What do you consider the most suitable grade ?-Had I heen allowed in the first instance to follow the Dividing Range, I would have got a grade of 1 in 2.3, and I would have saved money every year.

1356. Is not that too steep ?-No, I could have had a locomotive running for the whole of the eleven years.

1357. Would it not be very severe on the rails coming down those steep grades with a heavy load 1 -We have had a good deal of traffic on these rails the last four years, and I do not see any great wear and tear with our traffic ; it altogether depends on the traffic weight of the trucks.

1358. What are the sharpest curves ?-I have one of 40 feet radius, and I brought a log of 65 feet round there on bogie trucks. The tear and wear on the truck wheels ia considerable on such a radius as that.

1359. Do you not think a gradient of 1 in 40 would be quite sufficient on a railway intended for public traffic ?-I should say a grade of 1 in 30 would answer for carrying passengers with safety. I am quite satisfied that you cannot beat a 3ft. 3in. or a 3ft. 6in. gauge, with a grade of 1 in 30, in hilly country and timber traffic.

1360. What load could you carry up on that line ?-If I had a 15-ton bogie engine I think I could carry up about 70 tons. I have seen it done in America.

1361. Do you not think a grade of 1 in 40 would be more practicable and safer ?-No doubt it would be much easier worked, but the question is, can you get such a gradient round the hills. You have to rise to a certain altitude in a certain length.

1362. That means going round the spurs and making bridges across the gullies ?-You are limited to a certain length, and you have to rise to a certain height. I would take the easiest grade that the country would allow.

1363. You think that it would be perfectly safe for an engine bringing down a big load of passengers or cattle to come down a grade of 1 in 30 ?-Yes; I can put the brake on now on a gradient of 1 in 1 O, and stop the train with 40 tons weight on it.

1364. By Mr. Ha1"ris.-Have you met with any accidents on the line ?-A few, but they have been chiefly through carelessness.

1365. Have there been any that you could trace to any other cause ?-No. 1366. If you had to construct a tram now would you still make it 3ft. 6in. ?-Yes, for timber. 1367. What is the longest bridge ?-1,200 feet. 1368. What height is it ?-65 feet from the foundation. The decking of the bridge is 9 feet wide,

and there are side rails. 1369. What does the flooring consist of?-4 inch sawn timber. 1370. What is the girth of the piles ?-They would average 13 inches at the small end. The

bearers are 14 inches by 14 inches. 1371. What is the cost of a bridge of that character ?~That bridge cost £450. 13i2. By 1Jfr. J. S. White.-Was the timber on the ground ?-The timber was close by. 1373. By the Vice-Chairmam.-What height do you rise between Wandong and your highest point?­

In the distance of four and a half miles that you went over with the locomotive we rise .?00 feet. The bulk of the rise takes place in two miles; you go up about 250 feet in two miles, then you fall on the cable rope 250 feet, and by gravitation you go another 250 feet to the bed of the Sunday Creek; then you have an easy grade of 1 in 30, rising steadily for about three miles. Then you rise in three miles 1,000 feet.

1374. Is that the maximum distance you are above the sea ?-·The maximum distance is 2,500 feet above the sea level, and about 1,500 feet above the level of the station, and we do that in eleven miles.

1375. By Mr. Harris.-How long have these bridges been built ?-Some of the oldest have been built eleven years.

1376. Is all the traffic that you have been doing on the tramway taken over these bridges ?-Yes; I have had to strengthen them lately for the locomotive traffic.

87 R. A. Robertson, 7th August, 1895.

1377. Do they stand well ?-Yes. 1378. Are they subject to floods?--Yes. I have had only one little trouble with a flood ; it

nearly carried away the sill of one of the openings. 1379. Are all the bridges bubject to floods ?-Several of them are. They have all stood very

well. 1380. You have not had any trouble with them ?-No. 1381. If you had to reconstruct them would you build them in the same way?-Yes, I would

adhere to the same principle, only I would take more care in the stiffening for locomotive traffic. 1382. Would that add much to the cost ?-No. 1383. By 1lfr .• I. S. Wltite.--If yon are 1,000 feet above, you can run up 1 in 40 in eleven miles?­

yes ; but we rise and fall. 1384. If you can go round the hills you can get l in 40 ?-I have just got finished a contour

survey by the engineer who did the .Frankston awl Kerang lines, and he gets a grade of 1 in 25 in the six miles. Certain portions of the eleven miles are almost level, and then you come abruptly to a rise.

l:i85. By Mr. Craven.-Do you consider as to sleepers it is quite sufficient to put down rough logs ?-Logs of messmate split in two have heeu down 32 years at the Woodend station. That is quite sufficient without having them sawn.

1386. Suppose the line were trimmed up to snit passenger traffic, what would be a safe rate of speed to go at ?-With a locomotive, fifteen miles an hour.

1387. VVhat do you find the difference hetween horses aml an engine for haulage ?-The difference is fully a halfpenny per mile. If a tramway was brought to a working grade round the mountains, and had plenty of traffic to bring it down to 1 ~d. per ton with horses, you can take the halfpenny off and do it for a penny per ton per mile with a locomotive.

1388. Where you have very sharp pinches are not horses cheaper ?-If you are obliged to go with horses where the locomotive cannot go, and no other way of working, you could not do it with a locomotive at all.

1389. 'What does it cost you to haul on that portion going back from the mill where it is up and down ?-Not twopence a tou per mile.

1390. Your present traffic is 40 tons a day, or, roughly speaking, about 12,500 tons per annum­would that line pay if the traffic on it only amounted to 3,000 tons ?-No, it would not.

1391. On the other hand, if you had a greater qnantity of traffic it would pay better ?-Much better.

1392. Assuming that you are taking stuff away from the mill and bringing coa1 back, what would the haulage cost with a ruling gradient of 1 in 30 ?-I would do it for twopence per ton per mile, with present trafli c ..

1398. Wbat is the radius of your sharpest curve ?-About 70 feot; the steepest gradient for the locomotive is 1 in 14. The diameter of the cyli!l(ler is 7~ inches with a 12-inch stroke; there are two axles coupled and a rigid wheel base of 5 feet; the wheels are 3 feet and 4 feet respectively; with tank full of water the engine weighs 7i tons; the gross load that she can haul up an incline of 1 in 14 is four waggons and 10 tons of timber-that would come to about 15 tons altogether.

1394. Do you know anything of the Darjeeling line ?-I have reacl of it. 1395. On that line the engine weighs 13! tons, the steepest grade is 1 in 28, and the gross load

35 tons-do you agree with that ?-I do not say I could do it working every day, but I say I could pull 70 tons on occasion-the working load I would reduce to 50 tons.

1396. By 1lir. Ilan·is.-What experience have you had in America ?-Chiefly in tramways in con­nexion with the timber traffic.

1397. Were they tramways of a similar character to the one you have constructed here ?-They were lighter-the sleepers and the rails were lighter ; they were altogether of a cheaper construction.

1398. \Vas the country as difficult as the country at \Vandong ?-No; there was no such country there. I have seen several lines in America, 3ft. 6in. passenger lines, worketl by private companies.

139.9. What do you pay the Government for the rails ?-Four per cent. on the price of £3 10s. a ton ; they are very good rails, though they have done a little traffic.

1400. Is it not a fact that horses are cheaper than steam for a short distance ?-I should sav not for a distance of ten miles. •

1401. For what distance would they be cheaper ?-It would depend upon the tratfic; you cannot beat steam if there is plenty of tmffic. Of course the locomotive cannot work a gradient of 1 in 10-there the horses come into play. \Ve could not even have iron rails on that grade; that grade extends for a distance of a mile and a half.

1402. By JJf'l'. J. 8. White.-If this ten miles had a ruling grade of 1 in 30, what would be the difference in cost between horses and steam ?-I would tlo it for half the cost with steam with plenty of traffic.

1403. Have you ever known any local body to construct a tramway of this kind and maintain it as a feeder to any main line as a substitute for a main road ?-No, I have had no experience with local bodies.

1404. In rough country where there is no road at all, and it is necessary they should have some sort of communication, would a tramway of this kind be as cheap as a road ?-Cheaper; it would serve the people better and would be more easily maintninetl.

1405. By the Vice-Chairman.-What do yon wish to say about the Beech Forest ?-After uoing for 22 or 23 miles yon tap the country on the northern side of the range, where there is timber enough to keep six mills with a capacity of 5,000 feet a day going for ten years. On the southern slope you have beech and blackwood and other timber sufficient to keep three or four more mills going. I estimate that I could put in a road with iron rails at a cost of £900 per mile, including the cost of the rails, with a rulincr grade of 1 in 30, if the Railway Department gave the rails at 4 per cent. on £3 I Os. a ton. The cash "'needed to build that railway would be £20,000 at the outside, and adding £10,000 for the rails would brino- the interest to be paid yearly to £1,200. The working expenses I put down as .£I ,300 a year, to which I a!ld £500 for contingencies. I estimate that my revenue would consi><t of 100 tons of timber a day, at 3s. a ton-£15, produce £3, passengers (six per day, ~t 3s. 4d. each) £1, and a subsidy from the main line of

R. 4. Robert&on, 7th 4ugust, 1896. 88

£~; ma.king a total of £20 a day. Taking an average of 250 days in the year, that will give a revenue ot £5,000, and if you deduct £1,000 for contingencies, it leaves £4,000 as against £3,000 for interest and working expenses.

1406. Do you consider the rung timber of any value even if it has been rung for two or three years it is still valuable.

1407. You think that if the railway were built, those enormous dead trees would be worth cutting? -From what I can see, the majority of them would.

1408. Forsawntimber?-Yes. 1409. Can you us a rough idea of the value of those trees for milling purposes ?-On an

average there would be, suppose, fifteen trees to an acre, which would be worth to a saw-miller 1 Os. a tree at the mill, or £7 10s. an acre.

1410. You could afford to give £4 or £5 an acre for the timber if you had a railway ?-I have paid more an acre in different parts of the colony.

1411. Do you desire to say anything about the timber in your own district ?-There is abundance of timber in the watershed of the Yan Yean.

1412. Would it injure the watershed if you were allowed to cut the timber there ?-No, I think it would improve the water.

1413. By JJfr. J. 8. White.-How manv acres are there of this Beech Forest ?-From what I could see I reckoned there was a milling area of 20,000 acres at least.

1414. A large amount of that is Crown lantl ?-Yes; I should say that more than half the timber was on Crown lands.

1415. If you were allowed to put a saw-mill there would you see your way to construtt the line in exchange for the right to cut the timber, undertaking to charge the people a reasonable rate for the carriage of their stuff and to hand the line over at the end, say, of ten years ?-I will do so to-morrow if you give me the money to build to the foot-hills the main line, and the iron rails for the saw-mill line; and the timber traffic for ten years would give an ample revenue.

1416. By the Hon. E. Morey.-What are the selectors (lisposed to take for their timber ?-Some of them volunteered to give it to me at my own price-whatever I thought it was worth.

1417. Would you thorn £7 an acre for it ?-No; I say it is valued at that at the mill. 1418. What do you think you would give ?-£2 an acre on an average at the present prices of

timber. 1419. Would there not be a probability of some of those men selling the timber and leaving the

land ?-No, I think not. The majority of the land there is good. 1420. What would they be likely to take for their timber ?-As far as I could gather, they would be

very glad to get £2 an acre for it-some of them would take lOs. an acre. 1421. Would you have to clear the tops away after you had cut the tree down ?-No, we simply

take the saw-mill log out and leave the debris behind. They need that to burn up-it does the land good.

1422-3. By Mr. J. S. White.-Would £2 an acre be sufficient to clean up, burn off, and fit the l~nd for cultivation ?-Yes, ample, in that country.

'l'ke witness withdrew.

Adjourned.

THURSDAY, 8TH AUGUST, 1895.

Members present:

MR. C.urERON, in the Chair ; The Hon. D. Melville, M.L.C. Mr. Burton,

Mr. Craven, Mr. Harris, Mr. Trenwith, Mr. J. S. White.

Thomas Hale W ooclroffe, recalled and further examined. 1424-5. By the Chairman.-What are you ?-I am one of the Acting Commissioners of Railways, and

also head of the Locomotive bra.nch of the Railways Department. 1426. We received a letter some time ago, dated the 29th of April, signed by Mr. Kent; were you

the author of that information ?-I might say that particular memorandum was prepared by the Engineer for Existing Lines and furnished to the Commissioners in response to certain questions asked by the Standing Committee, and was then forwarded to you by the secretary.

1427. Were you aware that he sent it?-Yes. 1428. Then you agree with the contention of this letter ?-Generally I agree with it; that is to say,

I know that the various experiments were carried out, and those arc the results. 1429. Is it your opinion that if a 5-chain curve were used on the present broad lines it would be

necessary to use a 75-lb. rail ?-It would not be absolutely necessary. 1430. Would it be necessary on a curve of that kind to have an inner check rail on the curves?­

It would not be necessary unless you ran at very high speeds ; with moderate speeds it would not be necessary. The introduction of a 75-lb. rail on curves and.a check rail on curves would be .better from .a maintenance point of viow-that on sharp curves there 1s always more o~· less wear, a.nd 1f a c~wck _ra~l is placed inside the inner rail it les;,ens that wear and tear, an<l from a mamtenance powt of VI.ew 1t IS

better to have a check rail; but with the experiments at North Melbourne there was no check ra1l. 1431. Do you agree with the proposal to use 40-lb. rails on the 5-chain curves?-We have no stock

fb!!-P would be suitable for the 40-lb. rails on those curves.

89 T. H. Woodrolfe, Bth August, 1895.

. 1432. If you had the stock, would it be advisable to use a 40-lb. rail on a 5-chaiu curve ?-I think It would be better to put down 60-lb. rails. It all depends on the sort of engines you are going to run. If they are very light engines, light rails would be suitable, but if they are engines of moderate power, and there is good traffic on the line, a 60-lb. rail would be far better, especially on those curves.

1433. You would not like to take the responsibility of working the bread gauge with a 5-chain curve, a 40-lb. rail, and a l in 20 grade ?-It would depend, of course, ou the traffic you have to carry. As a matter of opinion, I should prefer on such a line 60-lb. rails.

1434. Could you work it with a 1 in 20 geade?-Yes. 1435. And a 5-chain curve ?-Certainly.

. 1436. It would be perfectly safe to ru11 passengers ?-Perfectly safe at a moderate speed, say, twenty mtles an hour. .As a matter of fact, these results were from trials on a curve laid down at North Mel­bourne, which now exists, a circle of 5 chains radius, and types of all those vehicles have been run round at this speed-it is a matter of actual test.

1437. What grade ?-It is a leYel there, but the grade does not make any difference. 1438. Does not the propelling power of a siring of engines make 11, difference coming down a curve

of that kind ?-It would make a difference coming down, but with such curves as these you would have every vehicle in the train braked with a continuous brake.

1439. Have you known such being used on any other line except the experiment you tried at North Melbourne ?-There are plenty of 5-ehain curves and :3-chain curves on .American lincs-I haYe not been oyer them, I have read about them. . 1440. What are the grades there ?-There are plenty of grades l in 25, and grades as high as 1 m 20,

1441. Would they haul up, withal iu20 grade, say, lOO tons, without a rack rail ?-Yes, you could construct an engine that would run 011 that.

1442. But with an ordinary engine-our engines ?-Our engines vary. 1443. By M1'. Craven.-Say the Beeehworth engines ?-The engines that would ha most suitable

for such a line would be the .American engines which we have, the S's and the vV's. 1444. Would those be suitable for a place like Walhalla ?-They would be suitable for any line

with 5-chaiu curves ; they would go round those quite easily . . 1445. What is the gauge iu .Americn where they rnn on 3-clmin radius ?-The instances I have in

my p:und, I think, are 3ft. 6ln. gauge, but, of course, as far as cunres are concerned, the Manhattan Elevated Hallway, iu New York, of 4ft. Hin. gauge, has curves round the corners of the streets of 90 feet; and there is a well known instance, quoted hy a high engineering authority, where a military train was run for a considerable time round a curve of 50 feet- I believe that was a 4ft. Stin. gauge line.

1446. Have we any quantity of 40-lb. rails in Victoria ·?-We have none. 1447. If the 40-lb. rail were adopted for any line you would have to send for them?-Yes; we have

SO-lb. and 60-lb. rails in stock. 1448. Have you any idea what would be the cost of the 40-lb. rail per t()n per mile ?-I have not

the figures in my head ; they would run about 60 tons to the mile, and you would have to pay somewhat more for them than you would for the heavier rails. The last rails we got cost about £4 lOs., and for the 40-lb. rails I should think yon would have to pay £5 IOs. or £6 a ton, landed here. There is the same labour in rolling a light rail as a heavy one.

14-19. Suppose you constructed a line with a 40-lb. rail, what would be the total weight of engine necessary to work the line ?-It all comes back to what business you have to do. The weight of the engine would have to be determined by the load per axle. If you wanted an engine for light tratl1c, an engine with only two pairs of wheels coupled might suit. If you wanted a more powerful type of engine you would couple three or four pair8 of wheels, or, as in .America, five pairs of wheels. 'When you get an extremely powerful engine, which is not too heavy for the road, the concentrated loads on the wheels are in proportion to the weight of rail, thus, a 40-lb. steel rail would approximately carry 8 tons per axle, so if you have an engine with three coupled axles it would be a 24-ton engine, or with five, a 40-ton engine.

1450. You would not recommend those for a line like the one to W alballa ?-If it were laid with a 40-lb. rail, and there were any traffic, it would have to be such an engine, one of those types, but it would depend on the amount of loading.

1451. Or anywhere else with a grade of l in 20 or l in 25 ?-Yes. 1452. The usual engine for steep grades is what is called the eight-wheeled coupled engine, the

consolidation engine ?-Dccapod engines are those with five pairs of wheels coupled. 1 453. What is the steepest grade we have in the country'?- One in 30. I believe that is the steepest

on any of the running lines. We have grades running to the coal stages of l in 25. 1454. Where is the one of l in 30 ?-That is on the Beechworth and Yackaudandah line. 1455. They eau manage that?-Yes, there is no trouble about that. 1456. Do those very henvy engines destroy the roadway more than the light ones ?-Not if the load

per wheel is in proportion to the rail. 1457. What should bo the loatl on a 40-lb. rail per wheel ?-.About 4 tons. 1458. Have you any eugines of tlmt description ?-We have a few of the .American class that, I

think, do uot exceed that. 1459. B !J the lion. D. ilielville.-Snppose the Committee were to decide to recommend in certain

districts here branch lines 20, 30, am! 40 miles lollg, on the many models we have got. Here is one for example:-

H.-SWEDEN AND NoRWAY. Kosta-J~essebo Railway:-

J,ength of line, 12 miles ; gauge, 24 inches. Greatest ascent, l in ;~5. Smallest radius of curves, 30 yards. Hails, 18 lbs, per yard. Cross sleepers of pine wood, 6 inches x 4 inches and 3ft. 4in., laid at a distance of 20 inches from each other. Locomotives :-(Compound system) with 4 cylinders, 8 ddving wheels; weight in working order, 14 tons;

working pressure, I~ atmospheres. Performance :-Tltey haul on a grade of l in 35 trains of 40 tons, at a speed of about 12 miles per hour.

Goods vans :-Weight empty, 1·5 to 2 tons; capacity, 5 tons.

T. li. Woodroffe, 8th August, 1895. 90

That is the smallest type working, and they have them all the way upwards till they get to our gauge. Can you construct and complete a twenty-mile line here of similar character to that, and, with your machinery, construct the engines to suit such a railway?-Yes.

14?0. By the Chairman.- Wlwt is the difficulty of having a railway here at a cost of from £500 to £600 a mile on a 2-ft. gauge on a level country like the mallee plains, say with 50-lb. rails ?-The diffi­culty would be the cost. I do not think it conld be done.

1461. You cannot construct with the 50-lb. rails you have in stock railways on plains in the country at from £500 to £600 a mile ?-The rails alone would cost £500 at the very least.

1462. Coming to the new rails, from 18 lbs. weight and upwards; we have 30 lbs. to the yard, 36 and 48 lbs. to the yard in England. In Austria, they have 170 miles, 30-inch gauge, rails 28 lbs. to the yard, and 32 lbs.-can that be done ; can you construct the railways with any of those lesser weights at £500 to £600 per mile ?-I do not think you could construct a railway that would be of any service at any­thing like that money.

1463. Those railways are doing an immense amount of work in passengers and traffic; some are 170 miles and some 40 miles long, and those are the weights given to us. ·what do you mean by "any service"? -To carry any traffic. ·

1464. They have a heavy traffic. We have obtained information through Mr. Gillies, which is in print here, from Austria, Hungary, England, Switzerland, Sweden, anti Norway, all of which countries are using those railways for both passengers and goods, and all at that cheap construction rate. Do you think it is practicable in your hands to do this kind of work, say, on the malice plains ?-Personally, I have not gone into the questiou. I have not taken out the necessary quantities, aml I could not say just at thi~ moment. If you wish me to answer a question like that I will go into it.

1465. Somebody has to come from your Department to put the Committee right in the matter of figures ?-I can only say this, that I presume you have had a number of estimates of those lines prepared by Mr. Rennick, and I am quite certain that they are perfectly honest and reliable estimates.

1466. That is right, but what view do you take of the real practicable working in other parts of the world of those lines ?-Unless one knows the whole of the conditions one cannot say what can be cone. What would be suitable or could be done in one country would not be suitable or could not he done in another.

1467. Do yon see anything in our conditions in the malice why we should not have those ecomomical lines I have indicated. On any of our plains is there any special condition in this country why we cannot obtain this desideratum ?-You can make n railway at any price and of any gauge, hut whether it would be desirable to do so is another thing. You can make a 2-ft. railway in the malice at a cheap rate, much cheaper than our broad gauge, but the question is, would it be desirable to do so-would it not in the end prove more expensive. I think it wonltl.

1468. The advice of the Engineer-in-Chief is that we camwt do it, that the cost of the 2-ft. gauge so nearly approaches the other, and with the atlvantage of having a surplus of rolling-stock we are confined almost to the necessity of continuing the present gauge. Is it a fact that we can construct 2-ft. gauges almost at any price on the same basis of quality ?-The position is this-yon may make a 2-ft. gauge rail­way, and you may make a 5ft. 3in. gauge railway through the same country, but you would find in the end, although the 2-ft. gauge costs less to construct, that it was more costly to operate and keep up; that is to say, that its capacity would be reduced, compared with the broad gauge, and the operating charges would be very much larger-you must take that into account.

1469. Is it a fact that we are so near as to cost of construction that the 2-ft. gauge leaves us a narrow margin as indicated ?-Unless I saw the estimates I could not say.

1470. By J.'tfr. Craven.-As to that experiment that was carried out in the yard, is the line still laid down ?-Yes.

1471. Could we see that ?-Certainly, at any time. It is all there, the 3-chain radius and the 5-chain, and any time the Committee like to inspect it we will have the stock there and the engines, and the Committee can see for themselves.

1472. Have you heard of curves of 100 feet radius in America-in their country railways ?-No, I do not recollect 100 feet.

1473. Have you ever been on the "\Vandoug tramway ?-Yes. 1474. What do yon think of that as a means of carriage of ordinary merchandise or goods, excluding

passengers or rolling-stock ; could you not, in a lot of districts where we have made cockspurs, with something of that description carry all the actual traffic exce}Jt the passenger? -Certainly, except the passenger. It would not be safe for passengers. In many places it could carry all the goods".

1475. Have you had experience of horse tramways ?-Not personally. 1476. Have you any idea whether a horse tram for a short length, eight or ten miles, is cheaper;

is it cheaper to carry small lots of goods by horse than by locomotive ?-I should say it would be a good deal cheaper. Of course the capacity would be limited.

1477. Say for a place that produces 3,000 tons of goods a year; do you think tha.t a horse tramway would be ample for a place like that with a lead, say, of fifteen miles ?-It would, provided that the grades were not too severe. In moderately flat country it would.

1478. Do yon know the type of engh10 that the Cape railways use, the "B" engine, which is used on the 3ft. 6in. g-auge, fonr coupled aud two bogie; carriage wheel base 12 feet, made by Messrs. Neilson, type B, total weight loadeu 45 tons without a tender; the tender is 29 tons ; would that be suitable to 40-lb. rails ?-It would probably be 3~ tons a wheel. That wouhl be a class of engine that could be used on the 40-lb. rails.

'The witness withdrew.

Francis Rennlck, sworn and examinetl.

14 79. By the Cltairman.-Yon are Engineer-in-Chief of the Victorian Railways?-Yes. 1480. You have given an estimate of the two gauges, broad and narrow, for the Walhalla line?-

Yes. 1481, Over the same ground ?-Yes. 1482. One is one mile longer than the other?-Yes.

91 Francis Rennick, 8th August, 1895.

I483. Do you think it would be safe to run a broad-gauge line on 40-lb. rails with a grade of I in 20?-Yes.

I484. It would not require a rack rail ?-Certainly not, unless you had a very heavy traffic. 1485. We would not expect that at Walhalla ?-No. 1486. Which do you recommend for that line, the narrow or the broad gauge ?-The broad gauge. 1487. ];'or all the difference in the cost of construction?-Yes. I488. Y on make the ono £7,40 l, exclusive of the laud and rolling-stock, and the other is £5,239 ?­

Yes. On the 2-ft. gauge there are 2-chain curves in some places, and it makes it longer by a mile. You cheapen the construction by lengthening the line. This estimate is for seven or eight miles only-the most difficult part of the route.

1489. I suppose you estimated the probable traffic on the W alhalla line ?-Yes. 1490. ·would it be necessary to construct a broad-gauge line to carry that traiiic ?-I have given

you my opinion in writing on that very point, and you will see that I say that for the 11resent traffic, and for some years no doubt, a 2-ft. gauge railway would serve \Valhalla, and I discuss the point, and I have nothing to add to that unless you ask special questions which 1 can answer. That gives my views exactly.

1491. Have you any reason to alter that estimate ?-None whatever. I492. You still adhere to that ?-Yes. I493. Yon think you could constrnct a broad-gauge line, 5ft. 3in., to ·walhalla for tho amount you

state ?-Yes, as given in my estimate. 1·194. You could not reduce the estimate of the narrow gauge, the 2-ft., to any less ?-Not to make

an efficient. 2-ft. gauge railway. I495. Do yon think there would be the same difference in level country, such as the malice, between

the cost of the nvo lines, a difference of £2,000 a mile ?-Oh, dear no. I have also supplied the Committee with estimates of the others. In my first evidence before the Committee you will find that in easy country I estimated the difference of cost at about £600 a mile, and the difference for any rail way in any particular country would range from that, it might be to £3,000 or £4,000 per mile, depending, of course, upon the difficulties of eonstruction. In the one case I estimate for a 40-lb. rail on the 2-ft. gauge as against a 60-lb. rail on the standard ga11ge, and, of course, I also told the Committee that there would Le no com­parison as to the efficiency of the two railways-that is, their eapabilities of dealing with traffic with a quick service and a cheap cost of transit.

I496. I see you put down here for clearing the narrow gauge more than for the clearing of the 5ft. 3in. ?-The narrow gauge goes through more timbered country; the routes are different.

1497. Do yon clear the same width of ground for the narrow as for the hroad gauge ?-Yes, it would not make a foot difference ; you must provide for the safety of the trains ; it is not a question of guage, but the s::tfety of the traffic.

I498. Would the rolling-stock project more over the 2-ft. gauge than the 5ft. 3in. ?-Yes, a good deal. The 2-ft. gauge engines would be 7ft. Gin. in width, and your 5ft. 3in. 9ft. 6in. or lO feet at most, and a difference of 2 feet would make no difference at all as to the requirements of the clearing. You have trees lOO feet high, and you have to cut those down to preserve a safe passage for your trains, no matter what the gauge is. I was speaking of clearing trees 150 feet, which have to be cut down within 150 feet of the line.

I499. Are you in the habit of clearing the line from high trees for a train to be safe?-Yes, where the trees are reckoned to be dangerous-not every tree, of course.

1500. I see you put, down for cuttings, earthworks, Is. 3d. a yard; is that the lowest?-That is all rock country, and you could not do it for less than that and pay your men 6s. a day.

HiOI. You do not do the same width of cutting for the narrow gauge ?-No; the formation width is 9 feet in the narrow gauge, the 2 feet, as against 13~ feet in the case of the broad; but th::tt makes very little difference where yon have deep cuttings to take out, because the bulk of the quantities is in the slopes and not in the centre, and the slopes are the same, no matter what the gauge is, or the formation width.

1502. In rocks you do not want any ?-It depends on the rock; we have to make some rock as fiat as I! to I, to save slips.

I503. As to bridges and trestles, 'vhat do you estimate them at per foot lengthways ?-It depends altogether on the height of the trestle or the bridge. Timber bridges cost from 20s. a foot up to £5, and even more, if they are very high.

1504. What would be the height of the average bridges going to Walhalla ?-Sorne of the trestles are 40 and .50 f6let, or more.

1505. Say 40 feet high and 200 feet long, what would be the price ?-I cannot tell from memory, but I will send the information-the gauge would make no practical difference in the cost of the trestles, because if you have to build a trestle 40 or 50 feet high you must take the timber growing in the country, and must have stability and strength, and provide againet rot, and it makes very little difference what the gauge is in that case.

1506. Where wonld you get the timber for the Walhnlla line ?-Most of it in the vicinity-there is some fair timber there.

I.507. Would not it be better to take the ordinnry timber in the district, even if it had to be renewed every four or five years at a nominal cost than to have to carry it hundreds of miles, even if it lasted four times the time ?-It depends altogether on the first cost. Tbe labour of erection and preparing the timber is the same, no matter where the timber eomos fi·mu, nud that is a very lnrge element in the cost of the bridges; and if yon take the inferior timber grown in the locality nnd adtl the cost of labour to it, it brings the cost sometimes to nearly the same as good timber brought 50 or lOO miles by rail.

1508. What would be the life of the bridges made of the timber in the locality ?-They would last fifteen or twenty years.

I509. It would cost nearly a9 much for a bridge on the narrow gauge as on the broad ?-Yes. 1510. Do you use the same width of ballast on the broad and narrow lines ?-The same depth but

not the same width. I have supplied the Committee with diagrams showing the permanent way, ballast, and sleepers, and the cost of a mile of railway in each case.

15IL You have had no occasion to alter your estimate since ?-No, my estimates are based on a very careful eonsideration of the whole subject.

92

1512. If we had asked for an estimate for this line twelve months ago, would it have been as high as it is to-day?-Yes, from me it would, because it is a. line that in framing estimates for which I should consider should not be constructed immediately, and I would have provided for the contingency of a wage of 6s. a day.

1513. You put down 3s. a. yard for the ballast for the narrow gauge and 2s. 9d. for the broad?­Yes, because there is so much less on the narrow gauge that you would have to pay a higher price per yard for the less quantity.

1514. We never find any difficulty between getting 500 yards of metal and 700 ? -We find a differ­ence. If I le1; a contract for 5,000 yards as against one for 10,000 yards, I will get the latter contract at a less price per yard ; and there is more trimming per cubic yard in putting on a small quantity of ballast than a large quantity-there is more work on it per yard, laying the ballast on, and packing it, and laying the sleepers.

1515. It would surely depend on where you were to get it?-Yes. I reckoned that as an average price for getting ballast from the cuttings and cou11try along the line. Of course an estimate is only an estimate; it is a speculative thing altogether, but those are the rca~ons why I put a larger price ag1tinst the smaller quantity for the narrow gauge. There is more labour in putting it on the line, and in packing and trimming in proportion.

1516. Vfhat struck me wns that you reduced the weight of rail from the ordinary weight used, 60 lbs. to 40 lbs. ?-I was comparing two railways, one on a 2-fL. gauge and the other on the lift. 3in., with the same weight of rail. I do not, say I would advise the country to make a railway to ·walhalla with a 40-lb. rail; that is quite a different thing ; lmt when I was asked to compare gauges I put it this way­you can build your 2-ft. gauge railway with a certain weight of rail at so much, and I eau give you a 5ft. 3in. gauge railway with a 40-lb. rail at so much, anu the latter will be twice as efficient as a railway on the 2-ft. gauge.

151'7. In comparing the two lines, what do you think would be the diHerence between the weight of the two engines ?-I have also supplied the Committee with n diagram ~:howing- the weights of the engine!! that can be used on different gauges. For a 2-foot gauge the heaviest engine you can use is about 20 tons. With a ,)ft. 3in. lino, if you like to provide a suitable rail, you can use engines np to 100 tons.

1518. In that case, ·why do you want a 40-lb. rail on the narrow gauge ?-Bccanse experience shows that for an ,,fficient 2-ft. railway it is desirable to have a rail not less than 40 lbs. to the yard. That is the experience on the Festiuiog Railway, and so it is on some of the Indian railway.; where they have adopted the 2-ft. gauge, and you could not work economically on a lighter rail. It would not snit a 20-ton engine on a 2-ft. gauge because the rail is not well supported ; your sleepers are short and your ballast is narrow, and your rail is not so well supported as on the broader gauge, and therefore yon must put more metal in the rail to carry the same locomoti vo on the narrow gauge. I have provided for a rail to suit a 20-ton engine.

1519. You would not advise a 20-ton engine to be put ou the broad gango to Walhalla ?-I would, if it were sufficient to do the work, because a heavier engine would increase the working expenses. It depends altogether on the amount of work to be done on the railway, and the gmdes, as to what weight of engine you should use.

1520. Why do you recommend a broad gauge; although there is only £2,000 difference in the construction per mile, would it not be better to save that £2,000 a mile and do with a lighter engine and the narrow gauge ?-It would if you conld provide an efficient railway at a lesser price, but in my opinion yon could not, and you ha.-c to make a railway, not for the present only, but for pl'ospective require­ments. I mentioneu in my letter why I prefer the broad gauge for the district, because ultimately that line will have to be exteuded.right across the Dividing Range to connect with the line from .Alexaudra.

1521. Yon are not talking seriously about the requirements of this generation '!-No. In designing railways I thiuk an engineer should take a view 30, 40, or 50 years ahead.

1522. Supposing you saw the community starving for the want of railways to carry their goods and products to and fro, wonld it be a goocl thing to say, "\Ve cannot, until the time arrives when we can construct a line hundreds of miles beyond yon, relieve yon "?-No. I do not take that view at all. I prefer the 5ft. 3in. line, because it can be made nearly as cheaply as the other, and when you take into con­sideration the break of gauge nnd the cost of transferring goods, the balance is in favour of the 5ft. 3in. under present conuitions.

1523. You have no 40-lb. rails in the country ?-No. 1524. What is the lightest rail you have in stock ?-We have no large quantity of usable 50-lb.

rails, but have a considerable mileage of second-hand 60-lb. iron rails, and that is n mil that might be used for unimportant county li.nes in easy country. A 40-lb. steel rail would be the equivalent of a second­hand 60-lb. iron rail.

1525. How many mile(have you got of those ?-I cannot tell that from memory; perhaps 200 miles.

1526. Have you any idea what is~the value of those rails to the country, if they are not used on any of the lines?-They are only the value of scrap-iron, perhaps £2 lOs. to £3 n ton at the most.

1527. It would be well to use those; do yon recommend to use those for 5ft. 3in. new railways?­Yes, in easy country, with light traffic, I would use them.

1528. Why have t.hey been taken up?-They have been taken up in recent years to lay down 60-lb. and 75-lb. steel rails for the heavier traffic, and there has been no opportunity of using them in new railways since; but I do use them for sidings on the new lines that I construct now.

1529. Have anv of those been sold ?-I think not; not any usable 60-lb. rails. 1530. Have any rails ?-We have sold useless, worn out rails to saw-millers iu small quantities. I

understand no good rails have been sent out of the country. 1531. 1ff1'. Woodroffe.-Some double-headed rails were sold at a good price in Englanu, 35s. to £2

a. ton. They went home free as ballast in the wool ships ; they are not all gone yet. 1532. By the Chairman (to Jli;r. Woodroffe).-Are those quite useless for tramways and saw­

millers ?-Quite useless. They are very uneconomical if they are worn out, and you put them down, and the best use you can make of them is as scrap iron.

93 Francia Renniok, 8th August, 1895.

1533. What do you get from the saw-millers for the other second-hand rails ?-The recent price is £3 10s. a ton, but an arrangement h~s been made to lend them at a certain rate; we encourage making those sidings. It was under those conditioH'! that a _great portion of that Wandong line was put in. I think they pay 6 per cent. or 8 per cent. on the v::tluo; some of them have been sold right out.

1534. At what do you value those you lend ?-At £3 lOs. a ton. 1535. Have you ever seen that Waudong line?-Yes, I have been over it. 1536. Were you afraid to risk your life on it ?-We came along in the dark-we went in daylight,

and came down by the gTavitation in the dark. 1537. You had not much jolting '?-There was a good deal of it. 1538. Did you go behind the Comet mill into the forest?-No further than the mill. 1539. Did yon see the new bridges beyond the mill?-Yes, we saw the bridge across the gorge. 1640. (To tAe witness.)-Do you inspect a line after it is surveyed always. 1641. "We have seen the narrow-gauge line to Wandong, and we were pleased with the simplicity of

the construction, and although not fit to carry any great weight, it does its work wonderfully well, and it has very sharp curves. vVe came down four miles and a half in twenty miuutes, and on a previous occasion in thirteen minutes. It is a very comfortable ride, and we think lines a little more permanent and perhaps a little more ornamental would serve country like V\r alhalla-something a little better perhaps, but not as good as you have iu your mind, that could be done for oue-balf or one-thin! of your estimates, and that it would ~erve for the next ten or t\velve years, and would pay the country to supply those districts with that sort of road rather than keep them starving for the want of it, and last for ten or twenty years. \Ve have reluctantly come to the conclusion that we can never supply most of the country places if the li11es are to cost as mnch ns estimatetl by your Department-you understand the position ?-Quite. I have considered the que~tion from every point of view, autl my opinion is that in hilly country where you cannot mnke a railway with steep grades and sharp curves, such as I would advise for a railway, at a low cost, and, unless it i~ a well developed piece of country, a railway is not required at all, and you need to open roads where yon can use ordinary farmers' carts and vehicles or a traction engine-that is what you want for such a country. Country like that is not ripe for a railway, and, at the most, you should construct a tramway, lmt Temembering nt the same time that on a tramway, if you make it pay, yon have to charge such rates as are not comparable wit;h the rates on a railway. Yon would have to charge, instead of ld. or

per ton per mile, 3d. or 4d. The farmers can cttrt on ordinary roads now at 6d. per ton per mile, and the country will not pay for a railway you should help them with their roads.

1.542. How can they make the ro!tds if they have no material in the locality-they make tramways and light railways in other countries cheaper than wo can make a road ?-Let them do it then if it can be done.

1543. You say municipalities should make those roads-that would be impossible, because I tried to get some of those roads made some years ago, but we were so harassed in the Department when we sub­mitted an e~timate that what we could get done for a very small amount they added to it about 200 per cent. and would not budge an inch unless we agreed to their estimate ?-It comes to this: the other alternative is for the State to make you railways at a. high cost. It is better for the State to contribute one-fourth or one-tenth of that money to the localities and let them carry out local roads and tramways, and wipe their hands of the whole thing; because in working the milways it is not only the first cost, but they are worked at a loss every year on the freights now charged on tratlic. It is better for the localities to be subsidized by the Government to carry out 5nch local works.

1544. By 1lh. Trenwit!t.-Uo you think the estimates that you have given might be materially reduced if there were ttu nndertakillg that the rate of speed was not to be more than, say, eight or ten miles an honr ?-No.

1645. By the Hon. D. Jfelville.-Frorn the ~election of lines made by the Mini~ter, the Orbost and Walhalla, arc we fairly to judge that in your opinion those are typical of the colony ?-It depends altogether on the eharactcr of the conntry. They are typical of only moderately difficult conutry.

16,16. The vValhalla line woultl Le oue of the most tlitlicult that you would have ?-No; seven or eight miles n.re lliffieult, but not the most dlfficnlt in the colony.

1547. l11 yonr opiuiou the selection i:; fair?-Yes; my estimates and the mrveys are for lines of fair efficiency on the two flanges.

1548. Anti the conditions of our Victorian country is represented by the Orbost and W alhalla country ?-Yes, for such country as those two in any part of Victoria.

1549. Then yom estimate woul•l comprise the Orbost country and all country of a similar nature; tho total cost per mile of the one gango being £3,460, and of the other £2,897, or merely a difference of £653 per mile. Will the Committee be safe in jndging of all country similar to Orbost in the same ratio?

for almost identically the F.ame. l5i)0. And in extre~nely hilly country you show that all eonntry similar to \Valhalla the total cost

of the line in the one case is so uear tlmt it costs and in the other £1-17 ,23:l, or leaving only a of about £1,315 ?-That is taking into account the transfer station and workshops for the break of

gauge. 1651. Is that final ?-That is a £air estimate for that particn!ar case. 1552. And for similar country ?-If it were identical eountry, of course it would be the same.

The witness withdrew.

Adjourned to Tuesday next, at eleven o'clock.

94

TUESDAY, 13TH AUGUST, 1895.

Members present :

MR. CAMERON, in the Chair;

The Hon. J. Buchanan, M.L.C., Mr. Burton, The Hon. D. Melville, M.L.C., :Mr. Craven, The Hon. E. :Morey, :M.L.C. :Mr. Harris,

:Mr. Tren with, :Mr. J. S. White.

Charles E. Norman, sworn and examined.

1553. By the Clwirman.-You arc Engineer of Existing Lines ?-Yes. 1554. How long have you occupied that position ?-About two years. 1555. We received a letter from your Department the other day signed by the Secretary, Mr. Kent,

which set forth certain things. In examining Mr. Woodroffe the other day he said you could give the information we require on those points ?-It is with regard to the experimental curves and the class of rolling-stock that can run round curves of a certain radius. I gave that information.

No. 1556. Do you think you can run round sharper curves with a broad gauge than a narrow gauge?-

1557. Does it require heavier rails for the narrow gauge than the broad gauge ?-No. 1558. vVhat was the 7 5-lb. rail for ?-It was proposed in case we laid curves of 5 chains radius or

less on the broad gauge, to suit the present rolling-stock. 1559. Would it be necessnry to lay a 75-lb. rail if the curve was less than 5 chains radius ?-Unless

the speed was very much limited it would be advisable to lay 7 5-lb. rails on curves 0f 5 chains radius or under, and also a guard rail.

1560. What is the difference in cost of the 60-lb. and 7 5-lb. rails ?-To lay with a 75-lb. rail and large sleepers will cost about £2 LSs. per chain more than with 60-lb. rails and small sleepers ; that is about £220 a mile. That, of course, is only on the sharp curves. It would not include the easy curves or the straight portion of a line.

1561. vVould it be safe to run a sharp curve of less than 5 chains if the speed were lessened to seven or eight miles ?-Yes.

1562. Would there be any difficulty in going at that low speed for the distance of the curve ?-It is always a difficult thing to limit the speed of trains. vVhen they get certain loads on on certain gradients they will exceed the limit of speed at times. I think we ought to provide against any probable running at a high speed round the curves.

1563. You would not require to guard against it if you use 7 5-lb. rails and a guard rail ?-That would not be a provision against high speed.

1564. Would not the other provision be cheaper by slackening the speed, the same as you do coming into a station ?-You might slacken the speed generally, but you would find at times they would run at a higher speed than your limit. We fiud that everywhere where we try to limit the speed at times the drivers will exceed the limit.

1565. That will apply to entering a station as well as going round a curve ?-No ; at a station they have to pull up. They slow down because they have to stop altogether. Express trains run through stations now at fast speed.

1566. If the regularions were that they were prohibited from going at that speed rounu a curve would it not be possible to compel every driver to adhere to those regulations ?-Occasionally the limit of speed would be c"ceeded, as it is now. It is not possible, either here or in any other country, to compel a driver to limit his speell to five or ten or fifteen miles an hour. Occa~ionally the speed will be exceeded. Various conditions come in, the state of the rails, the rails being wet and that sort of thing. The brake may not act perfectly well, or the rails may be greasy, or there mn.y be reasons of that sart that give an excuse to the driver if he does exceed the SJlCcd.

1567. Whn.t is the steepest grade you would be prepared to travel up on the broad gauge in a mountainous country where you could only get n very steep grade-could yon climb up 1 in 15 ?-We could with the aid of a rack rail. It would not be payable to trust to adhesion only with a 1 in 15 grnde.

1568. What is the lowest ?-One in 20 or l in 25. I would say 1 in 20 is about the limit of adhesion grades.

1569. Have we anv of those in Victoria ?-No; we have l in 30. We l1ave small pieces going up on to coal stages where it ·'is more than that; but that is all.

1570. You have no experience of l in 20 ?-No, not as 11 working grade for a railway. 1571. Have you any experience at all of a narrow-gauge railway?-Vv e have never had any here.

I have seen the narrow·gauge railways of the other colonies at work. I have travelled over them, and I have also seen nm-row-gauge railways at work here for temporary purposes.

1572. vYhat is the gauge generally for temporary purposes ?-Latterly for temporary purposes it is about 2 feet.

1573. How do they act ?-Very well. 157 4. Are they safe?-Yes. Where 1 have seen them in use principally was on the West Melbourne

Swamp works, where the tracks were picked up and laid uown wherever required. 15'75. At what speed do they go there ?-A slow speed. They did not travel at any part more than

four or five miles an hour. They took a long train of waggons behind them. 1576. As many as forty '!--I never counted them; it was a long train. 1677. What was the sharpest curve on the swamp ?-I should say there were curves of a chain and

a half to 2 chains radius, but I never measured them. Li78. Would it be safe to rnn a train teu miles an hour on curves of 21-, or 3 chains radius with ten

or twelve carriages or trucks on a narrow gauge?-Yes, I think so. -1579. What would be the width of those carriages ?-About Gft. Gin. 1580. What rails should be used on that line ?-Not less than 40-lb. rails. I think that would be

sufficient for a narrow gauge.

95 C. E. Norman, 13th AugUBt, 1895.

1581. You do not know what height you could climb up with the narrow gauge ?-It could not climb up any steeper grade by adhesion than the broad gauge. It would climb up as steep a but it would take a less load.

1582. It could go up 1 in 20 ?-Yes, engines could be made to climb up I in 20. 1583. You would risk your own life in going round a curve of l in 2~ or 3 chains at a speed of ten

miles ? -Yes. 1584. What is the cost per train mile of the narrow gauge? -I have not got that information; I can

get it for you. 158.5. What is the cost of maintenance of the broad gauge ?-The average cost on the Victorian

railways is about £110 per mile per annum at the present time. 1.586. How many are in the repairing gangs ?-Generally three, sometimes four. 1587. How many miles do you allot to a gang ?-It varies considerably with the class of the line

and the amount of traffic. "With the lightest traific we have we give about two miles and a half to a man. 1.588. What is the lightest line you have ?-Not more than one train each way daily. 1.589. Have you many of those i11 the country?-Yes. \Ve have some where there is only one train

a day, three trains each way per week. 1.'590. Take the line from Hinl!wood to Healesville; what do the men have there ?-On the line

between Ringwood and Lilydale there are eight trains each way daily. There we have a man to every mile and one-fifth. On the line between Lilydale and Healesville there are three trains each way daily, and we have a man to every mile and seven-tenths.

1591. Are there no goods trains on the Healesville line ?-I do not know w·hether there is a goods train now. This information is from a returu made up to the end of December, 1894, of the number of men on every line in the colony.

1592. Is that a difficult line to keep in repair ?-It is a wet country, and there are three trains a day on the line each way.

1593. It is fairly level between those places ?-No; there are 1 in 40 gradients between Yam1. Glen and Healesville. We have far easier lines than that to maiutain.

1594. There are no land-slips there or anything of that sort ?-No. 1595. By the Hon. D. Melville.-Referring to the West :Melbourne Swamp ; if you had a dock of

similar proportions and "Were the contractor, would yvu adopt the 2-ft. gauge or the 5ft. Sin. ?-If I had to haul the material out of a deep dock I would hanl it out on the Mt. 3in. gauge.

1596. But take that particular instance ?-I would haul it out on the 5ft. 3in. gauge. I would not adopt the 2-ft. gauge for the bottom of the dock. I think the contractors took out all the bottom on the 5ft. Sin. gauge.

1597. Did he alter it ?-I do not know whether he altered it at any time. I know he had the 5ft. Sin. gauge laid throughout the bottom of the dock, and he used to work the large engines with the 5ft. 3in. trucks.

l.'i98. Heavier engines would take more trucks ?-Yes. 1.599. Would it have been more economical to adopt the .5ft. 3in. coming up the bank ?-Yes. 1600. What is the depth of the bank ?-I suppose the bank was about 2.5 feet from the surface of

the swamp. 1601. They had to climb up the 25 feet with a short run ?-Yes. I dare say the principal reason

he had for adopting the narrow gauge was because of the easy manner in which the trucks were unloaded. They were tip-trucks.

1602. About the cost of laying on the surface a similar railway to Robb's dock railway for ten miles, can you give an estimate of what that would be, simply laying it on a plain, the same as Le did. What would it have cost :Mr. Robb per mile to lay that 5ur£aco rail way for five or ten miles under similar circumstances ?-That depends upon the class of rails usetl.

1603. You gave him the rails?-Yes; it was laid with our old rails. The sleepers were his own. 1604. What would it have cost you to make ten miles of that, as he made it, on the surface ?-The

bare rails and sleepers and laying would cost about £600 a mile. 1605. Then you would have to add the ballast ?-There is no ballast on this line. 1606. What rail is that ?-I am estimating for 50-lb. iron rails at the value of £3 per ton. 1607. The 2-ft. railways are univcrsalnow?-No; I do not know that ru·e universal. 1608. They are used in Germany, Saxony, Italy, and India. Do you know why 2-ft. railways are

made instead of .5ft. Sin.?·- To save first eost, J presume. 1609. Are you sure of that ?-I think so. 1610. The estimates the Railway Department have sent in do not appear to bear that out ?-I have

not seen the estimates ; I know nothing of them. 1611. Do you think they can be constructed to save first cost in Victorirt ?-I should say so, certainly.

I have already given evidence that in f1at country a 2-ft. gauge railway with 40-lb. rails wonld cost about £650 a mile less than a 5ft. 3in. railway with 60-lb. rails.

1612. Do you think that we should construct all our spur line~ on that gauge J do not. 1613. Supposing we couhl mnke a line like this swamp line on those spurs that the eountry is asking

for for .£600 a mile should we do it?-You could not do it for £600 a mile or nnvthirJO' like it. I could uol; an estimate of what they would cost without knowing the country or • a st~rYey of the country through which the line is to be mnde.

1614. Jly ilh. Trenwitlt.-You have said that on le;·el e01mtry the 2-ft. between £600 and £700 per mile cheaper than the broad gauge. A; the country would not the difference become ?-Not greater in proportion to the total cost. There would Le meore difference in the cost because \vonld be more difference in the earthwork, but in proportion to the total cost the difference wonld be less though the aggregate difference would be

161.5. Bz; the Hon. D. Jfeldlle.-\Vbat is your advice respecting those spurs-should they be a cheap broad gauge or a 2-ft. gauge to attain a lessened cost of construction ?-"l\fy opinion is that if it is worth \vhile to make a railway at all it Ehould be made on the broad gauge, and further, if yon make a railway on the narrow gauge, in order to pay working expenses you will have to charge higher rate;; than the ordinary rates now charged.

C. il;. Norm&n, lath August, 1895. 96

1616. That is a wonderfully stereotyped answer by the Department ?-That is the answer I have given before.

1617. Why do people construct those 2-ft. railways if that is really the case ?-They charge special rates on them, I have no doubt.

1618. Why should they do that when they can do the other ?-To save outlay in the first instance. 1619. Then why have the broad gauge ?-If you have traffic enough you can carry it far more

economically on the broad gauge than ou the narrow gauge. 1620. By .Llfr. Trenwitk.-If you had traffic enough for a small railway but not enough for a broad­

gauge railway, is it not possible for a narrow-gauge railway to pay where a broad-gauge could not ?-Not unless you charge special rates on the narrow-gauge railway.

1621. How do you arrive at that conclusion-is the cost of haulage proportionately dearer on a narrow gauge than a broad gauge it must necessarily be so because you have to use far lighter engines, and, while all your labour expenses are the same, you cannot haul more than 25 per cent. or 30 per cent. of the quantity that you could haul on the broad gauge.

1622. If you have to use lighter engines is it not necessarily a cheaper engine, and consequently an engine upon which the interest on capital is smaller?-Yes.

1623. Does not a smaller engine consume a much less amount of coal, so that the expense of drawing is smaller?-Yes.

1624. Is it not possible that those factors, though you have no reduction in the cost of labour, may cause a considerable saving ?-Yes, there wouhl be a comidemble saving in the interest on capital and on coal, but the saving would not be proportionate to the reduction in the load. You would not save 75 per cent. of the coal or 7 5 per cent. of the interest on capital, but you would take 7 5 per cent. less load.

1625. Suppose you had a district in which your broad gauge is running up to half its capacity, and consequently losing, is it possible that if that were a narrow gauge with less cost of construction of rail­way, a smaller engine, and less consumption of coal, the half traific for the broad gauge might pay the narrow gauge ?-No, not at the present rates. I am of opinion that a district such as that should not h11.ve a railway.

1626. By tl.e Hon. D. ,}Jelville.-Would you give us a district that is up to the capacity in your mind's eye, and should have a rail way constructed on the 5ft. 3in. gauge? -Take the line from Morwell to Mirboo, that is in a rough part of the colony. I should say that line justifies its existence.

1627. Can you give me another ?-Take the Moe to Thorpdale line. The district there is good enough to justify the construction of a railway; that will not pay immediately, but it will very shortly.

1628. Do you not know that that is one of the worst cases we have ?-The country is improving every d:1y, and before very long the railway, I think, will pay as well as a branch line of such a short length eau be expected to pay.

1629. It does not justify itself now. Can you give a clear case that illustrates your idea-that has justified its construction ?-The Morwell and ::Viirboo is one that justifies its construction.

1630. Are there any more ?-Yes, there are others. Take tbe line from Tallarook to Mansfield; that does not pay at present, but it very shortly will I think.

1631. Does the Morwell to Mirboo line pay ?-I fancy it does. The Thorpdale line does not pay at present.

1632. If they are not paying at present should we make further experiments of that nature at present ?-I do not think any line should be made unless you have a reasonable prospect of paying interest and working expenses within five years after its construction.

1633. Have those paid in fiTe years ?-The 1\<iansfield line has not been opened five years yet. 1634. The Thorpdale line does not pay the cost of working ?-The country is improving every day.

It must have a chance to be opened up. 1635. You have constructed the broad gauge according to your idea at Thorpdale, and it leaves a

loss of £5,000 a year-if we put the narrow gauge there would we lose as much?-You would not lose quite as much perhaps h: interest, but you would lose quite as much in proportion to the traffic you carried.

1636. By 2rb·. Trenwitlt.-Would the narrow gauge be sufficient to serve the requirements of that country up to the present I think so.

1637. Would the aggregate loss per annum be £5,000 a year with the narrow gauge ?-I cannot say without making an estimate of the cost of the narrow gauge as compured with the broad gauge.

1638. The first r:ost would be less, the cost of working from the fuel point of view would be less, interest on engines and rolling-stock would be less, and the amount carded would be as great. The working expenses and labonr would not be greater·, consequently it seems obvious that the aggregate loss must be less ?-Yes, except that you are taking the country and the line as it is now. When the country develops there should be more tmffic upon that line than a narrow-gauge line could carry profitably. The present loss would be less probably on the uarrow gauge than on the broad gauge, but as the country improves the loss would be more on the narrow gauge than on the hrmtd gauge.

1639. By the Ghair-rnan.-ls it on account of the coal that the prospects of that line are good ?­There is a little coal there, ttnd there is also good land.

1640. How much have the products of the land apart from coal brought to that line ?-I could not say.

1641. By the Hon. D. 1l1elt,ille.-Am I to understand clearly that if 've had similar lines to the Thorpdale and :\ioe, one on which there is a loss of £5,000, your recommendation is that of the two systems you would prefer the 5ft. 3in. gauge ?-Yes. ·

1642. Is there any other line that will illustrate your meaning better than that one ? -We have lots of branch lines that pay; they mostly run in flat country.

1643. You would not give any district a railway unless they gave a loss of not more than £7,000 a year ?-I cannot put it in those terms. I say when a railway is made there should be a reasonable pros­pect of paying working expenses and interest within five years of its construction. Those railways have been made into a very rough and thickly timbered country, and it takes longer to develop such a country than it does the greater part of the colony.

1644. Will not those railways eat us up, root ~md branch, before we get to the paying point?­There must be a limit to the number of railways that are constructed.

97 U. E~ Norman, 13th August, 1895.

1645. With all the loss, you stick to the 5£t. :iin. gauge ?-Yes. 1646. By .the Chairman.-What is the differenee in cost between the 50-lb. iron rail and the 60-lb.

steel rail ?-There are practically no iron rails made at all now. I take the rails at £3 a ton as being the valuation on the rails that we have on hand. Thev are second-hand mils. That is tlw elass of rails that was used on the swamp by Mr. Robb .. I think :VI;, Melville asked me .for a similar line to that laid down on the swamp by Mr. Hobb. . .

1647. Suppose it takes .£8,000 per annum to pay for a twenty-mile railway with a broatl-gauge !me, how much >~"ould it cost for a :oimilnr distance with a narrow-gauge line ?-I eould not tell without making a separate estimate of the cost of the two lines.

1648. By 1lfr. Craven.-I notice on the }foe to Thorpdalo line, Lluring- the yt•;;r ending 1894, the outwards goods amount to 21,000 odd t(!ns, inwards to 1,300 tons; :Ylorwell to )\o,·th :Yltrboo, G,OOO tons outwards, and 1,:iOO tons inwards; do you think a tramway similar to the one from Wamlong, roughly con­structed with the aft. 6in. gauge that you could use common trucks on, would be quite sul1icient to develop the traffic up to a certain point, nnd prefomble to the broad gauge, until sueh time as the cotmtry developed, to enable the broader gauge to bo introduced ?-The vVawlong tramway is J1t for nothing but a horse to work. It is not fit for a locomotive.

1649. By flfr. Trenwitlt.-Not the first five miles from Wandong ?-No; they do it, but it isuot safe to do it.

1650. By the Cha.irman.-Could it not be made ~afe with a little extra cost ?-It could. 1631. Do you know what that tramway cost ?-No. 1652. The portion on which the engine runs cost about £150 a mile, apart from the rails. If you

added another £150 to that it would make it more solid ?-Yes, it would improve it ; l am not prepnred to say that would make it safe. The rails are not curvea where it is neeeosary, allll the whole line is laid very badly indeed.

1653. By 1rir. Craven.-Do you think a horse tramway would be quite sulfieient in many instances where we have railways to bring out the traffic, and it would be quite enough to have built a horse tram­way in tho first instance ?-There may be one or two railways in the eolony whieh do not carry mnre traffic than a horse tramway eould take, hut those railways should never have been construeted.

1654. You would not have lmilt those railways at all ?-No. 1605. To places that will not give more tonnage than t.ho two eases that have been mentioned woultl

you give a railway?-The Thorpdale line has about 7:) tons a day; that me~ms, say, ten truek loads. 16,16. Seeing that you can with a horse tramway, ou a !'(!Ugh cheaply eonstmotetl Jine, carry 12,500

tons per annum, you tlo not approve of establishi.ng smuothiug on a rougl1 way iu tho first instance, and then, when you have established the tratfte, putting dowu a proper lino !-Xo. E you are goi11g to work it as a railway it ~houhl be laid so that it can be worked safely. If you are going to take tl1e re6ponsihility of carrying passenger traffic especially you must hn,·o the line safe.

1657. You favour the system of development by roads or tmmwnyH, proYidNi they me put under the local couneils, so that the State slwulu have uo respou.;ihility at nil ?-If they are lnid clown !:limply as horse trnmways they could never be made to pay unless high rate:; were clmrgetl on them. I think if it was likely that horse tramway:; would have paid for themselves wonl<l b~,vo hoeu nmdo long before this. The districts themselves woul<l have made them. They h:.rre ;wd the power for some years to make tramways.

165H. By the Cl,airman.-Tbey did not avnil themselves of the )JOWor?-No. 1659. By the lion. D. 1~1elville.-vVlmt b the difference between a horse tmmway anu engine

power ?-I have no actual estimate of the differo11ee in eost of hnulage between ::. horse tmm and a lino that woultl earry an engine and pa~songer tmiu safolv. Yon have soou the \Yamlong tramway; that is a horse tram. That is a very badly eonsti·netcd line. •

1660. Do you know that the actual co:;t of the 12,000 tons there is between l<l. anu 2d. per mile, with thio addition that if he could double ltiH traffic he eonld do it for Ios'' ;cJHl buat tlw Vietorian railways? -If you have only about 0110 wnggou n day to hn.nl np null ono io haul <luwn you eau do it cheaper ,;ith a horse than with a locomot.ive. Twelve thousaw1 ton,J a ycnr would be about JO tons n day; that would be two trueks each way. You could haul that choaver with a hor~e ~lmu will1 a loeonwtivo. . . 1661. Would you have made fc railway from \Vandong to that plaeo if you hat! ha,L the settling of 1t ?-Not from what I saw of the couutrv when I was there. I do not think the country is good enough to justify the construction of a railway. •

. 1662. By Jlh. Burton.-Have you any objcetion to a :,;peclnl rate being if a narrow-gauge railway wer~ construeted ?-No, except that there ib always a ditliculty in maldug one LliRtrict in the colony pay lug her rates than other distriets pay. WhoneYor differential mte~ nre pnt ou they are generally taken off in a corn11aratively short time.

1663. Do you know any caBo where they have lJo<.m takea oif' boCvr·e the circumstanees warranted it ?-No.

1664. If the inhabitants of a particulnr locality were willing to submit to the impo~ition of a speeial rate would you have any objectiou to construct a narrow-gango railway to that locality unless for the reaso~ ~hat if n railway is justifiable the broad gnuge would pay better tlmu tlw nnnow one.

166o. You would favour the eonstmeiion of a nnrrmv-O"auo·e railwav if a sp;:;clal rate were to he imposed ~-Provided the traffic wmo :;o smnll that :t would not"justil'y the e~ns!ruetion of a broad-gauge; hut that ~~ a plaee I still think a railway is not ju>~tifbble to.

1666. By ilfr. l~arris.-:Ylight not a 2ft. ()in. railway be justifialJle for all purposes that were appare.nt where ~ .:>it. Bm. would uo out of the qacstion altogether ?-~There i,; not so mnch difioronce in cost of construetwn between a 2-ft. or a 2ft. ()iu: gauge milway aBLl a ;)it. Bin. milway.

1667. Can you run round sharper emvcs with a uarrow thnn wlt!t a broad gauge '?-Yes. 1668. 'What radius could yonlimit tJJo nn.rrow gauge to ?-In some parts of the ·world they mu rounll

very sharp curves indeed-perhaps 50 feet or 60 foot radius. \'le have l'il'll1illLT in tho city of Melbourne 4ft. 8~in. gnuge lines with curves of abont I 00 feet ratlius ; that is on the 1\folh~urne trmnwavs.

1669. You do not think narrow-gauge railwuys would be more cnitable for sharp eurves thau broad­gauge?--:="'o; I_say you can nm round sharper cmves on a narrow gaug:J than ou thebroau gauge.

16zo. It ts admitted for difficult eountry the narrow gauge is most ~nitable ?-1 do not adroit that. 161 l. You eau run rouua sharper curves ? -Yes.

NARROW G.A.UGll. H

c. E. Norman, 13th August, 1895. 98

1672. Having a difficult country to deal with, it might be more expensive to construct a broad gauge than a narrow one ?-Yes, it is.

1673. Having regard to the cost of construction and the expense afterwards attendant on the broad gauge, is it not certain that the narrow gauge would meet all the requirements of a locality where the broad gauge would be a failure ?-I have already stn,ted that unless there is a prospect of sufficient traffic in five years to justify the building of a broad-gauge railwav I do not think a railway should be constructed at all.

1674. :Might not there be a prospect of a mtr;ow gauge paying for a considerable number of years, where there would be no prospect to justify the broad gauge ?-No.

1.675. Suppose the North Mirboo line had been constructed on the narrow-gauge principle; is it not possible that it would have been paying at the present day ?-No, it would not have been paying.

1676. The total cost of construction was about £150,000 for twenty miles; if that twenty miles had been constructed on the narrow-gauge principle would it not have answered all the requirements up to the present time, and perhaps for many years in the future ?-It would have carried all the traffic up to the present; but as the country developes it would not carry the traffic so cheaply as the broad gauge would carry it.

1677. Not if there were sufficient for a broad-gauge line to do; l1ut judging of the nature of that country would not a narrow gauge for many years to come meet all the circumstances of the case ?-No doubt a narrow-gauge railway would do the present tmffic, and would probably carry the traffic for the next few years ; but I am quite satisfied in my own mind that that country will so develop that in time it will be found that the narrow gauge would not carry the traffic.

1678. By Mr. Cmven.-Would not the cost of the improvement of a narrow-gauge line, when you reach the time that the line must be laid down on the broad gauge, mean practically a new line altogether? -Yes.

1679. By .llfr. Trenwit!t.-Would not the earthworks do?-A portion of them would; but if you put in easier curves you would have to alter the earth works in a great many places. The line would be located probably in different places in many cases.

1680. By 1lf.r. J. S. Wltite.-Before the Mirboo line was constructed there was an estimate of the probable revenue made, was there not?-I do not think there was.

1681. Do you know that the ~Iorwell line was estimated to cost less than £60,000 originally; in view of the Government estimate of under £60,000 is it not likely they made some estimate of the total revenue they would get ?-I cannot say.

1682. If the Department thought it ;vas necessary to communicate with Mirboo and North :Mirboo, when they found it was costing double the amount which the broad gauge was estimated at, would it not have been well to construct a much lighter line on the narrow gauge until the traffic developed ?-They could not construct the narrow gauge for half the amount of the broad gauge.

1683. If it cost double the estimate, was it advisable to make the line at all ?-That country must be developed somehow. At the time the Mirboo line was made there was no line south of the main Gippsland line, and I consider the construction of the line was justifiable.

1684. If that line was justifiable, and does not pay, does uot any other part of the country justify being opened up ?-I have not inspected the different parts of the country to be able to say as to that.

1685. By Mr. IIarris.-Do you know of any line to construct in Victoria at the present day that would pay on the broad gauge ?-I think some of the mallee extensions will probably pay in a very short time on the broad gauge. I am only speaking from what I have seen travelling over the present mallee lines. I think the country is good, and is a long way from railway facilities ; and there is every prospect of sufficient traffic to pay for the construction of a railway.

1686. That is, having regard to the distances the lines are apart from each other?-Yes. 1687. By the ChairmaJ<.-Do yon know the distances those people in the mallee are from a railway

at present ?-Only from the map showing the good land and inferior land. From looking at that map I have an idea that some extensions of the mallee will pay in a very short time.

1688. By 1llr. Harris.--You say there must be a limit to rnil way construction; what do yon mean by that; is it not a question of whether the game is worth the camlle ?-Yes, but it is not likely for many years to come in this colony to pay to construct lines very much closer together than they are in the mid­land district. Take the district bounded by Inglcwood, ·w oodeud, Bemligo, Ballarat, and Maryborough; the lines there lie as close together as they should be for many years to come.

1689. By Jlir. J. S. TVhite.-One line was to be constructed from Beaconsfield towards Gembrook ; that is very rough country, and ItS it would not be likely to pay to open np that country with the broad gauge, would it not be worth while doing it with the narrow gauge ?-I have never seen any of the particulars. I do not know anything about the smveys that have been made in that country.

1690. By }rfr. Harris.-Would a 3-chain curve on the broatl gauge be as safe as a 2-chain cm·ve on the 2-ft, gauge ?-Yes, with suitable rolling,stock it would.

1691. You are aware that they have a standard eurvc for the different gauges in India ?-No, I have never seen any list of standard radii of curves in India.

1692. You say they adopted the 2-ft. gauge iu different parts of the world as a matter of cheapness in first cost ?-Yes.

1693. Is it not a fact that they adopt them not because of their cheapness altogether, but because of the necessities of the case and the desirability of meeting the people's wants ?-That resolves itself into a question of cheapness.

1694. If there is almost a consensus of opinion in different parts of the world that it is essential to open up the country places with a cheap railway that will last for many years, shonld it not be seriously considered whether that system cannot be adopted here ?-Cheap narrow-gauge lines have been adopted on the Continent to a certain extent. I do not think there arc any in America, or very few.

169.5. By Mr. Craven,-As the principal for railways in a lot of the outlying portions of the country is for the purpose of carrying goods aml not passenge1·s, assuming a line on the 5ft. 3in. gauge were made for the purpose of' only carrying good::1, nnd excluding passengers for fear of accident, could you make a line of that description at a much lower rate ?-No. If you want to run a locomotive over a line, it must be safe to cnrry the fireman and driver.

1696. It must be constructed safely?-Yes.

The witness withdrew.

99

Francis Rennick, recalled and further e:x:l1mined.

1697. By the Chairnuzn.-You are Engineer-in-Chief of the Victorian Rail ways?-I am. The question was put to me the other day, ".Must the undeveloped bush districts wait for cheaper transit until the cost of efficient railways can be met?" That is a very pertinent question, and I have considered it, and will make a few observations on it. For billy country in Australia the stages generally are a pack­horse, a cart track, formed roads, sometimes cut out of the sidlings. Then we come to the metalle<l and corduroyed stage. In America there is another stage where they make plank roads in timber country ; they cut up the trees into planks, and make plank roads. I have often wondered why it iR they do not do the same here ; they are expensive to construct, but they are very useful roads ; they last for five or six years before they require renewing, and they are not expensive to maintain. The next stage in the natural order would be bush tramways, with wooden or light steel rails for horse traction; traction engines might be used on metalled roads ; they would haul cheaper than horses-that is another method of meeting such cases. The next stage naturally would be light railways, and then afterwards, when the country is thoroughly developed or well developed, and for the heavy traffic there would be substantial railways. These are the natural stages in which new country can be best developed; but very often light railways are asked f.)r as soon as ever the pack-horse leaves the track, and you come to make bush tracks for bullocks and horses. Then you have these numerons demands for the narrow gauge, because every one supposes that the narrow gauge can be made at a tithe of the cost of the broau-gauge railway. When you are at the stage of light railways, in my opinion, it is bad policy to break the gauge to secure a slight economy in cost, because, no matter how you estimate the cost, if it is estimated by a competent engineer, and based on the cost of narrow-gauge railways in other countries, the saving in the first cost of construction between the broall and narrow gauge railway is very small. :My view is that it is better to adopt even a 40.lb. rail on the standard gauge, and sueh a rail will carry nearly all our carriages and our existing trucks, and some of om· present engines, than to break the gauge, and make a narrow-gauge railway, with the expense attendant on break of gauge aml the extra cost of working the narrow gauge. A rail mueh less than 40 lbs. would involve new rolling-stock. If we adopt a 20 or 30 lb. rail we would have to get new rolling­stock ; the present stock could not bn run oH it because the axle load is too great. In this case, if it eau be shown that it is wise to adopt a 20-lb. or 1.5-lb. rail, then the break of gauge would be unimportant; but t0 justify this it must be shown that large economy of construction would result, because it would mean :;mall loads for loeomotives on sueh a rail, no matter what the gradients are, and high working expenses. Where you would not be justified in constructing on the existing gauge I think the project should be left to the local bodies to be subsidized by the St>tte where necessary-that is for bush tramways; or even if it is desirable to construct a narrow-gauge railway, I do not think the State should do it. It should be left to the local bodies, and the State might render assistance; but it seems to me that the broad question of railway development consists of this: how are we to meet the portions of Victoria that have no railways at all. Those portions comprise about two-fifths of the whole area of Victoria; cheap ln1uhtge is essential for the development of the country districts everywhere in the colony, and for those districts to be settled by a fimning and prOllucing population. This, in my opinion, eau be best secured by extending the lines on the present gauge in the form of light railways and, where necessary, adopting steep grades and sharp curves to chcnpen the coHt of construction. You have seen lists of the losse:; incurred on some of our branch milwuys; those losses are made up in this way: the earnings from the local traffic are put on one side and the loeal cost of workiug on the other side, and a balance is struck unfavorable to the line. There is another element that should come into this eHtimate, and tlmt is the estimate of development. A branch railway may uot pay iu it~elf, but if it develops the district and increaser~ the production of the district, and makes tonnage to run along the trunk railway, which may be 100 or 200 miles from the capital, then the results of running tlmt new toiJuage on the old lines should he credited to the branch railway. That is never done, and many of our lines which appear not to pay now are jnstifietl if that element is taken into aceount, beem1se the business on the trunk railway would not be anything nearly so great as it is at present, and those line~ would not pay as they do nt present without those branches. It is said rural railways or local railways at low cost are common in Europe and in India. and why not here ? That is a very natural qnestion. In Europe and in India those lines are made mostly oH existing roads where the cnttings and the banks, the culvert~, and the bridges are already prepared, and there i,; very little more to do than simply to form the surface of the rond, and lay dowH the sleepers, ballast, and mils, so they are made inexpensively in that >Yay. \Ve have no cases of that sort in this eonntry. Where we have to pro­vide such railways we have to take the :;:nrface of the gronud in a state of nature, make onr cutl:ings and banks, and provide bridges and cu[ verts, and in hilly country that runs into the bulk of the cost of the construction; hut even the lines that are made in lndia and Europe under such favorable circumstances cost from £a,ooo to £;),000 a mile. In such ,places the populatiou is dense where those lines are mwle an(l where they are held to be justifiable, and where they do anything like approaching to pay, being from 200 to 600 inhabitants to the square mile. Our population in Victoria is about thirteen to the square mile, and the circumstances ure totally different, ~o that what might justify a metre·gauge milway on a public roatl in Belgium, Fmnee, or Italy would not justify the constmction of a railway here at all. The holdings of the farmer~ or peasant proprietors, as we all know, in those countries are from 2 to .?, 6, and 7 acres ; horses are scarce in proportion to the population, and hor$e cartage ifl dear. Here we have a sparse popu· lation, the holdings are >cry largo, lwrses are numerous, horse-feed is low, and hence the cartnge is eheap, so that in many districts now horses compete successfully with our railways; they .:all carry at 6d. or /d. a ton per mile. Those rural railways in those eonntries are generally short, rarely averaging more than twelve or fifteen miles in length, and they connect rural districts with the market towns or the loeal centres, and the traffic is ehiefly passenger, more passenger than goods~people going to the market towns and transacting their business. They nm from three to five or even more trains each way a day; the lines are mostlv made on the metre gauge, that is 3ft. 3Hin. Their roads are narrow there, and hence I presume they have adopted the narrow instead of the standard gauge, because there wonld not be room for n 4ft. 8},in. railwav on the roads ; but even those lines in a thiekly populated country and with traffie enough to take four a,nd five. tmin~ each way a day very :·ar~ly pay int~rest, and working exp~nses~that is to say, they ure hardly paymg ratlways. No doubt the md1rect uenefit and the aecommodatwn that they are to the inhabitants of the country more than justify their existence. Those railways have cost from £3,000 to £5,000 a mile

HZ

Francis Rennick, 13th August, l89fi. lOO

with labour nt half the price that it is with us. This is an clement that l'lohould not be forgotten Ly the ~ommittec in COJ1t'i1loriug the cost of our railway,; here. B1'iug mndc 011 the metre ~tango, there is very httle 1liJrcrcnce in the cost Lctwecll those railways all(l wch a,; could be provid1d ou tl1e st:1ndard gauge with the same rail. Yon have that iu my evidence Ldore the Iormcr Collanittcc, o·nd it is borne out hv estimates nutde Ly engineers all over the ;vorld, that where yon haYe to nmb~ anythi11g over a 3-ft. gaug~ railway the diiicrence between the cost of that and 4ft. 8~·in. is inconsiderable if you have the same rail. Practically all onr country lines thnt lmve been made recently and for some years are light railways, and have been made as light railways. \V c mlopte1l the 50-lb. iron rail to start with in 1870 ; unfortunately that was done on lines that might almost immediately haYc been expected to become tmuk or main lines, and that necessitated the removal ol' that rail. \Ye have hml two or three change~, fire;t putting down a 60-lo. iron rail, and then a GO-lb. steel rail, and then we have l1ad to put down 7ii-lb. steel rails ; that was a mistaken policy-the adoption of the 50-lb. iron rnils. Om lim•s aru now laid with 60-lb. steel rails, and for economy of maintenance tlmt rail is Letter and cheaper than the -10-lh. mil. Our lines made with the GO-lb. mil in many o£ the conntry districtH arc no more costly than those lines I have mentioned on the metro gauge in Europe.

16!:18. There may Le a considerable diffel'ence in the nature o£ the country '1-In Europe those rural railways arc matle on existing roads, where the formation is alreatly prepared for them. Here we have to deal with the natural country, am! make cnttingfi, banks, bridges, and culverts. Belgium is a country whor3 the lJost example of these railways is to Le fount!, am! 1 have sent to the Committee three numbers of a periotlical, the Engineer, describing nll the details o£ these Bolgimn milways, which are laid on the metre gauge mostly, and the Committee can obtain a great deal of information by the perusal of those nnmLers alJont the light rail ways of Europe. You are advised that lines ou a 2-ft. gauge with light rails can be built at one-fourth of my estimate for the 5ft. 3in. gauge in Victoria.

1GD9. I do not think that applies to the whole colony ?-~Ir. Melville said at the last sitting that they were advisctl by competent evidence tlmt they couhl have milways 011 the 2-ft. gauge at one-fifth of my estimate. This cnnnot possibly be true, except in the souAo tlmt the line can be laid on the surface of the ground without any prepnration, or on the ~ur£ace of the existing roml at i-iuch u figure, without having to Hpcutl any moucy on preparing the gradients, making cutting,-, hanks, Lriclges, and culverts ; in this sen~o it is tme, bnt in the sense of making a completed railway, snd provitling everything from the beginning, it ici a long way wide o£ the truth. I have "upplied the Committee with estimates of the cost of permanent way, 8leopcrs, and ballast, comprising a mile of milway of tlifferent classes, and I will add to that another one providing 20-lb. ~teel rails, snch as you are advised to use for 2-ft. gauge railways. These estimates Yary fi-om £1,525 a mile for a iirst-class sul.Jstantial raihn1y, laid with a 75-lb. rail and suitable sleepers and ballast, such as we 11se for our heaviest lines. The next class, laid with 60-lb. rails, come~ to £1,265. For the thinl-class railway, laid with a 40-11. rail, the ostim.ate is £9o5 ; these are the 5ft. 3in. gange. For what I call an inferior railway, the 2-rt. gauge, laid with a 40-lb. rail, the estimate for those items is £733 a mile-these figures you had from me before. I now wish to add a very inferior railway, by my definiti0n, laid on the 2-ft. gauge, with 20-lb. rails, antl with stool sleepers, such as those portable railways that you haYe been recommended to a!lopt, the items of l.Jallast, sleepers, rails, laying the line complete ln the same way as fol' tbe others, come to £641 a mile, and there is this remark at the lJOttom of the estimate. " Note.-Owiug to the higher rate for 20-I!J. railR as against 40-lb. rails, the difference between the two for ono mile of line is only £91 l8s. l1l. in favour of the 20-lb. rail." I will take an example of a tract ol' hilly country, which mm be made to produce from 5,000 to 20,000 tons o[ proflncc a year, thirty miles distant from an existing railway centre. I will give my estimate of the cost of working such traffic, including the interec;t on the outlay for a 2-ft. gauge railway nn<l a 5ft. 3in. gauge railway. The question is, what is the LeRt kind of railway for such au example as that-take the ruling grade at 1 in 30. To make a cheap railway on any gauge you must have steep grades. The relativu size of the trains would be one to four; that is, your train for the 2-ft. gauge wolllil be one-fourth of the Rize of a train for the 5ft. :3in. gauge. Yonr engines for the 2-ft. wonld be 10 tons; for the f>f"t. 3in., ;);j tons; ami the paying loads on the 1 in 30 grade on a. 2-ft. gauge mn:o;l ],c l::; tons of produce, as agaiust o\l to11s on tlto otl1cr; that is, a load which you would gut eamiug.'i from, leaving a f:tir amount of room for pas,-;cngcr and other tml'tic. The number of trains reqnirell per annum for 5,000 tolls woltld l.Je 030 each way on the narrow gn.uge, a:; against 83 each way ou the other. The Jmmber of trains required fol' 20,000 tons "'onld Le l ,320 en.ch way on a 2-ft. gauge, as against ::l30 oa tlw broad. The annual cost for working the smaller traffic of 5,000 ton:; would be, on the :2-ft. gauge, 660 traiu:i, tlmt is 330 c:H~h Ynty, because the imins that take the protluue to rhe railway centre must return, by 30 miles by :3,.;. a train mile, would amount to £2,9i0 per annum. The interest on the cost at 4 pPr ecut. on, say, £3,000 a mib for 30 mile~, or on £DO,OOO, wo11i•l be ±::3,600; together £G,570 is the annual f'ost or working expenses a1~el intc;rest. On the broad g:lllgc your working expenses would be 1G6 train,; by :lO miles by G~. a train mil,,. The trains coul1l be worked at twice the cost per train mile compared with the narrow gauge, amounLing to £1,512. The inlerest on tbe cost of construction, taking the cost at £4,000 :: mile, which I think is about a fnir comparison between the two for such country, wonlll be £,t,800, or together £(i,:ll2. Practically those anunal costs are the same; so that apart fi·om the qncstiou of break oi' gnnge ami the spco<l on the railway, it wonl1l be iudiffcrent whether you made you!' rail-;my on the 2-ft. gange or the 5ft. 3iu. gange for such a smaJI lmf'ines:; ao this. But take the next case, whd·e the produce amounts to 20,00\i tons, ancl we bnve t.lte working expenses of a 2-ft. gauge for the 20,000 ton,;, 2,640 train,; by 30 miles by 2~. 6tl. per train mile, equal to .£D,900. The intere~t on cost, the same ::1' before, -! per eent. on £90,000, i~ £:3,600, w your tot:d annual ontlay of working expenses and interest woul1l be .:fl:l,iJOO. This lmsiucss on a Dft. :lin. gauge "·onl!l Le done in GGO trains by 30 miles at 5s. a trai11 mile, Rtill t!oniJle the rate per train mile of the 2-ft. g·ang,, amounting to £J,950, interest added as before at 4 per coat. on £'1,000 a mile, :1.>1,800, altogether £9,/,)0 for the Mt. 3in. gange. The two would therefore eompare-tho ·2-ft. gauge £13,;')00, ns again:;t £9,7 50 Oil i.he broad gauge, showing that when you have a considerable bn:-iness to cope with on the railway, the qne:siiotl of gauge tells in favour of the broad gange, nnd so,. of conrsc, your rates woul1l he cnt acconli11gly. You could ehargo your farmers or the prodnrcr'' so much les:l on the stawlard gnugo tlum o;t the narrow ;:::uge allll "till make :''our railway pay. You examined l\Ir. Xonnau on the qucstiou o£ tiw '75-lL. rail, with eheck railii for ,)-clwiu curves, on the 5ft. 3in. gauge. The statement that wus sent to you from the Commissioners 1 take no responsibility

101 Francis Rennick 13th AU!,'llSt, 1895

for, and I only ind01·"e it in a qualified sense. H you have a heavy traffic J,o cope with, and you want to coll(luct that tmHie ~t a consirlcrn.hle ~JlPrd, :md you have 5-chain cm·ye~, them I say it is uciral;~c b have a 75-lb. rnil.nnt! even checked to ailow hiuh ~•Jeod ronntl w~:h cnrv0s; hnt. i( you lmve ou!v 11 smr.illmsiness to cope with, and yon want a cheap raiiwa}·, it i,; neithct· necoss:try to hn~·o 75-lb. rnil~ m'r cheeks with 5-chaiu cnrves. There is nothing to hinder yonr pnt.ting down aml working a 4.0-11. rail witb perfeet s11fety 011 such etlrvef;, the same as they baye done in America for the last 50 years. There plenty of such lines there no\L Their fir:<t line>' were construeted la.raolv with snch IiO'ht ra.i]s. and cveJJ ;)0-lb. rails. alltl worked on sharp curves withont any check rails; nu:l w'hat r.:::m be d~ne in Amcrie::1 can be doue i~ Victoria.

1700. \V!mr. wonld the l1u rouJH! those eurn:s ?-You eau rm1 nt twenty miles an ltonr with perfect safety, mHl if yo11 go down io the ~wamp yon can l:e taken roml<l the 5-chnin ean·es there with almost worn out 60-lb. iroil rail~. nml I wonhl cn~n:·e that no accident ,\·ould befall you. JYlr. \Voo.lroffe told you whcu he wa,; here o[ n r::ilwav in New York Oll whicl1 h::.d cnrves of 90 feet mr1iu~. I g:1ve von ·the :;amc inform;<t.ion i11 my fil'si uvidenet'. with the rcrnark rlmt roHIHl those cnrves tl·oy ' rhe heaviest passenger tullflc in tht~ worhl-1 snppo·''' on the ~1rmhattnn Hail way they have ti1<: he:' dcst passengvr traffic in the ,,·orltl; tl1ey work it rouml cmves of that rmlius, and they han• dont' ,;o i'or ]'ellwrs twenty ye:lr~. A~ 1111 An,c·:·ien ::nthority poinH ont ~.h(J~e enrvcs eoul<l ho increased at JJO eon;;iderrdJie outlay by pnrchasi ng c bea p property no::n 1ho ;;treet comers, bn t. they Itr"·o never eonsitlel\'d if, noce~sary to do so for the sake of ret!ueing· the wear auJ tear or lessening the eo,;t of working, so that eveu tho~e curves can be O\·err·omo on the slandard gauge, alld there \\"OUltl be no difficuli.r in this r:onnhJ ill workiug 2-ehain cnrves on Ot:l' iift. :3iu. gauge witl1 suitahle rolling-·tock. You could h:we t;;·o nxlos eouplcd with 28 tons, or l J ions oa :m nx!e, a!Hl have 6 ton~ tra<'tiYo foreo on your locomotive to \York such <'UJTes. If you wanto1l lwnxia on.!:dne< to work on them you l:oultl couple two such engim!S together, or yon eonhl haYe what they nry ni'tPH hn'e on tl;c :£-ft. gauge railways, duplex engines-that is, csc;,•Jtti:t!ly. 1wo cngil:e:< worked by r,11c ll>·ivur awl oue fireruan, w ~l1at the 8hnrp curve qnest.iun i" not nn ill~ll}1l·ml>le obstacle to the working of our strwdanl-gaHge lino~. Sow 1 cume to the existing 2··ft. g·anp;e; railway~; you have heard nbont rlw Ee,.;tiJJiog atnl the lllllinn 2-ft. gauge liue~. The Fcstiniog is tl line in Wnlos fourteen miles long to :siate quarries. It is one of the olllost railwa:rs i11 the tramway form in Great Britai;1-it is n lino made np to the The 9late traffic is all down hill ; tbo ruling gn;de is l in 80. It has cost nhont £10,()00 n is the capital cost on which they have to provide interest.

1701. By 111r. IlaJTis.-Do you know what the land cost ?-~o ; the charges nre nearly l!;d. or ].1:

10 per ton per mile. All that the engine has to do ill taking the slate:; down to the port is to Rteady the

tl'ain, and the only work for the eng-ines is to Illlll the empties back and to take ~ome passengers a!Hl goods, provisions, and so on, for the mimr8. That i;; a line made under exceptionally favorahle circnmstanees, and. the c!J:ugo~ are fully twi1~e or even three times what t.hey are for the mineral traffic 011 the British rail ways on the a vcrngc. Allol.hPr line that ha~ been liH·n tioued as a fbxorD blc speeimeH of the 2-ft. gnnge is the line that w''' mnde to the l'nri,, Exhihirion : that is abo a special JiBe ; it "'as made for six months' trrrffi1~, and uo doubt. if. \Y:Li an :Hlmirabk line for ilw pmpo:<e, hut it was 11 costly lino to make and :1. •:ostly linu to work. The charge for pa~sengcr trallie on dmt line "·as l}11l. per pa~seDger per mile. The clmrgt·s on our subnrbau line., a botH J\Id1)(lmJJc wiJ I uot avernge more tbnn ~d. That line was made with che2,p lahonr, arul waR wo.·k<~d with chc>ap labour at half the eo;;t that we are in the habit of paying for our labom' here.

1702. B!J t/,c C!tair,nrtn.---Can yon clmw a eomparison between that anll the country lineli here?­Thero i~ no analogy. That was a <:ity rai]IYaJ for an enormous traffic oH which the fares coulrl be at the minimum. Tho l[U!'stio:J of the working expenses of a railway depends ehidly upon rho cost of labour, and in Paris the lai>otlr was not mom than half of wh11t \YC have to p11y here.

1703. "\.t. that tinle it was rlon\llc wl1at it was at rmv other timo?-I do not know a.hout that. Then as reganls the lll<linn special gauge line~, they have ;bout 267 miles of 2-ft. and 2ft. 6in. gnuge mihl"ay~ in India. :tt•£1 it ha~ been "altl that tlley arc extcll(ling their lines; hut I thiuk that: is a mistake; if they oxte11tl tht'y <'Xlt'itd n•1y :;.),,wly-thcy ha1·e hrHl that mileage Jor a number of years. Those lino;; <'Ost ou th•· :t verag:P m·e1· ±::~.000 a mile, an<l arc made prineipnlly oH made road.~, roads that existed and wen• n iet\ l'orJllC<l rt·ady f'or lln· laying of the rails. The eharge~ 011 those lines to make them pay are five times fen ntHl twie<· for pu~~eJ1gers what t.he Indian charge~ are 011 the metre-gauge lines, which b rlw noxt ro t h<' ;;peeial line;;. The train load~ are about one-fourth what they are on the metre-gauge line-, aud the f'X pell>PS workiug th<ISC lines per train mile is identically the smno or within a penny or wh~.t they arc on the metre-gange line~.

l i04. By the !Ion. D. Melvillc.--Whcre did you get that information ?-From the Government returns puhli~hed in the J3lno-hook by the Pnulio Works Department of lullia-the latest, I think, is for 1893.

1705. JJy tlte Cftainnan.- Do yon remember how those lineR paid?-Yes, they paid well at those high rate". They pay() or 7 pur eent.; bnt to enable that to be done they arc obliged to charge ne11rly fi1•e times nH rmwb for gootl~ and twice as much f\H· passengers as they do on the metre-gauge lines.

1 ;o;;, By Jj,,,_ li:'Jrwitlt.- fbo,; that cover the whole of the JJ:.lrrow-o·ango system ?-That covers 2.J.9 mile~ O'Jt of :2()';', "'

1707. 'What H:crion do you refer to?-It inelntles tho Darjeeliug Raihv~y, which cost £5,000 a mile, and i~ laitl largely on the exi:;ting· road-a portion of it. was a 1ww section altogether.

1708. B11 the Cl11rinnan.-Doe~ that mcm• that they have obliterated the road a!i.ogother ?-No, it ~imply rnen11s thnt they i1ave tnken po"session of the side of the roatl to lay this 2-ft. railway, just under the Honw conditions as thermal railways am laid in Enrope. 'l'hey adapted where possible the existing roads, where 1hoy wortltl not have to pnrclu1~e ihe land or prepare the ground hy earth works, bridges, and culverts to lay the railway.

1709. )u :-)eotlami there was a road cut into a reJCk. I sec tha.t the line of railm•y is laid parallel to thb road, would that a.ako it chPnptr ilian if 1he1·c was no road there ?·-I do not think it would. The lat!'st railw~ty in Scutlnnd iH ono in t.bc I-liglllam1R, whieh has cost a very l:uge ilgnre for eon~tructioJL

17 W. By ;}fr. llm·ris.~Is it not n J\tet that people have ascended thousands of feet with narrow­gauge railway~, where there never was any road at all ?-No. It may be possible, but I have uot heard or read of any such case, and even if it were the ca~e, I do not see what it proves.

Fra.ncis Rennick, 13th August, 1895. 102

1711. You say they are all on roads ?-No, I have not said that those lines are all on roads; I say they are chiefly on roads. I have suggested to the Committee, and also to the Government, for the exten­sion of our railways here, steep grades and sharp curves to cheapen the cost, and I have mentioned that we might even adopt 1 in 20 grades. Of course, you cannot work 1 in 20 grades as cheaply as you can flatter grades, a:nd you would adopt 1 in 20 grades only where they would conduce greatly to economy of co.nstruetwn ; but where they would have such a result as that, in my opinion, for plenty of districts of V1~toria, such as the hilly or mountainous districts of Gippsland, they would be more than justified. As an mstanee-you have all heard of the Canadian Pacific Railway, and I hold that America is the best place for us to go to for the best examples of railway construction and cheap railway working; on that line they have worked ever since the line was O}Jctwd, whieh is something like eight or ten years, a grade of 1 in 22. If a grade of 1 in 22 will answer and has been economical on the Canadian Pacific Railway, I should fancy t~at a grade of 1 in 20 might suffice for such a place as "\Valhalla, Bruthcn to Omeo, Healcsville to Marys­VJlle across the Black Spur; and for such eases as that I am sure that a steep grade like that on our existing gauge would greatly tend to cheapen the construction, and it. is for snch cases that I would recom­mend it; there might even be cases where you would be justified in making a steeper grade than that. But then you would want to introduce the rack rail or rope haulage, or a centre rail with the Fell system, or some other system to surmount the steeper grade, as they have in hundreds of places in the world, and those things would be perfectly justified. As regards railways that do not pay, you should be very careful in advising the construction of a railway to ascertain that the loss would not be very great in working the traffic, and no railway should be recommended that has not a prospect of paying immediately or within a fair time, either directly by itself or in the traffic that it would add to oue of our trunk lines, because I hold that you have to take both those elements into consideration in advising a new railway. I do not know that there is anything that has ever been invented that condnces more to the development of the country and the increase of wealth than railways. It has been over and over again pointed out by experts who have gone into the statistics nnd economics of railway traffic that for every £1 you spend on a new railway prudently you increase the wealth of the country by fnlly £.5. If we apply that formula to the Victorian railways it leads to this result. We have spent :tbont £37,000,000, and five times that would be £185,000,000. The wealth of Victoria hns been computed by Hayter, a very good nuthority, at £400,000,000 sterling or more, so that out of the whole of the wealth of this country there is due to the development by the railways £18.5,000,000, or all the wealth except about £215,000,000. That may be a little exaggeration, but I do not think it is very fat• from the mark. ·what would Victoria be without her railways, and these are considerations apart altogether from the mere paying aspect of railways that have to be taken into account in advising and recommending railwnys in a new ceuntry. I am not opposed to innovation or even radical changes in milway construction and working ; all I want is a good souml reason for whatever you propose to do. If you advise a break of gauge, then you should have geod reason for your advice. There is this consideration in a break of gauge, that no matter what you can carry or at what rate you cnn carry on the branch railways, where you have a break of ga.uge the pro­ducer and the trader have to meet the cost of the break of gauge at the junction, and the transhipment from one train to another has always been reckoned by authorities who have had to deal with breaks of gauge at from 20 to 30 miles; but if you take it at 10 miles, then all your producers who use those branch railways have, in addition to the mileage rate at which these things can be worked, to pay an extra mileage of from 10 to 30 miles for the break of gauge. A man 20 or 30 miles away from a present railway centre has to face having an addition to the charges of 10 or 20 miles extra haulage.

1712. By the Ghairman.~How do you make that out ?-There is the transhipment of the goods; there is the detention at the junctions ; there is extra rolling-stock duo to those detentions ; there is the waste, and the loss, and the damage in changing the stuff from one train to another, and the general nuisance and inconvenience of the thing. I should recommend the Committee to get evidence from South Australia, where they have a break of gauge, as to the inconvenience and cost of the break of gauge. As far as my reading goes, and as far as my knowledge goes, for every break of gauge that you introduce you have to face those extra charges and inconveniences.

1713. By the Hon. D. MehJille.~Have you heard of Sir Alexander Rendel, Consulting Engineer to the Government ef India ?-Yes.

1714. Are you aware of what his opinions are as to narrow gauge?-Yes. 1715. He says-

It was long held, and may be so now, by 110me Indian authorities that whatever was saved in first cost by the use of the metre gauge was lost in its heavier working expenses. This is now proved to be mistaken by the results of the Hajputana-Malwa. This line is worked by the broad gauge Bombay Railway, a linr! which is dead flat from end to end, which enjoys cheaper fuel, and which has more than twice the traffic of the Rajputana, nevertheless the cost of transport per ton, and pet passenger mile, are materially higher on the BombRy-Baroda than they are on the Rajputana, vide the figures given under letters (i) and (l) in the statistical tables.

The passenger train loads of the metre-gauge line are also larger than thoBe of the broad-!!auge line. The narrow­gauge passenger loads in India are in fact generally higher than those of the broad-gauge lines. The goods train loads are only half those of the broad-gauge line. But the Bombay-Baroda is practically the terminal length of the Rajputana, and has therefore particular facilities in this respect. The Hajputana loads are a long way above those of the broad-gauge Great Indian Peninsula Railway in passenger, and equal in goods trains.

I do not say that the results of the 2ft. 6in. gauge would be equal to those of the metre, but I do say that, if the metre gauge is so great a success, the 2ft. 6in. gauge cannot well be a failure.

He says further on-.Although I have no accurate information as to the amount of traffic which a 2ft. 6in. gauge will carry, I may say of

the metre (3 · 3f) gauge that its capacity has proved to be far greater than was at one time expected of it ; and the difference between a metre and a 2ft. 6in. gauge is not so great that a statement of the latest results of the metre gauge would be useless in considering the prospects of the smaller gauge.

I will take the Rajputana-Malwa as the most developed of the Indian metre-gauge lines, though I might take with almost equal results the metre-gange Bengal and North-Western. The Rajputana-Malwa systt:.-'!11 comprises 1,674 miles of metre-gauge railway, and the results of the first balf of the year (1894) are as follows:-

(a) Cost per mile (/i) Earning per mile for the half-year, passenger Rs. 2,153 + goods Rs. 6,288 + sundries

Hs. 127 ... Total (c) Working expenses per mile for the half-year (d) Profit per mile for the half-year, just 7 por cent., or at the rate of over 14 per cent. per annum (e) Average earnings per passenger train mile (j) .Average earnings per goods train mile ...

Rs. i8,480

Rs. 8,568 Rs. 2,941

Rs. 3-37 Rs.3·70

103 Fra.ncis Rennick, 13th Augus", 1895.

(g) Average working expenses per train mile, taking cost of gooltS and passenger trains per mile as the same

(lt) Average receipts per passenger (all classes) per mile .. ; ... . ·:· . . (i) Averarre cost of carrvince a passenger (all classes) per mtle on basts of dtv1s10n of expenses

b;tween passenger a~1d goods adopted by Government of India NorlJ.-Corresponding cost on broad gauge, Bombay and Baroda, was

(k) Average receipt per ton of goods of !tll kinds per mile .•• ... ... ... ... (I) Average cost of carrying I ton of goods one mile on basis of division of expenses between

passenger and goods adopted by Government of India NorE.-Corresponding cost on broad gauge, Bombay and Baroda, was .. . . ..

Rs. 1•26 Pies 2·12

Pies ·51 Pies ·78

Pies 6•19

Pies 2·32 Pies 2•41

(m) Average load of passenger trains, treating all items of coaching traffic other than passengers as passengers at average passenger rate ... .. . ... ... Persons 305

(n) Avera"e load of all classes of goods trains, taking empty as well as full trains 114·89 tons (o) Average number of tons of goods which passed over each mile of line daily during the half-

1,078 tons year ... ... ... ... ... ... . .. (p) Average number of passengers who passed over each mile of line daily during the half-year,

curiously enough, the same 1,078 pass. (q) Average train mileage, passenger and goods, per mile per diem over whole system ... . ... 12·8 .

i.e., the average traffic of the line was carried by the equiYal~nt of 6'4 passenger and goods trams passmg over each rmle of the whole line each way daily, of which I •75 were passenger trains, !tUd 4•65 goods trai~1S. .

The minimum traffic over the busiest section of the line would, I have reason to belleve, be more than 50 per cent. m excess of the average above given, and the rn:tximum traffic on the busiest section wouli! be certainly double the average ..

Tho line was lai<l originally with 40-lb. iron rails, whieh are now being gradually changerl for 50-lb. steel. It 1s entirely a single line, and long grades of I in !50 are found on the busiest section, and there are five miles of l in 40 on one important section, and curves of 600 feet ra.tlius are, I believe, frequent, specially on the inelines. The heaviest goods engine, known as F's, are six-wheeled, all couplcu, weighing in running order from 21 to 23~ tons. The heaviest passenger engines, known as O's, are eight·wheeled, four coupled, four on a bogie in front, aud weighing about 24~ tons.

Then there is this (quoting from :Mr. :Everanl Calthrop, late Assi~tant Locomotive Superintendent, Great Indian Peninsula Rr~ilway) :-

GENERAL RESUI.TS OF THE vVOnKING OF ALI, RAILWAYS IN INDIA FOlt THE YEAR ENDING

31ST DECEMBER, 1892.

Particutars.

Average cost per mile open Hs. i

Standard Gauge, 5ft. 6in.

162,893

Metre Gaut:e, 3ft. 3j!in.

71,119

Special Gauges, 2ft. 6in. and 2ft.

31,252

So that I was ahsolutely right in saying it was only one-fifth. lie gives the average cost per mile of single tr:1.ek, inelndiug sidings, at Rs. 125,25(), Rs. 6;),256, and Rs. 29,670 respectively, and the percentage of net profits on total capital outlay on open line (including suspense) as .) · 4 7 per cc ut., 5 • 26 per cent., and 6 · 92 per cont. He wiudR up by saying-

The traffic to be carried in any district through which it is contemplated to build a new r:~.ihmy is the same whether the propose<! line is carried out on the 2ft. 6in. or the standard gauge; but it is shown that the amount of cRpital over which profits are to he spread may make a.ll the difference between bankruptcy and perdition on the one hand aud success and dgorous development on the other.

These arc the Indian railways that yon have quoted to ns. To insist, regardless alike of natural conditions and of the aspect of any project fro01 the commercial and business

standpoint, as many would-be advisers of the Gm-ernment of India do, that every line in India should be built on the standard gauge, or not at all, is prm•ed to be mere madness.

That is what the greatest authority on railways in India says ?-I do not think he is the greatest authority. 1 may say that when the Indian authorities contemplated a break of gauge Sir Alexander Rondel, with other :EngJi~h expertA, recommended the 2ft. 9in. gauge ; he was then Rt.rongly in favonr of the very narrow gauge, and, apparently, he i~ so ~rill. Against his jndgment the Indian authorities adopted a greater width, cbiefly, I think, on the advice of Sir ,Jolm Fowler, so that Sir Alexander Rendel is not altogether a diHintorost.ed witness-he iH simply stieking to his own opinion. As regards the profit, he shows that the 2ft. and 2ft. 6in. gauge railways in India pay 6 and 7 per cent., but I have sho1vn you that they do that by charging five times as much for goods, and twice as much for passengers, as the metre-gauge line.

1716. The fares there are qd. per mile first class, ·56d. second class, and ·28d. for third cla.ss; that is on the Morvi Railway, which is a 2ft. 6in. gauge. The length of the line is 93,t miles; the cost was £2,124 per mile, or a total of.£ 199,691-i. The main line was completed in 1R87 and the branch in 1890, so that your information is altogether antiquated ?-1\'Iy information i~ the very latest.

1717. The goods charges m·e as follows :-"Goods (1st class), Hl2d. per ton per mile ; goods (2nd class), I·.53d. per ton per mile ; goods (ard class), 2·04a. per ton per mile ; goods (4th clr~ss ), 2·55(1. per ton per mile; goods (5th clasr:), ;)'(lid. per ton per mile; foot! grains, 0·89d. per ton per mile" ?-Iu this country those faro" for freight aml passeugers wonld he reckoued very cheap, but if you look at the Indian quotations for the metre-gauge line you \Yill find that they are only one-fourth or ono-fifth of those for freight and only one-half for passougers.

1718. The description goes on-" The line is laid on the existing bridged and metalled road for the first 24 miles, and then runs along the lino of unhridgod and nnmetalled road to :Morvi. The permanent way consists of steel raib,, 19 Ius. and 29 lbs., laid partly on patent steel and partly on timber sleepers. On the Riijkot branch l~l-lb. rails on light sleeper~ arc used, except on the curves, which have been laid with 29-lb. rails, and sleepers partly patent and partly timber. Hevenuo, 2,95,227 rupees. Total working expenses, 2,07,677 rupees. vVorking expenses per train mile, 1·61 rupees, or 70·34 per cent. of revenue" ? -I have all that information, anrll still Ray that the fares are four of five times as much for freiaht and twice as much for passengers as they arc on the metro-gauge line. N o,v, as to tlw cost of constructior;, the standard Indian railways were the firM tluH \Yere made. Thev were g-en('rnllv OYC'r \lilrienlt ('mmtn· and larae treacherous rivers, and tlwy were made in Pxtrav>~g->tllt tinw·s whr:u eitginet;r~ 'tnd otl10rs had ve;:Y extra;~­gant notions, allll those ntilwa;-s were lntilt as our enrly mihvay,; were !milt. here. Th,•y cr,slc ar; enonuous amount, hut they were built for an enormous traffic, and there is no comparison between their cost and the

Franc is Rennick, 18th August, 1895 104

cost of railwa~s here, either on the broad gauge or the uarrow gauge, which are built for the limited traffic. I may .here ~rmg under your notice extracts from the Government Hcport on the Indian railways in which t}1ey g1,;e e.stnnates for the standard gauge, the metre and the 2ft. 6in. gauge, in the specific case of 11

hne 16;f miles long. On the standard gauge with heavy plcnty;of ballast, large sleepers, large equip-ment anll everything in I he nsnal ;;tyle of the standard gauge in India, the estimated cost of the Public VV: orks. Department was Rs. 796,147; on the metre gauge ·with the usual metre-gauge stamlard of lighter ra1ls, hghter bal!a:st, lighter olcepers, a1al lighter equipment in every way, the estimate was Rs. 534,282 ; and for the 2ft. 6[u. gauge \rilh t1 lighter standard still, and a line constructed for a lighter traffic, it was Rs. 49±,:345. That is 11 recent Indian estinmte for a line constructed on either of the three gauges, and it does not bc~tr out vom statement that a 2[t ()iu. Ene can be constructed in India for ono-fourth or one-fifth of the eoHt of the ~tandanl gauge. As to the comparati vc cost of railways, we ha;e in this country lines that have cost £12.5,(}00 nr £130,G\JO a mile, ttud we arc now constructing lines on the same at £.1,500 a mile. Tli:ct 11imply shows tltnt the original were con~trneted at a different time with dtfferent ideas to meet clifferent clrcnmstanceB. Those lines were suburban lines, whereas the low-priced lines are country ones, aml materials and labour co:;;t very much lc10s than when the high-priced lines wore made. That pron;, nothing.

. 1719. By J}Ir. Cra1•cn.-Did yott read Mr. Van de Veldc's evidence ?-Yes, and I have commented on Ins Paris example, his F estiniog example, and his Indian example.

1720. Assuming in making an excavation for a narrow gauge it eosts a ce1tain amount per cubic yard to excavate, if yoa in the same place make a wider cutting for n broad gauge, by reason of the extra quantity of stuff to be excavated, woald not t,he extra amount be taken out at very much less per yard in proportion ?-Yes. The earthworks in a broad-gau~e line can be executed at a less price per yard than on a narrow-gauge, becauso there arc greater facilities for doing the work.

1721. The prices that we have had given ns as t,he cost of the variou& railways include compensation for land h•:a vy compeu~ation in many ca~cs, and a much heavier rate for labour and materials than we pay now.

1722. Do you approYc of the policy of opening up those outlying portions of the colony by means of horse by the .State, lmt the res1Jonsibility resting npon the shoulders of some local body?­Yes. If you have to make a departure in railway construction, and adopt a break of gauge, m· if it is advisable to construct trannntys instead of railways, my opinion is that it should be left to the local bodies to do so, subsidized by the Government.

.1723. By the Chairmcvn.- Would they be under Government supervision ?-Somebody would have to certify as to the cost al!(l the quality of the work.

1724. You arc aware there wonhlncver be many miles constructed under that system ?-It is a IJ?atter to be thresbecl ont by the powers that be, and the wisdom of Parliament, as to whether you should give the local bodies a free hand, and let them do they work at any cost they like. If they are to be subsidized, unles~ you have a coud[tion that the money is to be prudently and properly spent, I have no doubt whatever that there would he misexpcnditure and gross extravagance.

1725. The local bodies make £100 go three times as far as the Government can ?-That has been stated. I that the local bodies !tre a very useful institution, and should be encouraged to do more work than tlwy are <loing.

1726. By Jfr. Harris.-You have read the Tramways Act of 1890, with regard to the tramways, and mode of borrowing money ?-I do not know that 1 am well up in it. I have heard of the Ad.

1727. You do not kuow the principal reasons of failure in the administration of that Act except that the local boLlios were not desirous of taking advantage of it, because it would not pay them.

1728. I am informeu that some of the local bodies were desirous of taking advanta~e of it, hut the plans aud ;:;peclficatious for constructing the railways were so high that they had to give it up in despair? -I do not know that of my own knowlmlge.

1729. These are extracts from information supplied to the ::\:linister of Railways by :Mr. W. H. Robarts, of India : have you re~td them ?-Have they been published in the newspaperl:l. If there have been extracts published in the newspaper:>, it is very likely I have read them. Nothing in railway construction that is puhli,hed escapes my attention.

1730. It says-

1. Re STATE~IENT THAT RAILWAYS OF NARROW GAUGE WERE HE1i'i'G TAKEN UP.

During the course of the recent inquiry in New South Wales into the question of the advisability of introducing narrow-gauge into Australia, it was alleg·ed, or rather, by the examining Committee that in numerous instances rmrrow-gauge line~ laid down in India were lJeing np ago.in and replaced by lines of a broader gauge. On the witnesses, who were under examination, being questioned on this point, they were unabl(e to give any reasons for a course of proceJure on the pare of the railway authorities in India, Their reticence on this point may be well understood, the Bimple reason being that the allegation is without any really substantial foundation, In fact, for the natural inference of tbe AUf!gestion is that narrow-gauge rail ways have proved to be a failure in India, whereas the exact reverse is the aetual state ot affairs al the prc,ent time. Ouly quite recently the Government of India have issued a publication stating the terms on which they are prcparcLl to consider offers for the construction of branch lines or extension lines of existing railways. This has been welcomed by all mercantile and commercial men, as well as the agricul­turists in fact, by every one interested in railway communication in India, as a meanE of supplying thr much needed feeders the main lines.

* * * * * * * 2. LINES ~IADE FOR THE TRAFFIC.

• * * * • * * * The reason for this anomalous state of things is that, as in A.ustralia, there are vel'y few districts in India capable of

producing sufficient traffic within th~eir loc:.l area to cover the equipment and maintenance charges, and give profits on the large capital for the construction of a broad-gauge line. This being the position of affairs, it has been satis­factorily shown that can ha only effectually dealt with, in the true interests of the State and the producers, by the intro-(luction of the narrowest gauge that c:m he practic:tlly wol'ked as a And, in arriving at this conclusion, the great financial succeAs of narrow-gauge lines in Sweden, Denmark, :md well as in India, was tltken into account.

In pursuance of tbis mTaugcmeHt it was decided t,o t,ap rich of Chuteesgnrh, iu the central provinces, and a 2ft. 6in. railway, about lGO miles in lenp;th, wns a feeder from the U.I.l'. Terminus at .\'agpore to Hyepore, tra1ling eii.y of the Clmtc·(·sgnrh Plateau, in order that tlle immense supplies of grain produced there might be conveyed to other parts of India. Prim· to the advent of this milway, wheat was selling in Hyepore at as low a price as 50 seers or 100 lbs. per rupee, simply because, from the district beiug so isolated, grain could not Le exported;

105 Fraoois Rennick, 13th August, 1895.

and after the op3ning of this n:trr<Hv-g:;ugD mil\v,ty t 11e price of w-heat ras~ to 21) 'eer3 or 40 lb3, per rupee, which is now the current rate for that commodity. Tb.e railway dirl all that was required of it and every one benefited; the producers by the increased prices, the Government, 1ts the supreme landlord, by the enhanced rents they were able to impose, and the general trade of the country by the iucr~ascd demand for general merchandise owing to the opening up of the isolated tracts and the greater spending power of the prouucers.

* * ·X. * ·)!. •*

3. TRAXSUIPMEX:r. As against the institution of feeder lines of narrow l{auge there is only one disadvantage which e"n !Je quoted with

any show of reason, aud that i!1 the obligation of havin!.( to break hulk at the junctions with the main line. Formerly the arrangements for this purpose were of t1 crude aml inadequate naJ.nre, :wd there were munerous complaints of detention and damage. Of late years, however. so many improwmeuts have been effecte1l tlml. train loads of heavy and bulky goods can be transhipped wit.h an ease and cleverness which insures safety awl dcsrntch, and this has so eompletely minimised the delays and the risk of injury that this argument t>o longer has tlte force it use<l to oiJtain. Tlte aclvautn~·es which accrue from thl! use of a light fee,[cr line of narrow gauge arc"" lllan,Y and so obvious that they far outweigh the incon­venience of transhipping goods, The gn:nt prhv!iple which is dc~:t with in deciding the question or gnnge is that any machine is, comparatively speaking, c:conomicnl only when it is working at fall power. The hest gunge for this pmpose is undoubtedly that of 2ft. 6in., which, as practical railwn,l·, pos,;csscs carrying per cent. of capital cost than that of any other ga.uge. The most important fcatnr<!S regarding are its attendant tiw fact that it can he laid on ordinary cart roads and without detriment to the existing locnl traffic, and inhcrant adaptability of alignment when any natural obstacles to he surmounted. As n instance, OJlC of the best paying railways iu India is the Darjeeling-Himalayan line, which lws to ascend some 7,000 feet, whcnl P'Lssengers in the fore pnrt of the train can sometimes see the lamps of the break-v~n in the rear of the t.min.

* * * •

4. CoMrAnA:rrvE CosT. For ordinary cross-country pmposcs, the cost of both the broad and the metre gange is deterrent. The broad-gauge

lines dnring recent years haYc ntrie<1 from £.5,000 to £10,000 per mile. Tlw nwtre-gauge lines aYemg-e from £1,500 to £2,000 per mile less than those smns; short lengths of these lnYC, it is trne, cost proportionately less, !Jut that i.'< only where there are exception11l circmnstauc•Js, such ~" no heavy cnttings or expensive bridges. "\s a contrast to these heavy items, some of the narrow-gauge Inilirrn lines he quoted; the Pow,.yan t:iteam Trn->nway in the N O''th- \\'est Provinces, where a line 31 miles cost less than £1,000 :t ; in tbis instnnce it was f:mnd feasible to talcc ndvantagc of cross-country roaus. The .:\IoTI'i State Rnilway, 6~ miles lon;r, eost about £1,100 per mile; this was laid pr:rtly on the existing ro11ds that ran across conntry :mu required new earth works, &c., f•w some miles. The ~ft .. 6in. liue made aml cqui;Jped for His Highness the ,Jincowar of Haroda, was constructed across new country n.t a cost of £1,600 per mile, and ihe Kaunia­Dharlla line cost still less. Of course, these amounts are E'Xclusire of rolling-st•>Ck. It may be taken as an established fact that with the same outlay, and cxclnsil·e of rollinl-(-stock, ·IOO 10 500 1nilcs oE feeder lines can be laid on the ~ft,- Gin. gauge for every 11)0 mile8 of branch line constructed on the broad gangc, The minimum radius of the narrow gauge on open line enables it to wind iunnd out and follow nil the convolutions of nu ordinary road, whereas in the case of the hroad gauge the stiffness of its alignment obliges it to through valuable property and undertake expensive brillges, embankments, and euttings, the bulk o£ which arc of being avoiclecl by the more flexible alignment of the snu1ller gauge.

5, XArtROW-GAUGE nxGIXES WILL lLHJL ~lOUE.

Anu it is, furthermore, a well-known fact mnougst locomotive cngin<'ers that on the same grauicuts a narrow-gauge locomotive, in proportion to it-s weight, can haul a !argt:r !mu! than a broari-gaug-e locomotive. On curved gradients, this auvantage is stiU more apparent, anu tliis is slwwn in the Dt<rjccling-Hinmlayan Hail way alrettdy mentioned?-

I have seen all that before. I Jo 11ot agree with two-thirds oi iL As reg:uds the cost o£ construction of Imlian railways where it is allege;! that ou an existiug road a lino has been HHnle at £I,OCO a mile, uo doubt that is true, and in goi11g through cn~y eonnlt·y they c:1n be made Yery cheap. 'Vc make them here at £1,000 :t mile, across the natural country, with heavier rails and more ballast th:m those lines ; but, in India labour is not one-third or one-fourth what it is here. Yon pay men there 10.'. a month; here you have to pay a mau £8 or £9 :: mo:1~h. Reckoning tlwt the Inllian coolie mea du one-third of the work of a Emopean, still you hnve yom lahom at erne-third of the co,;t, !ten eo it is >hat you caa nwke snch cheap railways in a country like Iuclia, an,J Emopean countries \Yhere you get the labour o~:e-half what we have to {Hty here. Thlln, n:' to the coBvei'Sion of gauges in India, there i;; very lit.tlo (~onversion of the 2-fL or 2l't .. Gitl. gauge to middle or iilandanl gange, beetHt:>e t1wy lwn• only ahouL ;;uo miiu,; nf this raih1"ay out of 18,000 miles. There cannot be nmdt necessity for connTtiug tho~o. have converted the J-ft. gauge to the "rambrd g;1nge, at:.d they have couverteJ a nnmhor of metre-gauge lines that were found to be insuliicient for the lmsiness of ttw •listrict to the ~tauuanl gauge. On the other ham!, to produce uniformity ami to confine their gangos within certain limits for cheap working, tlwy have couverted the 5ft. 6in. railway to the metre gunge. ThoHl aro all done as matters of policy, either from the narrow to the bron,d to meet the greater traffic, or from the broad to the narrow to facilitate working, and to ccntmlize the gn.ngos. Cnnmh is an instance that has been refef!'cd t:J since I have been in the room. In Canada they went in and had a very brul attack of nmTow-p;auge railway construction in I ~70 or lR72, anll they lllctde a good many lines on tl1e 3i'c_ 6in. g.tny;o. Those lines lmvo all heen convertor[ to the standard gange, except in Prince Edwanl I< 1and, when· they have the :3rt. (iin. gauge-~that iH an isolated system.

1731. If the circumstances warrant, t.lw Lroail gnnge is the ;;taiHlnnl ? -That i:; a 'luc~tion that has to be dealt with at the initiation of a raihYay, and in '~ 11ich authorities may •liffcc-. Yon may think that to meet the present c·ircnmstnnce,; a narrow-gauge line will be ample, am! will be so for twenty years. I might think the reverse, and that to meet the growth of snch a llistriet in ten years' time the narrow grwge will be Ionwl to be insutfieieat, and yo:t woulll have to change it. Theoe questions apply to special circumstances which have to be dealt with on tl:eir merit;;. In C:lllatla, I may also point to the Government lines as being worked, not as a ~yc;tem of paying railways, bnt as a system to develop the country, and increnAe the wealth and prosperity of the inhabitants, and the Government railways of Cmmda are worked simply to meet t.hc working cxpcH5c'"· A;; a n•ntter of natiounl policy they do that, awl on nil the Canadian Gorcrnment railw,tys·-what :trc ealbd the intereolonial railways-tlwy il:~ve e!trl'ied grain (and I do not know that they do not do it now under that sy~tl'rn of simply making the liuo pay working expenses, and not expecting anything to\vanls at one-eighth or one-tenth of n penny per ton per mile where it competcR with wal.er carriage. Thu haYc 1lone that for yem·s and years on the stawlnnl gauge, nor on the murow gauge. They could not. t!o it on the n~nuw gauge awl meet tlw working expenses. That shows wlmt can 0{; done hy n prOJY~rly consl:·nclcd .'yst.om o[ stanut~.rcl r.<tilway,;. In th<~ United State~ they have eon\Terted most o[ their mtn'OI\·-gange liue,; to tlw stambrd g·auge, nml instea<l of narrow-gunge railways extending there 1hey are n;:ln~ud cnH'Y .rear. I take it that Amcr.ica is tlw hest example for the Anntmlian colonic,, to take, hnth in rail co11stmetion aml working. It is a new eountry, and it has had to go through all the plm~e,; that we shal I to go through. Tho "Yaukecs" arc an ingenious and

Francis Rennick, 13th August, 1895. 106

progressive people, and I think if we follow their example, in preference to going to Europe or India or any of the old-country places, we shall be on the right track, at any rate in their railway construction and working. If we can secure the results that they have secured in giving cheap freights and cheap traffic we will accomplish the most good for the country.

The rvitness withdrew.

\Villiam C. Kernot, sworn and examined.

1732. By the Chairman.-What are you ?-Professor of Enginering, Melbourne University. I put a put n few notes together roughly last night, which I will read.-[ The witness read the following pape1·:-J

RAILWAYS.-BROAD v. NARROW GAUGE.

The following conclusions are based primarily upon certain experiences that took place about 25 years ago, when a vigorous effort was made to persuade the Victorian Government to introduce the 3ft. 6in. gauge, together with the Fairlie locomotive, in lieu of the present gauge. Drawings were prepared for both gauges, for a hilly and a level country respectively. I took part in making those drawings, and consider that they were fairly made, if anything Rlightly favour· ing the narrow gauge. Tenders were calied for, and it was found that the difference in cost was so trifling (being, I think, under 5 per cent.) that all thought of introducing the :lft. 6in. was abandoned. Since then I have taken a general interest in the question, have twice visited England, France and Italy, and once North America, Queensland, ~tnd New Zealand, and h~tve several times been in Tasmania and South Australia, on each occasion taking special notice of railway matters for the purpose of my lectures at the University. My conclusions are as follow:-

!. That the Irish gauge of 5ft. :lin., as used in Victoria, South Australia, and Ireland, and the English gauge of 4ft. S!in., as used in New South \Vales, Europe, and North America, are of very nearly equal merit. My own predilection is slightly in favour of the Irish, or 5ft. 3in.

2. That the economy so often and so loudly claimed for gauges of 3ft. to 3ft. 6in. is much smaller than usually represented, and is accompanied by several serious disadvantages.

Under the heading of construction, the cost of land, fencing, gates, and drains will be independent of gauge. The cost of earthwork very nearly so. For example, in a cutting 10 feet deep, with l to I slopes, and an 18-ft. formation for broad and 16 for narrow gauge, the difference is only as 280 to 260, or 7 per cent.; and against this must be set the extra difficulty of handling contractors' waggons on the narrower formation. In embankment the difference is slighter still. A culvert under a bank I 0 feet high will be 48 feet long for the broad and 46 for the narrow gauge. The fronts, which are the complex aud expensive parts, will be the same in each case, and the 2 feet of barrel saved will not represent 2 per cent. of the total cost. Bridges must be adapted to the stream~ they cross. Piers strong enough to resist floods must be provided in each case, and 2 feet more or less width will hardly affect the cost appreciably_ Same remark will apply to tunnels-the size of which is regulated by the dimensions of the carriages and not by the gauge.

For equal loads per wheel and speeds the narrow gauge will need us much ballast, as large sleepers, and heavier rails than the broad. The heavier rails are needed because the weight is not so well divided, and the lateral oscillation and effect of wind greater the narrower the gauge. Further, as the effect of inequality of level of the two rails is greater on the narrower gauges than on the broad, more careful maintenance will be required.

The economy in construction actually realized in the case of many narrow-gauge lines is due, not so much to the gauge as to the adoption of lighter engines, carriages, and permanent way, and lower speeds. All this, however, could be done on the wider gauges, and, if done, would merely be a return to earlier types. J<;ngines of 10 to 12 tons weight were at one time general on the J~nglish gauge; now engines of three times that weight are sometimes seen on :Jft. 6in.; and so with carriages and waggons. Further, advocates of tl,e narrow gauge are fond of vaunting the low cost per mile of their lines, but do not direct attention to the fact that this, in difficult country, is due to the adoption of circuitous routes avoiding difficulties, but leading to a great increase in the number of miles of line between two giv0n termini.

As regards working, it must be remembered that expenses of administration will be unaffected by gauge, and that, further, if, as is usual, lighter and smaller stock is used on the narrow gauge, that a much larger number must be provided, which will add to the expense of handling a given traffic. A small engine, let us say, will pull two. thirds the load of a large one at two-thirds the speed ; it will therefore take nine small eugines to do the work of four large ones, and each of these nine engines will each need as many men to attend to them, and, owing to the lower efficiency of small machines compared with large, will, in the aggr0gate, burn more coal and cost more in repairs t.han the four large ones ; and similarly for carriages and waggons.

I should like to lay particular stress upon this, because a member of the Committee rend out from a document certain stntements, and all I can say is, that either I am mad or that writer is mad. From all my knowledge, the statement that narrow-gauge engines would pull greater loads in proportion to their weight than broad­gauge engines is absolutely incorrect. I have not seen the Darjeeling and Himalaya Hailway, but I sny that statement is about as incredible a statement as nny I have met with for a long time. The larger the machine the more perfect it is, whether it is a loeomotive or a steamer. The lnrge ones always work better than the small ones, and give better results for the fuel they burn.

1733. Why not extend the gauge and make it 7 feet ?-Thnt question is not before the Committee. It is a question of reducing the gauge.

1734. If 5ft. 3in. is better than 3ft. 6in., why not extend the gauge still further ?-It is a question of traffic.

1735. By Jllr. Harris.-Tbis paper Rays-

The fact that narrow-gauge locomotives are usually required to surmount much steeper gradients than are generally to be found on standard railways makes adhesion a question of the first importance. lt is very generally supposed that the coefficient of adhesion between a wheel and a rail is a constant fraction of insistent weight, varying slightly with the molecular structure of the metals in contact. There is, however, reason to believe that it decreases considerably with an increase of weight. In locomotives of the standard gauge, with from 12 to 16 tons per driven axle, it is generally held that a coefficient of adhesion of one-sixth is all that can be counted on with certainty. From a number of experiments on the Festiniog Hail way, with the results of which the late Mr. Spooner, who himself supported the theory, was good enough to supply me, I found that the load there per driven axle was 5 tons, the coefficient averaging about one-fifth. Again, with my small engines that have a load on the driven axles of about 1·2 tons, the calculated coefficient is one-fourth, in support of which I give the foilowing experiment, eondnctcd in the presence of two gentlemen belonging to a firm of locomotive builders, when it was unJer consideration to build for military purposes some engines on the plan of the second described in Section IlL?-

That is only one of the extraordinary statements I am continually bearing, and a good many which I hnve succeeded in bowling out. I might point out at this stage that there are two things we should not confuse, that is, the actual tractive force of an engine of given weight and the efficiency of that engine as a machine for producing power from coal. As far as actual tractive force is concerned, I do not see what gauge has to do with it or what it has to do with the weight per wheel. vVe have heard of 5 tons axle load on the narrow gauge ; perhaps the Committee may be aware that Stephcnson's " Hocket," which was the first locomotive of the modern type, haJ only 2 tons weight per axle on the broad gauge. You may have light or heavy engines of any gauge.

107 William C. Kernot, 13th August, 1895.

1736. By the Chairman.-A light engine, if it would do the work, would be cheaper and would require less fuel ?-A light engine could not do the work.

1737. Where there is plenty of work for the broad gauge and heavy engine we do not want the narrow gauge; the narrow gauge is for the light traffic ?-It is quite possible that where the traffic is light it is desirable to have a narrow gauge, hut 1 :my it is possible to use light engines on the broad gauge, and they were used on the broad 40 or 50 years ago, and were replaced a.-; the traffic warranted ; the gauge has nothing to do with

1738. You will not contend that every engineer in the worl(l will agree with you ?-Not for a moment, nor do I agree with a good many of them. Small machines must be more costly and less efficient than large ones, provit!cd there is work enough fnr large ones to do. If there is not work to do there is no use putting a railway train to do the work of a wheelbarrow. Further, I would point out that with narrow carriages or waggons, trnins of cqnal capacity will need longer platforms and more extensive accommodation on the narrow gauge than on the broad. If you take a New Zealand train, there are tilrce people in the width of the tra(n, bnt you will find that in ours there are four or t!ve people in the width of the train, therefore the .New Zealand system will require so much longer platform; and the same will apply to goods. Ono strong argument of the narrow-gauge advocates is that narrow lines permit :;harp curves, thus :oaving earthwork in mountainous country. In reply to this I would say that very worked with success on the broad gauge. On the Blue iYionntains railway, in Kew South curves of 8 chains are traversed at fairly high spcede, while on the eleYatod railways of New York and the Sydney tramways (which really are railways worked with locomotives) rolling-stock of ordinary tion traverses curves far sharper than commonly found on the 3ft. Gin. railways. At New York the gauge is 4ft. 8tin., and cnrves of 100 feet radius are continually traversed by ordinary locomotives and cars at a low speed. 1 have been in New York myself and have travelled over them. The trains are passing over those curves all day long. Considering the importance of the line, the Third Avenue line more particularly, and the enormous amount of traffic on the cost of easing those curves would be nothing, and the fact that they do not do it is a strong argument that there is no need for it. The Third Avenue is comparable to our tram line coming up Swanston-strcet, turning up Lonsdalo-street, and then turning ngain into Russell-streot. The curves are sharper than ours. 1 believe Mr. Rennick takes hi~ curves of 90 feet from 'Vellington's book. There they m·c working all day long without any trouble. There is n notice up­" Speed 6 miles an hour," and that notice seems to be adhered to. From the curve point of view l shouhl deprecate the introduction of a narrow gauge until I was sure the possibilities of the wider gauge from a curve point of view had been exhausted. My conclusion, then, is that the economy claimed for the narrow gauge is really due to light construction, and that light construction is equally possible on our present gauge, and that while a light railway can he gradually changed into a heaYy one if the traffic justifies it, to change a narrow gauge to a broad is an infinitely more difficult matter. .Fmthor, I would point out that, owing to the continued reduction in cost of materials and labour, a substantial broad-gauge railway can now be made for a cost that twenty ycarR ago would have been regarded as incredible; our recent 1\ifallee lines having cost f\Jr rails, nnd ballnst, only £1,200 per mile, and inclntling other expenses, less than £2,000 a mile, a sum so that it hardly seems worth trying to reduce it. Compare it with the £10,000, £20,000, and even £50,000 a mile of some of the earlier Iincs. The Hendigo line originally cost £50,000 a mile, representing, at 6 per cent. (the rate then paid), £3,000 per annum interest per mile of railway. Kow we make lines for £2,000 a mile and pay "1 per eent., giving £80 per annnm per mile. As for extremely small and light lines, sueh as the Dccauviilc, as nsed in Queensland, I e!m say but little. I should not be averse, however, to one being tried as an experiment, if it can be done under £1,000 a mile. I heard a lecture by Mr. Van de Velde, at the Institute of Engineers, upon these railways. What he said seemed very good and very much to the point, hut the only thing was that his narrow railways, when they eame to be made, seemed to co~t as much as our railways. I met my brother a day or two afterwards, and he said, "Mr. Van de Veldo seems all right, hut I can make 5ft. 3in. railways at the sftmo cost." Seeing that the difference is so small, it seems to me to he a mist,ake to make a break of gauge, which, at the smallest estimate, is equivalent to a number of extra distance, and with a good many classes of commodities there is a risk of damage every time the stuff is handled. You cannot put coal from one waggon to another without breaking it down into K!ack more or less. You cannot transfer fruit or agricultural products withont their suffering.

1739. If the traffic is stoppeil at Flimlers-strcet and hns to bo transhipped to Hpencer-street, that is a break of gauge?-Y cs; and I have no doubt the goods suffer a good deal by it. It seems to me that comparisons between this country and Indin are dilficult nnd somewhat uncertain ; the conditions are so totally different in a multitude of ways. I the comparison should be between this country and America. I have travelled in America, though not very much, ami I have been through hundreds and thousands of miles of country very thinly popnlatc<l, reminding me very much of the sparsely populated districts of Australia, and you do not find the narrow gauge there. The only place whore the narrow gauge seemed to have been mnch used was in the Dcuver and Hio Gramle line in Colorado. I saw n eonsiderabie space where the narrow gauge had been taken anJ the broad gauge put down in its plaee, und that was in a country so thinly populated that ono would a wheelbarrow wonld have taken the traffic.

17 40. Was not there a large city at the end to provide the traffic ? -There was New York at one end and San Francisco at the other. Denver is a city about the size of Adelaide, and Salt Lake City is about the size of Ballarat, and it took us about 24 honrs to travel. Pucblo was the only place between worth mentioning.

17 41. By 1J'fr. Craven.-IJ avo you seen any other tramways at. all in those eountrics? -No ; I have seen the electric tramways. For suburban work they ha n; the electric tramways coming in very largely, even in little villages. Villages with two or three thousand people will have an electric tramway.

1742. Are they worked cheaply ?-I have not the statistics of them. With the enormous extent of them and the way they are extending it looked as if they were worked cheaply.

1743. By the Hon. D .. i~felville.-Sweden and Korway have 12 miles of 2-ft. gauge lines; Switzerland has 17 miles of 30-ineh; England has 16 miles of 20-inch; Austria and Hungary, 170 miles of 30-inch ; Alsatia and Lorraine, 1.5 miles, three-fourths of which are on the high road ; in Sax on-W eimar there are 27 miles of metre gauge. Why do those people make those railways if they are so utterly

Willmm a. Kert~ot, 13t1t August, 1895. 108

wrong ?-I do not say they are necessarily utterly wrong; one would have to know all the circumstances of the case. They might be right; but the point that seems to me to be so notable is this, not that they have made them, but that after having had narrow gauge preaclted to them ad nauseam for a quarter of a century they have made so little.

17 44. 1Ve are not entertaining the 3ft. 6in. gauge as a narrow gauge ; the narrow gauge of to-day runs from 1ft. 9in. to 2ft. 6in.-you are comparing two broad-gauge railways for ns; what we want are comparisons as to cost of construction in tliJficnlt eountry and eaRy country of the two gauges. Had you adopted the 2-ft. gauge as the Engineer-in-Chief has done, would the gross result of the cost of con­struction in such a country as 1Valhalla come to nearly the ::ame ou a 2-ft. gauge and a 5ft. 3in. gauge ? -I may say that if I saw the sn!'Yey dra"rdng~, including- u sufficient IJumber of cross sections of that line, I could answer very much better than I can withont them. I have hcen in the \-Valhalla country, but it was long before railways were thought of there. From my rc:cdlectiou of the country at \-Valhalla, it seems to me that if you cut out a shelf on the sides of those ranges that would cany any sort of railway at all, very little more would carry a oft. 3in. railway. If you could have a railway with one rail only, and have it balanced like n bicycle, :;tm your cutting wonld rnn to very little lese than the present cuttings.

1745. You agree with Jiir. H.cnnick that in difficult coantry the 2-ft. gauge, all other things being equal, will cost as much as the Jft. 3in.'!-I do not sec that there would be any enormous difference in country like the \-Valhalla country, as I remember it

17 46. \Vhat would it be in an easy country ?-Nearly the whole cost then is for ballast, sleepers, and rails; there it is not so much a question of gauge as weight.

1747. Do you agree with Mr. Rennick that. in the construction of a line there is very little margin?­If the rails are of equal weight they will cost as much whether they are near together or far apart, and yon must have a certain quantity of ballast. If you make both railways with 60-lb. rails there will he very little difference in the cost, hut if you make one with 60·lb. rails and the other wirh 10-ll:. rails, there would be a difference of about thrce-fonrtlts in the cost of the rnils. At the preseut time we have the fact that the mallec lines are being made, everything r1bove the formation, ballast, sleepers, aml mils, for about £1,200 a mile. I looked over those figures a day or two ago, and they 1werage, rnils, £700 a mile; sleepers, £300 a mile; ballast, .£200 a mile; total, £1,200 a mile, exelusive of formation, which iH constant whether a line is in easy or difficult country; then the cnttings, embankments, tunnels, aud "0 forth will vary with the country. I understood from :Mr. V 1111 de Veldc that in France, where they have public roads to use, it was not worth while making their 2-ft. railways for less than about £1,800 or £2,000 n mile-that puzzled me very much.

1748. Then we understand that whether it is a level easy country or !J, difficult country, your idea is that the margin of mere construction varies very little ?-Y os, that it is 11ot a largo matter; that the cost of construction of a railway de11ends on a vast number of things which are independent of the gauge.

1749. \Vhen Mr. Robb's engineer excavated that swamp containiug milliom; of ton8 of stuff, he adopted the 2-ft. gauge; why should your engineer abandon his theories and a cl opt the 2-f:t. gauge, when there is practical work to do ?-That wa~ a special cabe-that waR not an orclitmry railway.

1750. I have noticed t!Jat in practice the practical men for the narrow gauge when there is weight to carry. At Walhalla for the past uine years they have constructcll a narrow-gauge line of 18 inches, and have taken 89,000 tons of stnii along th:tt 18 inches; they empty it clow 11 the hill?-Yes, with horses and small trucks-it: jg a wheelbarrow business compared with a railway lm~iness.

17 51. It is 9,000 tons a year ?-A railway would take that in n day. 17 52. :l\fany of those railways that have been constructed at a cost of £100,000 in Victoria do not

even take the \Valhalla traffic. Take two railways that have been discussed to-day-1\ioe to Thorpdale and :Morwoll to North Jiiirboo. Our engineers set them out us great examples of where they would have constructed the 5ft. Bin. gauge because of the trnffic ; do you know those places?-Yes, I have been at 1\ioe.

1753. Could you have suggested a railway that would have covered tllC working expenses of that line and a little of the int:ereRt ?-I do not know enough of the district to answer that. My recollection is that the population was excessively small. I saw the railway while it was being; :mrveycd, rmd the largest part of it was through a primeval forest.

1754. Now take the .'a:l.orwell to North .Mirboo line ?-I have not been on the .Mirboo railway. All I can say in these cases is that the rnilways have been pushed out into the wilderness, whore there was next to no population, and th!>,t for some reason the population did not follow the railway, therefore the railway has not cnongh to keep it going, so one on any glmge would not have done mnch better.

1755. If \VC had 11 2-It. gauge line in either of those districts, what would the result have been'!­I do not thiuk the result would have been hotter.

1756. The net result ha8 been that in both cases we have lost £7,000 I! year; which would have been better in that case, the 2-ft. or the broacl gauge ?-I ilo not think it would have made very much difference. Doe::< that .£7,000 include itlterc,;t?

17 57. It includes intercst'?-It seems to me it would have been be~t not to have mal le the railways at all.

1758 . .By the Chairman.-The total cost for those lines was £87H,67l, and the tonnage carried annually is 50,168 tons ?-It is plain that when those railways were made tl1ero was not the population there to justify their being made. They were made, I presume, in the hope that the popullltion would come when the railways were made. The population has not come, and the rail ways are stranded.

1759 . .By JJh. Craven.-Have you been to Queensland and seen the sugar lines ?--I have heard of them. I do not know them of my own knowledge.

1760. Would they be suitable here ?-It seems to me that to try one of those little lines in some place might be an interesting experiment-I could not predi(;t what the result would be.

1761. .By the Chairman.-Have you any idea why a11 the saw-mills in the country make those light lines ?-Their tramways are simply wooden ones, and are, as :t rule, intended to curry one big log down from the forest to the saw-mill. I believe that vory good work ean be done 011 nanow gauges, but my position is that the cost of working a railway to a great; extent, imlepmulent of t1H_, gauge, and that a vast deal has been done and eau be done without altering the gauge.

109 Williaul 0. Kemot, 18th Augm;t, lll96.

1762. There are hundreds of that are uctually starving far want of means to get their pro-duce to market, and they would be with something like what we saw 'Vandoug, at a cost of £±00 or £500 a mile ?-That is not wlmt Mr. Van do Voldo recommends, a £400 or £500 a mile line. One would have to work the thing out from 1t proper survey to see if that were likely to he realized.

1763. By Jl!f.r. Harris.-It has been realized and is at work?-In some pluees. 1764. By the Chainnan.-The diffieulty in in working out the."e things. There is a bridge in the

district [am interested in, and it cost under the Government snpervi~ion £1,400. It lasted a fair number of years with a little repair, but it fell down, and we had to put up another one-we put up a very much better bridge for £600 ?-That is opening ttp another l[Uestion as to private engineering Ve1'S~ls Government engineering. lt may Le that since the Cflrlier brit1ge was built the priee of labour and materials was reduced -many things may change, and it must be remmnbere(l that probably the £1,400 for the earlier bridge may have ineluded a considerable sum for mnbankments leading up to the brirlge, which, once there, were there for goorl, and wore used for the new

1765. By j~Jr. B1aton.--Aro you aer1uninted with the sharpest curves on the overland line to Adelai(le between Servieeton and Adelaide ?-It is 10 chains, I think, if I recollect right. I am not aware of any shnrper curve than that.

1766. By N't. HaNis.-This is a lette1· by Mr. Percival Heywood to the Times:-" It may be not.iee1l as a eurions fact that the and eommendable predilections of engineers for the stn.udard gunge, whenever obtaim<hle, appear to lead them, where circumstances compel the adoption of a narrower one, to advocate as little reduction as possible. Now the general result of foreign experience goes

to show that narrow gauges 30 inches approximate so dosely to a full-size line as to to a considerable extent, the of either ~ystem. This attitude is prohubly due to ignorance

of what can be clone on the mtrrowest gauges, for, in spite of the fact that many hundred~ of miles of lines of less than 2-ft. gauge arn at work abroad, onr profeH,;ional advisers persist in regarding such railways as mere to.vs. Yet a line of 1.5-in. gauge has been at work in this country for twenty year,;, on which thousands of passengors have been carried without a single accident, as many as 120 in ono train, over

as steep as 1 in 20, the good~ traffw l>ei11g worked in all weathers a long gradient of l in 11 without difficulty" ?-I do not know what particular railway is referred to. have travelled on a railway with one rail-that would settle the of gauge altogether.

The witness withdrew.

Adjourned.

[ Lette1· from 1lfr. Kernot in connexion witlt Narrow-gauge Railways.]

The University of Melbourne, 15th August, 1895.

The Chairman Parliamentarz; Standing Committee on Railways. SIR,

At the close of my evitlenre before your requested me to communicate any facts which, on further N>ln<lliPrn I might deem to he of value in conncxion question of gauge.

tlH,rero:rc su~mit the following as an amplification and compktion of what 1 have already said

I. Tbat I am COin-inced, after full tl1at tlH.>re is no difficulty in constructing on the present gauge of 5ft. 3in. railways lighter and chea]Jer any yet macle.

2. That such milwvys will cost bnt little more J.!Jau 2-ft. gauge lines of €qual strength in the first instance, will cost les; to maintain. aucl will be c:tpttble of carrying, at moderate speeus, the passenger carriages and good;;.; waggous at pr0scnt in use.

3, That tlw locomotives required for thc,,c lin0s will lJc practically iuentical in for equal loads on the 2-ft. but that owimr to the 2;reatcr width conveniently acecssiDic. will and more~ convenieiJtly shaped tmch<W<>o

to stcmn better, an~l to wodc_ vdrh fuel thnn the narrow-o·o,up-e enrrines. ~0 liable tO OYCrturn ~hod(] tJoey Jmppen to run off the rails. ~ c' "'

and cost to those required their mechanism more

and be able not be

-1. That 1 here will 1 c 110 <liilleulty oH

of tt;n lnil('s P~- r hour, aiH1 Ught hroad~gu.ugc railways in tra'tersingeurvC's of :3 chains radius at a

curves at lower speed. Consequently, in hillv country it will side>, ~ IH] reduce earthwork to a mmimum. • pos;;ible lo fol!ow the: sinnositit)8 of the

o. That the local ioH. "' choice of line for Flrst-Thnt

pnn:ll:t::ed o,~ n.:s~::rn:;1l.

railwayrJ, be conducted as follows

out. for a higl1-cla~s railway having mdins, and that the whole of the

grades of l in 40, for such a line be

Sccont1--Th:.t :et ail jlo;ut" ihe cost oE cnltin:,;s. cmbaukments, tnnnch1, or viaducts exceeded a ecrta~n a~~:c:nnL) to bt' fixed hy t:!w juclf;;nent. of the :Engine0r-in-Chief, te1nporary tleviation;.;, having ~; ruling g'nu1e cf l ill 2.) and a 1nlniluu1n cur\'a,turc of ;j chains rn.din~, be introduced, the 1naxlHlll~ll 1H in:~· ('Onfinct1 to straigl1!, portions oftlw line, n.nd being reduced n.s the curvature imTt·ased :ll~~._',:r:iaucc to the inuieul ious uf the dyuagraph on a sht)ft cxp~rimeutal piece of line_

G. ·From my lcGowlerlgc of th;c cutlntry ahout lie:.lesvill?, the Hl:wk fivur. \Yallmlla, Korumuurra, anu the l1lfiUH:.:nnot:s p:u·-,,, l e-tinmtc th:1t more than three-fourths of tlw wlw:c length Cc'uld be constructed on the p(•rmau0nt location witit imt i:J,ignille:mt earthwork:<, leaving lu<S than one-tourth to be made upon the lC:lll)IOrn ry deYiatiuns.

7. That a 1i1w '"' constructed would he ai< che(tp in ~he first instf\nce a• a :l·ft. line \YOnlu be in e>erv \\ tt;· >uperior to it in working, would break of gauge with all its attendant evils, ami, should ,·ircun:stanec>' <•t "".\'time warmnt it, could be f:radually impro>cu and completed by more ballast, he:cviet· 'leC'p< rs and ntils, and eon;;trnding vhtduets, tunnels, or heavy earth works, in lieu temporary deviations, until it became a first-cla;;s railway, capable of carrying the heaviest traffic. and all without interrupting the traffic for a single day. ·

William 0. Kernot, 13th August, 1895. 110

8. In conclusion, I desire to say that in the above proposals, including limiting grades and curves, I have kept well within what I know to be long-continued and successful practice in other countries. It is nothing new or untried that I am advocating, but something for every item of which I can quote successful and well­established precedent. As regards grades aud curves, which I should desire as far as possible to adhere to those named, I would not hesitate in extreme cases, when a large saving in construction cost was involved, to employ I in IS grades, and I!,-chain curves.

Appended to this you will find my estimate of a broad-gauge line which I consider to be in every respect equal, and in some distinctly superior, to that which the Engineer-in-Chief names under thl' title of "inferior railway, 2-ft. gauge, 40-lb. rails, &c."

Should you desire any further explanation, I shall be happy to attend and afford the same. I have the honour to be,

Your obedient servant,

W. C. KERNOT, Professor of Engineering.

ESTIMATE OF PROPOSED RAILWAY, TO COMPARE WITH 1\fR. RENNICK'S "INFERIOR RAILWAY," COSTING £733 I2s. 3d. PE!~ MILE.

£ s. d. Ballast, I ,000 cubic yards, at 3s. I 50 0 0 Sleepers, I,900, at 3s. 285 0 0 Rails, steel, 35 lbs., 55 tons, at £5 275 0 0 Fastenings of various kinds ... 40 0 0 I~oading, transporting, and laying 60 0 0 Carriage from Melbourne 50 0 0

----Total £860 0 0

-----Being 17 per cent. additional cost for a line which, in my opinion, would be at least 50 per cent. more valuable.

Ballast, 8 inches deep. Sleepers, 7ft. 6in. x 9in. x -!~in. Rails, 35 lbs. steel.

Particulars of above railway.

Light earthworks in hilly country I estimate at £I 50 per mile for the above railway, and £100 per mile for the 2-ft. gauge railway.

W. C. KERNOT. P.S.-The locomotives I should recommend would he similar to those originally used on the Windsor and Cambden

branches in New South Wales. The weight would be about 16 tons on six wheels, and the tractive force about 6,000 lbs.-W. C. K.

WEDNESDAY, 14TH AUGUST, 1895.

Members present:

MR. CAMERON, in the Chair;

The Hon. J. Buchanan, M.L.C., The Hon. D. Melville, M.L.C., The Hon. E. Morey, M.L.C.

Mr. Burton, Mr. Craven, Mr. Harris, Mr. J. S. White.

Alexander E. Duguid, sworn and examined. l766A. By the Chainnan.-What are you ?-An architect. l 767. Do you know anything about the construction of the narrow-gauge railways ?-No, I do not.

vVhat I have come here to bring under your notice is the mono-rail railway system. It was brought under my notice some two or three years ago by a contractor who had seen it working in the old country, and who, I know, was in touch with the inventor of it, and had a sort of agency for the purpose of trying to introduce it. He came to me to try and get it introduced to public notice. I was impressed with the thing-, and when the contractor went away he left all his papers in my hands. I think it is worth the Committee's inquiring into. After getting an invitation to come here I set to work to see whether I could find anybody who had actually seen a railway of this kind working, and I found a Roman Catholic clergyman from County Kerry who had ridden on oue between Listowel and Ballybunion, and he says it is working there per[ectly satisfactorily.-[ The witness explained tlte worl~ing oftlte system on tlte plan.] It is a single rail elevated about 3 feet above the ground and supported on trestles. There are two small guide rails to prevent oscillation. The carriages are on the bicycle principle, and tho carriage hangs ou both sides like panniers. The passenger cars are like our tnvn dummies, hut enclosed; the people sit back to back, with the wheels between them. K ubody would suggest such a railway where a broad -gauge railway should be built, nor do I think it would be choapcr than a narrow-gauge railway in a flat country, but in hilly country and where there are sharp bends it is c!aimcd by the inventor that, compared with the 3ft. 3in. gauge railway in the first construction, there is a saving of 60 to 70 per cont.; hut in the cost of the rolling-stock there is a ~aving of 35 to 40 per cont., and in tiJC cost of haulage there is a saving of at least 50 per cent. I merely take the figures from the book, and am not responsible for them. This railway has been running in Ireland for about seven years and is working satisfactorily, and there has never been an accident on it. It sometimes carried 1,000 passengers a day. It wa~ built primarily for the carriage of sand between the sea shore and the country districts; the carrying of passengers was not at first so much thought of. It seems to me that it is almost impossible for an accident to occur, because it cannot run off the rails or capsize. ,

1768. There are three rails then ?-Yes, but the two guide rails are so light that they do not count.

1769. vVhat are the rails ll1l1!le of ?-Stool, and the trestles are made of steel, but they can be made of wood if the lino goes through woodc1l country.

1770. How far are the trestles apart ?-They are set up like sleepers, about a yard apart, and the thing, from the pictures, loob like a post :1nd rail fence.

1771. By the Hon. D. Melville.-What about the speed ?-It travels from fifteen to twenty miles an hour.

111 A. E. Duguid, 14th Aug·ust, 1805.

1772. Does one engine do the whole work ?-The engine is a locomotive lmilt for the purpose -[showing a sketch]. A fairly big train will turn a curve of ,')0 feet radius.

1173. vYhat is the weight of the engine '!-An engine with three wheels weighs from 2} to 5 tons without the tender, and they may go up to 12 tons. I have a tlme-t~thle of the Irish !ine-[lwnding in the same]. That is not the only line of the kind running; there arc a good many running in America, and South Africa, and .France, but that, I believe, is the only one running in Britain.

1774. By the Ghairman.-What is the didtance between Listowel and Ballybunion ?-They call it 15 kilometres, which is between nine and ten miles. The thinl cl11ss single f11re is 1d. a mile, Hld., and Is. 3d. return; first class siugle ls. -!d., return 2s. I know it is very ingeniously worked out. The n11mo is the Lartigue Elevated Single Rail Railway.

1775. By the Hon. J. Buchanan.-What is the height of it ?-It 11veragcs 3ft. 3in.; but the height of the trestles varies with the inequalities of the ground.

1776. By the Hon. D. 1lfelville.-Do you know any one who could construct 11 mile of it here?­K o ; but I know what the inventor says he is prepared to do-that, given a contract for the construction of a line, he will lay down 011e-tenth of it, and if it is not according to specification and does not do the work that was 11greed on it should, he will forfeit the whole thing.

1777. Do you know any one here who could construct it ?-No, because it is paten teLl. :M.y friend who introduced it to me is in vYestern Australia~he was only the agent. It seemed to me that the saving claimed was so great that it was worth the Committee considering it. I will gladly communicate with the inventor and learn what he would do.

1778. Do you know any objection to it '?-The only objection this clergyman could give was that there was an uncomfortable noise. He says it is more than the noise on onr own railways. The passengers are sitting back to back, and the wheel is nearly opposite a man's car, and you cannot converse comfortably. The inventor gave an estimate that through wooded country, where the timber could be h11d for the cost or cutting, he would be prepared to build, say, 40 miles, and supply all the rolling-stock at £1,000 a mile.

1779. By the Ghainnan.-\Yill you kindly leave your papers for a day or two ?-Yes ; here are nlso reports by French engineers on it.

T!te witness withdrew.

J ames Quan, sworn and ex11mined. 1780. By the Chairman.-What are you ?-I{ailway engineer and manager, ]Jut not at present. I

retired three years ago from the management of the Glenelg Rail w11y in Adelaide; that is 11 6ft. 3in. gauge. With your permission I will read what I lmve written.-[ Tlte witness read the following paper

With the view of eliciting questions and assisting examination by the Committee, I have thought it well to thus jot down such information as without preparation may be fairly given.

My experience in the working of railwnys has been a long and varie<l one. I commence<l in 1858, and served an apprenticeship of five with the late Mr. W. H. Puge, then engineer and manager of the Rhymney Hail way, Cardiff-one of the art.eries for to the sea-board of the immense mineral product of the mmmtains and vallevs of Glamorgan shire and Monmouthshire. ·

I afterwards spent seven years wit.h the London and Sorth \V estern Company in practical work in the manufacturing and. colliery districts of L:tncashire nnd Yorkshire, and five years with theN orth London Company. whose line is one of the busiest passenger railways in the world. In the yrar 187<1~;) the late Earl of Devon and ot.her l;wded and mineral pro­prietors were providing as an experiment a cheap light line of rail way t.o connect the valley of Eskdale, in the county of Cumber land, with the :Furness ]{ail way, at the andent port of Havinglass, and also I or shipment at that port of the iron ore then being wrought from lodes in the Cumhrian Mountains. The vai!Py, though rich in natural beauties, wns notoriously the very poorest tratlic-yiclding district. in all England, a![(l the promoters of the HrLvinglass and l~skdale Hailway spent much time and thought over the solution of the problem how to construct., maintain, and work their proposed line in the cheapest possible ruanuer. The position was rendered much more difficult by reason TJrinmrily that it was quite out of the question to go through the small portion of good land, or to touch the homesteads in the bottom of the valley, so that the line had to be kept into the mountain side and follow the contonr of th<~ country. At intervals spurs from the mountai11s jtJtted out into the valley, and as the Board of Trade at that time prohibited curves of less tlmn 10 chain~ radim with 4ft.. 8~in. railways, it would have been ncccs;;ary to either eut or through all these rock spurs, providing the line was constructed on the standard gauge. To obviate such expense a.n of Parlil;ment, wns obtained to constrm:t the railwny to a g:aug·e of 3ft., with sharp cm·,·es round the rock spurs. The main line was eight miles in with high level branch of mile in addition, in order that tlw ore migl1t be raised by grn•lntion to facilitate its to standnrd·gange The whole line was a snceession of sharp curves and stc''P g'mdients, the sharp<>st of :l chains and the stcepe't gra<lient heiug 1 in 27. In one ('asc there was an incline li miles in length and haviHg several very sharp H curves in that gradient. I should perhaps here state that. I was selected as lllaimger by the promoters eolely on account of my eombination of much )>rrtetical, with a little theoretical, and training. 'file line was duly passed hy Colonel Yolland. Henior Inspector of the Board of Trade, and formally opened fot· public and goocis traffic. Bri<lges were built at all public road crossings, and we ha.d no signals, signalmen, or _gatemen. aml sidings were estahliRhed at intervals of about ~Yery 1~ miles, hut no station st.aff was kept at any intermediate llhtee. All points were kept loek(•d np, the keys being carriPd by the guard.

For the first five years about 10,000 tons of iron ore was carried annually for transhipment. to stan<1unl-gauge line, and about 500 tons of coal, lime, and general goods were also transhipped from standard for carriage into the valley. ]J'Jve trains were run daily <•ach way, two hcin11 trains only, and the r!'maining were mixEd-passenger, g0ods, and iron ore. To provide against hrPnkini! at sharp curves or sudden chaug-es of grade, I always lutd the passenger carriagf:s tnarshallc~rl next the f'ngine, van at extrerne TC';'ll". By this arrangement no necic1ent or damage ever took place, though break•away were n:ry occurrcnc<:'. We had no bogies. eitlHT radial or lateral, OP. any vehicle, either engines or stock, hut the wheel base not allowed to exceed 9 fel't over all in any instan<'e. The Jo,·omotivcs were t.lrc ordinary sidr: tank engine;;, weighing 15 tons when loatletl with coal and water, cyliuders 10 inches by 17 inches, •ix coupled wheels, 2ft 9in. in <liameter, the fire box ht:ing placet! between trailing and driving axles.

With one of these smrdl engines I have with p0rfcct Cai<e taken 20 trucks of two passeng-er earringes, and a brake-nn, the whole weighing about 120 tons gross, np gradients varying from 1 in 1 in -td nt the steepest point, cvrves also occurring in these grades. The locomotive working five trips, or 80 mile8 per dav. Consumed on aHrage 12 cwt .. of coal per day. ·

The whole cost per train mile for working this line under the conditions I have named averaged, including repairs and mnuagement, about lOd. per train mile.

'l'he permanent way of the line consistct1 of t.o the yard rail;, LtBtenctl to slccncrs with the onlinarv dog-spike, and, in addition, two faug-holts with cliTJS to en'ry found that neitlwr f:Lng-bolts or dog-spikes would hold the line in to gauge at the very sharp curves, and half chairs, under Bhonlder of rail, and spiked to sleeper;, had to be added at such curve>, after which no further trouble was experienced.

The total cost of the line, including 2 locomotives, i3 passenger carriages, 1 brake-van, and 36 trucks, over £10,000 for land a.nd compensation, and also the bnildiug of 1t hridg~s, and provision of about 14 snml!er sheep creeps aud water­course bndges, rrmounted to £32,000, or at, the rate of £n,h55 per mile stocked ready for work.

It is stated that inasmuch as the carriages and trucks used for narrow-gaul.(e lines lmve considerable overlmng, the cost of road-bed cuttings, &c, must, conse(llH'ntly, approximate closely to the cost (>f the idt. Bin. line; but if economv is the direct cause of departing from the hrnar\er gnug·c, then in pursnanee of that same economy, surely the overh:'tng of vehicles can be, and should be, curtailed. and the superstructure of rolling-stock np in accord with the width c.f the foundation. '

l think it only right to st;tte that the simple common utility standard I always adopt in railway working would, if adopted on the State railways of Australia, be calculated to have much to do with the successful working of the lines independent of whether suoh lines were either of one gauge or another. '

Jame~~Qua.n, 14th A11gust, 1895. 112

I will give the Committee an instance of what has been done with a 5ft, Sin, gauge railway here at home in Australia.

In 1883 the comlition of the Glenelg Railway Company, AdelaiUe, was considereu by everybody in Adelaide, including its own uirectors, as being hopeless beyond the shadow of hope. .No dividends had been paid for many years. The whole ~ncorne pf tile railway, and, in addition, large sums-proceeds of sales of surplus lands in boom time-,were swallowed up m workmg expenses. Bonds for £20,000 were falling due, and no one would lend money to renew. rhere was a heavy overdraft at bankers, who summarily stopped credit and demanded a clearance. To accentuate the difficulty, the Glenelg Railway, even under those conditions, was the cheapest worked railway in Australasia, and there seemed nothing before the company but liquidation, in which the whole capital would be lost. The real cause of trouble was that the gross revenue amounted to only 2s. lOd. per tmin mile, and the expenditure was greater than the income. .

I as a forlorn hope, specially engaged by the directors to put things right. I at once reduced the workmg expenses per cent. all round. The train mile cost, which I found Ss. 6d. per train mile, was brought dowri to ls. lld. to ls. ~Od. per train mile. The profits of the first eighteen months I devoted to paying the debts and renewing several of the engmes and por~i9n of the line. Then we began payiug good dividends the much ast~nished s~arehol~e.rs, and the pa;,:-ments of such diVIdends have regularly eontmued up to the present d<ty co3t per tram nule still remammg as I fixed It at ls. 9d. or ls. 10d. per train mile. '

The late Minister of Hail ways here employed me in a certain matter in January aml February, 1894. I spent much time on the railways, and took very great interest in the Yarious branches and aspects of the working of the Victorian lines. The cost of WGrkiug the Victorian railways could be phtced ou such a basis that Victoria's fiuancial difficulty would, once and for all-so far as regards the rail ways-be effectually disposed of.

Yesterday an honorable member of this Committee read out the startling statement that in India it was notorious that the small narrow-gauge engines could take o-reater loads than the lar"el' broad-gauge engines. The learned Professor of Engineering at once rejoined that either the ;~erson making that state~ent mad, or that he, the learned Professor, was the madman, because he accepted as an obvious fact that the greater must necessity be more powerful than the lesser. Now, it was at once plain to my mind that the Professor was perfectly correct in his theoretical conclusion, and the engi­neer writing in India just as correct in his practical though seemingly impossible statement, and that neither gentlemen should properly be charged with madness. The explanation of this seeming paradox is that the small narrow-gauge engine is built expressly for taking large loads at a low rate of speed, and by means of low wheels and concentration of power that object was obtained. As an instance, the express passenger engine of, say, the 10 a. m. King's Cross to Edinburgh, with its lO·ft. driving wheel, weight of 55 tons, and enormous awe-inspirin~ proportions, seems the very embodiment of all that is strong and powerful. Yet the little insignificant-lookirw six-wheels coupled tank engine engaged shunting coal trucks about the yard would, in a trial of strength, drag both th: express monster and its load. The small engine could not take the express train to York in four hours, and the larger engine is pmctically useless for shunting coal trucks.

When l told the people in Adelaide that the Glenelg Rail way would pay good dividends they looked at each other and said, "This man is mad." However, the method of my madne8S returned to the shareholders some 70 per cent. of their whole invested capital by way of dividend and preserved the property,

In 1890 and 1891 accident gave me a clear insi~:ht into the cause of the trouble Victoria was then just beginning to feel as regards her railways. I told the Hon. W. Shiels, then Premier, that I could place my hand on the dark spot, stop the trouble at once, and in six months present the Victorian railwnss to the world as a valuable profit-yielding concern. I received a very civil reply from a gentleman at the Crown Law Offices but no doubt my utteranee was looked upon in the usual way as the raYings of a disordered brain. '

In 1894 I told the Hon. Sir James Patterson, then Premier, and also the then :Minister of Railways, that if I had the working of the Victorian railways as a strictly commercial concern, just as I worked railways the property of companies, the cost of working and maintaining the lines would be £686,787 per annum. Sir J:unes, I rather think. set me down mad, but Mr. Richardson, beinjl, according to those who differed from him, mad himself also, at once saw the perfect correctness of what I stated, and desired strongly to secure my services permanently, but went out of office before he could make the necessary arrangements.

I repeated my statement as to the working cost of the Victorian railways to His Honour Mr. Justice Casey in the early stages of the Railway Inquiry Board, and offered (of course on certain terms of remuneration) to supply the Inquiry Board with full details showing minutely in every particular how the Victorian lines could be well and efficiently worked and maintained for the sum of £686,787 per year.

The Committee asked yesterday, but in vain, for a standard as to under what circumstances they would be justified in recommending the construction of a light cheap line.

I give the Committee a standard which I guarantee will come out correct providing the light railway be made, main­tained, and worked on principles of sound practical common sense and utility.

A narrow-gauge railway costing not more than £3,000 per mile stocked ready for work may be safely undertaken in any country district of Australia where the 11atural probable tmfrle of such tlistl'ict will yield to such railway a gross revenue of £250 per annum per mile of railway to be made an<l worked-Over 20 miles long, reduced to £200 per mile per annum. Cost, stocked ready for work, not to exceed £3,000 per mile of railway.

(See also my letter to the Committee, dated l!lth August, 1895.)

1781. You say if the earnings of the narrow gauge amount to £250 per mile of railway per annum it would pay?-Yes.

1782. Y on would have to show the cost of the constrnction ? -Providing that it does not cost more than £8,000 a mile. Take a railway ten miles in length, you need in the first place £1,200 a year to pay your that is £30,000 at 4 per cent. Your tot!ll revenue at £2.50 a mile would be £2,500; that would leave you £1,300 for working expenses, which would be ample.

1783. \Yhat gange ?-Whatever gauge you like. There is not much difference as to gauge in the urn.rln•n"' of light traffic.

We want the minimum cost of collstrnetiou for narrow-gauge lines-we are getting lines constructed at £1 ,.'lOO a mile with the present gauge ?-That is without rolling-stock. When I say a railway complete, ready for work, I mean provided with rolliug-stoek, supposing you made a railway by itself, not connected with any other.

17tHi. Have yon any practical experience of a 2-It. gauge railway ?-Only what I have seen by visiting the 2-ft. gauge railways in North \Vales.

1786. For how much could snch a railway be oonstnlCted in this oountry '?-If you can make a 5ft. 3in. gauge railway for £1,600 a mile without rolling-stook, you would be able to make a 2-ft. gauge railway for, I should think, about £500 a mile >vithont rolling-stook.

1787. \Vould You vouoh your renutation as an that that could be d011e ?-I do not think it would be fair· to do ~o, because" the JH~ohability is that twelve months the cost of everythi11g will be double what it is now. You can bny rails for £3 I Os. a ton now, but they may be £7 a ton then. The railway you speak of for £1,600 would cost, more in rongh country, hut the proportion would be the same. If the cost for that were £1J,OOO in a rongh eonntry, the narrow gauge would oost £1,000.

1788. Where would the saving come in in the construet.ion of a narrow-gauge line as compared with a broad ?-Your road-bed for a Mt. 3in. gauge lms to be three times !ts wide as for a 2-ft. gauge. The

for the stan,iard is 9 feet, aml for a light 2-ft. gauge railway it wonlcl be quite long enough if it were 3 feet in leugtb, that is witll 6 inohcs projecting on either side.

1789. Would a 3-ft. sleeper be solid enough without the extra amount of ballast ?-Perfectly; it all depends on the width of the superstructure. lf you make your rolling-stock too wide and too heavy, no matter what the gauge is, the train will not rnu steadily. The width of the superstructure must agree with the wldrh of the foundation.

1790. You did not say how much you raised the earnings of the Glenelg line ?--No, the earnings were not raised, but the cost of working was reduced.

113 James Quan, 14th August, 1895.

1 i91. Could von red nee the cost of working 1lmt much that you would raise that railway from a Rtate of bankmptcy '(o giving 7 or 10 per eeut. to the ~hareholders '?-Yes, the earnings decrease;<!. The postal authorities stopped the mail :otcamen; from cnl at. Glcmclg, and that l!H:lalH a loss of £50 !t "·cek to the r;tilw:ty;,;.

1792. An,] ;·on were able to maLe the lino 1793. You ·left it three year,; ago

?-¥es; to pay cliviclend~, thongh no one cxpectccl any. three years lncit November. It is paying now, and

coutinw•tl oa my svstem. 179-J. Yoni· system was merely re•lneing the wages of the people I raised the wages

of most people in the ;;ervice. l /9;1. Then you did away with some of those employed ?-If I ditlnot want a man I did r;ot keep him;, 1796. If you luul ten men working there, di1l you heep the ten or reduce them to five, s1x, or seven r

-1 kept no one unless he was required. The numher of persons employed was reduced. 1797. By wlwt percentage ?-1 ~lwnid say 20 per cent. of the entire ~tnli. There was one Yery

seriou~ item; I reduced the coal hill abont iO per rent., the fuel [,iJl for the They were coke; they thought coal was not suitable for bumiug iu their locomot.ive engines-they did not know how to use it.

1 79R. Did you rednclil the maintr-uaneo considerably. 1799. About how much ?-The ~ys!tnn they bad in ccuuexinll with t.heir perm:1nent way was the

old ono, the one in in Victoria, ~mall gangs, and thev had nineteen men with two i;Jspectors. 'Vhen I took the line 1 made one of eight men, who not only maintained ilw line a great deal holler before with the 21 also-- rclaid during my eight yea ne; :1hout "eYen miles of the li11c with new sleeper~ and new without calling in :wy extrB men to assist in the work. The sy~tUil of working that I ebtahlisherl in I::>i:i3 on the G leuclg line is there still in eYery part.ienlar.

11100. By i1fr. J. 8. W!tite.-llow did you arrange to share the expenses with the Govern-ment line. Your Ol\11 line ran to Vieroria-strcet, and ,YOU ran oYer the Government line with yonr line?­'Ve took om· own mnnev. \Ye paiLl them £50() a Year for runnirw fi \·e trains a into their stat.ion-it was £1,000 at one time. 'l'he tratllc wa~ £26,000 a" year when l'took the company's afbirs in hallll. The last Jl'ar it amomitc,J to £21,000 IJecanso of the bad times. Jt. was a period of great prosperity in South Australia in 188:l; new works were goiuo· on iu nll directions. I did not discontinue many of the tmiu~;

hat\ a ~:,ort ;:;ervicc; nmnin" ~> Goodwood. a mile nnd :1 half. I 8ai<l theJ' must do awav with ,., / .,. t.lmr-uo train seniee would pay with a lllile :.uul a half to nm, unless there is an immense tmil:e. There were uo train ncei<lents on the line in lllJ time.

lKOl. You Rtwed .')0 per eenL on the expenses, and the reasou was you burnt roal instead of coke ?-Tlnct was one v;;ry important item 1\llliJlll': many.

lilO~. Arc yon f<till in the eornpnny I left it three years ago. l so:l. Di<l yon resigu ?--I re,:igued to go 1 o I~nglnnd. I thought 1 was independent, hut nuforlu­

uatelv I hnilt mv huuRo on the sand. " 180±. By the Hon. D. )Uelville.-Cou!d yon haYe done your work on the Glenelg line with a 2-ft.

gauge '?-Ycl:i; with a 2-ft. g·ange it. is only a <1ne.srion of the working. lSOD. \-Vonld it have paid better \Yith a 2~ft. gauge or worse ?-I Jo not think there won id have been

much benefit in ihe ease of that line. 1806. Your cost of conRtrnetion, if applied, would have been only one-third ?-Yes, there would

have been that on the whole. I thought you meant the working expemcs. 1807. As the manager of a 5ft. 3iu. railway in South Australia you sny the narrow gauge would

have eo,;t onlv mw-thinl in coni'trnetion ?-Ye;;. loOS. -And the working expense;; would hanl been about the same or less ?-It is hardly a fair

ex11mplc, becanse on some <lays they had enormou~ crowds of people to cnrry. There have been over 40,000 people carried in ono day on that line.

180!:). You wonld h:1ve saved two-thirds of the capital in eonstruction ?-Yes. 1 S ](). \-Vonld you luwe saved anything in the rolling-stock ?-1 would uot like to answer that off­

hand. With a 2-ft. yon can do-anything that has to be clone-it i::> only 11 <pwstion of :1rrangement. The queRtion of gauge, in my opinion, awl I have ~tndied the m11tter for yo:m,, is not a matter of import­:mce at all-I mean, u.s far as the relative value;; of the line, or doing the work, are eoncerncd. I would rather sec a 3-ft. llne than a 2-ft., and \YOtdd not. atlYii!e going in for a 2-ft. gauge, hut you can do with a narrow-gauge raitway anything that is <lone on the busiest railway in the world-it is only a matter of arrangement.

181 J. By Jlr. J. S. Wkite.-lf you were going to constrnct a ruilway from A(lelaiJe to for the colony, would you coustl'Uct a 2-ft. line in preference to a ;')ft. 3iu.I-No, I think I won[,] make .'ift. 3in., because of its connexion with other ;)ft. 3iu. lines. H I had to do it indepc>ndently of auy oth-r eunrwcting r:til way I "·ould make it 3 feet.

1812. Take tho railw11y from IImnley to BalaclaYa, is that. 3ft. 6in. ?-Y cs. 181:). Is there mnch tr~mble iu exeb~nging goods on the main line there ?-Time is a comparath"ely

quiet lille, but the lino which is most wmth ;;tndying is the one to Brokeu Hill; there an enormous traffic to :mLl from Broken Hill has Let>n enrricd nn a 11arrow-gange railway, aml in the earl,Y tirneH, before they began stripping at l'ort Pirie, the whole of that wa;; transhipped from one line to 1hc other.

li'lH. By t!te lion .• 1. Buchanan.-You meut,ioncd that the Glenelg liue was not payiug, thnt there wa~ :<ome tronhlo; what was it-what. eaused it not to pay ?-The trouble waA thnt. the r('Venu~ of tl;o railway was iusullieieut to enable them to WOl'k it aceording to their st:mdttnl idca8. The tr(mble w;;,; tltat it cost more to work tho railway per train mile t,han they enrncd, n:Hl on my thc:n there were <lividends hcbiJI(l it, they tlwnght it wa5 too nhsurd. They pointed to the fact tl;at dw Govennneut cost per train milP was Js., and tin1t 1heir,; \\':t:i only 3s. 6d., so tlmt it was quite cvidPut that they eould make no fmthcT reduction-that was what tlw dircetors said. Their gro;,;s rc\-enue was le.~s than the \Yorkiug expense~ of the Govemmenl lines. 1 ;:uppose it is about the same iu Yi<"torin. ·-

li'll5. By Jh·. D·aren.-From Burnley to \Vaverley-road, Yi(>toria, the \\'orking c·xpPnses per tntin mile are ls. lOd.: Briglttou to Saudringlmm, ls. Sd. per t.min mile; Birrcgmm to Forrest. the worki 11 g c'xpen:<e:i arc ~b. Id.; Koroit to Port Fairy, ls. lld.; ::Yiurebison to Uu,dnnn·th, 3s. ld.; Ballarat East to !3uninyong, ls. Mordialloc to Frankston, Is. 6~d. ?-Dues that include repairs to rolliug-stock?

NARROW GAUGE, I

Ja.rrwsQuan, 14th August, 1895. 114

1816, Yes, maintenance, traffic, and loco.; so they are very much lower than you ?-I do not see how they can divide it that way to be correct.

1817. By }rfr. Bnrton.-I-Iavc you any reservation in yonr mind as to cheapening the cost of construction on the G lenelg line ?-K o, none whatever.

181H. In this W<Ly, where a line wonld certainly be constructed cheaper on the narrow gauge than the broad, would yon l•nve to spend a larger amount on the rolling-stock to convey a large number of people travelling suddenly 011 one day ?-Yes, you wonhl have to provide more carriages and locomotives.

1819. To remove 40,000 people from Adel:tide to Glenelg, how many more trains woulJ you require on the narrow than the wiJe ?-On the ~-ft. line I should say it would have been necessary to have had twice the number of vehicle~.

1820. And twice the number of locomotives ?-Ko ; you would want more carriages. I do not think you would require :my more locomotives-you would increase the length of the train; it is pretty nearly a leveL

1821. vYould not increasing the length of the train increase the weight of the load ?-Not materially, because the carriages would be much lighter, not half the weight, and probably would not cost half the money for construction.

· 1822. You would not have to discount your statement bv the cost of rollincr-stock ?-No, I think it would make very little difference. "\V e built a narrow-gauge ;:arriage in Englanu" ourselves for £70, for high days nnd holidays-it was presentable. ·when the pnsscngers were packed well in, I have seen lOO in it on the 3-ft. gauge railway; it, was a four-wheeled carriage.

1823. By Jlfr. Cmt•en.-Could you not have just as light rolling-stock for the ordinary gnuge as the narrow gnuge ; has the question of gauge anything to do with thnt pvint ?-It has, because you must make your timbers stronger if you have a wider vehicle.

1824. How much wider is your 6ft. 3in. gauge rolling-stock than your 3 feet ?-On the Glenelg Railway, 5ft. 3in. gauge, the carringes are 10 feet wide,

1825. And what width ou the 3 feet ?-About.') feet. 1826. Could you carry passengers on the narrow-gauge stock only 5 feet wide ?-Decidedly. 1827. You say that the 1mrro1v gauge costs a third of the wide gauge ?-The road bed, the earth­

works and formation, and the ballasting. 1828. The ballasting a thinl ?-Yes. 1829. The rail is the same ?-If you use the same weight of rail, yes; the sleepers, one-third less;

the ballast, one-third less ; and the excavations one-third. 1830. What do you base your calculations on as to earthworks ?-On the difference in the wiuth of

the line. 1831. Have you ever known work of that description carried out iu actual practice; have you ever

taken out the quantities of a line with a 2-ft. gauge and one for a 5ft. 3in., antl maue calculations as to the actual cost ?-No, only roughly.

1832. Then yonr estimate is guesswork ?-It is a rough estimate,. 1833. As to that enormous traffic you talkeu about-could you carry that on a narrow gauge during

the day without increasing the nnmhcr of the rolling-stock?-You would have to increase the number of vehicles.

1834. If you were pntting down a railway from here to Adelaide, and there were no railways, would you adopt the narrow or wide gau;rc ?-I \voultl adopt the 3-ft. gauge for a new country.

l ~35. What do you think wou Jd be the co~t of haulage per ton per mile on the 2-ft. gauge­geneml goods on ordinary lines '?-I have actually hauled lOO tons for lOd. a mile.

1836. \Vhat would you reckon as a general thing ; take om coal that we are getting from Korum· burra-what do you think it would cost to haul it on our present gauge per ton per mile ?-I would have to go into that.

1837. Wlmt does it coc;t per ton per mile to bring down mineral from Broken Hill to Hamley lhidge; the line is in two gauges--what would be the cost per ton per mile of hauling the mineral on the aft. 6in. gauge to Hamley Bridge, and then from there to the Port on the other gauge ?-I would have to go into some calculations.

1838. I mean simply the hauling cost, just as it runs now ?-Of course a larger engine on a 5ft. 3in. gauge will have a larger fire-box, and consequently will burn more coal than the other; but the difl'erence in the cost of haulage will not be very much.

1839. As to maintenance, would you consider this a high price-six-tenths of a penny per train mile run ?-I would like to have all the particulars.

1840. I thought yon were an expert on the cost of maintenance ?-So I am. :?vfnch woulu depend on the number of train miles you run. If you piled up your train miles it would decrease your nominal cost of maintenance, looked at in that way, though actuoJly not so.

1841. Would you think three men sufficient to in effi.eieut working order a line thirteen miles long, with a train runniug each way on it once a day do not believe in three men-I am very much against small g:angs-1 like more men autl longer lengths.

1842. ~With 11 line thirteen miles long to rrmintain, how many men wouhl yon put on ?-I t.hiuk three would be sulticient, if they are men, and tl10re is not anything cxce>ptional.

1843. If the Victorian Hailvmys Department are that amonnt for nmintenance to such a line do you think they are doing their work properly cannot i3ay without full investig:1tiou and consideration.

1844. As to locomotiYe charges, do you think ls. O~d. per train mile, including everything, would be a fair thing ?-I could not answer that without going into the figures. I never dh-ideJ my locomoti\ce charges ; I never had time for ornamental bookkeeping.

1845. Would yon consider, t11king the total for maintenance, loco. eharges, carriages, wages, traffic charges, and compensation !tml geneml charges, 3s. 2~d. per train mile too high ?-V cry much too high, I think.

1846. On the lines I read out the average is only 2s. 5d.; would you will give me the questions I will get those things and look into it. questions without :malyzing the details.

yon consider that too high ?-If It is impossible to answer the

. 115 J:tmesQuan, 14th August, 1~95 .

• 18·17. Take the liuc from l\1affra to Bringolong, llw revemlC is £354 a year; what would you do to give a conn1ry likn tbt nilwnv commnnieatioH ?_.:_I \nmhl uot give !t railway communication at all ,.d_th tha.t re\-enu'--'· Hut i:~ t!1i~ {'a::;:~ i.-.: l11·ukea Uo\VlL

1 . nu. 1:om•J ·11 p t•J £1,000 a year; would you· give distrl::ts ?-I Lava 10 the Co:unit.(:c;e a c<tand:ml, nnil it amounts to this-that no mil-

w:ty s~wnl· I be m:ule 1:nlc>ss th•.•ro " i'Oi'ott;:o of £250 a mile per annum up to twenty miles, and £200 over that di.:;t.·mcc.

The u>itness withdrciv.

Henry 'IYillir.m Hargrav.o, ~worn and examined. • lk.19. By the Chn:irman.-What :no you ?-An AssGciatl\. Member of the Institution of Civil

Engineer~, London. I h<;ve heeu in tb;,; (•ou.\try a~JOLlt -:ix yen:,.. I have ltitd twenty years' experience in engineering, mostly ia councxiou with t!to survey aml constnwtiou of railways of different gauges in th2 Austr::dinn eulonics;

1 H-'50. Have von n;udo anv 1H'ro ?-Nnt. in Vietoritl,. have ma,fo them in Sonth Ai1~tmlia an(] Tn.-;m;nia on the ,5ft. 3in. I mrule n 2-ft. one in tionth Australin. I wns on tltc eontr:tcto:·s' lines I! ere FitH'V0\'5 a1 the \V est ?.Iclbonrnc Dock. The railway was llOL cun~irnetctl in [)z)nt:1 Anstrali:t on the 2-l'i. survey r ma,le. I ont all the linos for }\Ir. Ro!Jli in the \Vest :Jielbourne ;;i:x, mile:> in 1dwr~ lciJgtbs, workerl by locomotives. \Ve had six nn.rrow~ gangc Joco1notiYcs ti:erc.

1851. What wa~ the r~mdo there ?-The steepest gratlc v•c wol'l>:ctl was from 1 in 60 to 1 in 66.

Dock. 1852. \\-as nut. it .taken out of a deep ?-Ye~; we wore o:x:mtvatiug the \Vest Melbourne

1853. Could yon got tlmt.e~;;y grade coming out of the pit.?-YeB. 1854, Was there any alrerntion from the start to the prescm time in the width of that railway?­

\Ye hntl twQ systems of railways tlierc, oft. 3in. gangc nnd 2-ft.; they were carrying out distinct items of work. The Mt. 3in. trucks >Yerc lblivoring to the HnihvnyB Department, ail(l going on to the Government lines. The Victorian H.ailways Dcpnrtment look om lmulecl trucks nnd ran them out to North :Melbourne. The 2-ft. g:mgc tmcks were nsed t!wre bocauBo they \Yore tip trucks, and we used them for reclamation purposeA, nml they were handier for getting about; we could tmn slwrper CU\'Yes with them.

18ii5. \Vas it ncce~:,ary to ,;harp cmves down tlwrc we had n lot of comers to fill in. 18;)0. How did they Wl)l'k on those slHu'jl cm-ves 'fhc sharpest cnrvo wo usetl there

>iUS a 2-clwin ntdins, except between and eroselings ; they were about 70 foot mdins ~tt the turnonts. Hl.17. Can you tell liS why tho 2-J't. mtlius '''!\S adopte(l there ?-Simply on account of the tmcks

being aule to tip; they were t'ide-tipping trucks, and bccanse wc couH use sharper radius cmves to fill in the comorfl of the r.Jclamrttiou at awkward plnees. Ii' we had not used the narrow gnugo thoro the corncm I spen.k of v,rouhl have to Le filled ii1 drnyB or !J:crrow~; wo could tum about anywhere with them.

1858. \Vhat was the sharpest cune it would be safe to run a broad-gauge railway on at tlte same place ?-Tho sharpest curve I laid out thoru for tlw broad gauge was 8 chains radius-we were taking loads.

You do not tLiak it would Envo t:o havnthem loss ?-No. lSGO. \You]([ it be ntlv:mtageou~ to have a curve if you thouu;ht it safe ?-No, I do not

think the m1gincs we hml there wo.uLl work on sharpel' curn:s satisfactorily with the heavy loads we had to take,

186i. What kind of engines do you use on the narrow line ?-They were German engines, made by Kranss nml Co., 1Iunicoh. •

1862. \Vlmt loaJ woni<I that engine cnn;t on the narrow gange with 1 in 60 gradient ?-The heaviest lo:;.,[ we took np vrit:1 one narrow-gaw:;o a 1 in G6 gradient, was 48 tons.

1S6;l. How man;- tnlloks would that C([Ual were 32 trucks, and they averaged about a c1bic yard n1>emt ;};) o•vt.-it IYflo 1101 our ens tom to take ~~ mnch as that.

U:l64. Al'e you working that line I left ?.'Ir. Robl>'s service some time ago. 1 SG5. wn;; n. lx·eak there iu eonncxion with that particuiar contract ?-It w:l.S

cconomiePI in certain ways. lt JJot if tltJ rnn was very long; if yon had to lead the stuff a couple (<r tl!rce J:Jiles, t 1tea the broa<l gn11.~e wrml,l he cheaper to work, because the engines could take more. Tho dUfm·o:Jec iu t:w eost of bauh['·o there when wo had to run a lon" distance WitS about us one

• • c~ 0

to four-tho cngmcs could take much more on the uroa.d gauge. 18GG. Do you know the Walhnlln <li~trict ?-Ye~. 18G7. Have you been surveyiug tlturo ?-Xo. I 8Gi:l. Are von aware there is a ~urYCY made there for a narr0w and n broad gauge ?-I have seen

the estimn tcs. · · l8G9. Could give an opinion now how mnch the hron<l gauge should cost there, and how mllch

the .narrow went np tlwro to seo the for that pmpo.ie two or three years ago-I thought I m1ght hen a private companv to cm1struct atH1 I thought uhout £2,500 a mile from Moe to Watrwlla for the narrow gauge, lmt I would not care to estimate what the broad-gauge line would be-I shoul<l think .£LO,OOO or £12,:>00 '"mile ali thron6h. The Engineer-in-Chief estimates the narrow­gauge at £5,894.

lfl70. In the face of that, nro yon prepare<! to S[cy that you could construct n line for half the amount ?-This appears to be for the last eight miles.

1871. By Jh.lfarris.-That is the mo:it diflicnlt part of the whole line?-Ye.s, I think that is excessive for the unrrow gauge.

1872. B!J tl(e Cha·i1·man.- \Vbnt is tho difference 1-I should think that eight miles ought to be done for under £3,000 a mile.

187.3. That wonld lJo a saving of over £2,tYlO n mile ?-Yes. I haYe not made a survey-that ia only a lrayeJling opinion. . 1H7·1. Tlwt line was snrvoyml for the purpose of ascertaining what would be the cost, and whether tt would be a good IJOlicy to give that country a uaJTOW gauge, and we are anxious to know the opinion of

l 2.

H. W. Hargrave, 14th August, 1895. 116

a competent engineer to show whether the Engineer-in-Chief's estimate is too high. Are you satisfied, approximately, to the best of your belief, that it could lJe done under .£3.000?-Yes, I think it could. I notice a lot of expensive equipment that I wonld certainly not go in for for a pioneer narrow-gauge line.

1875. By the Hon. D. Melville.-What experience have you had in the construction and working of narrow-gauge lines ?-On the 3ft. 6in. gauge?

1876. I mean on a 2-ft. or in connexion with surveys of a 2-ft. line ?-Thirty miles of survey in South Australia, and in working at the \Yest :Mtllbourne Dock for three years.

1877. Did you see anything in your working in that large contract of RolJlJ and Co.'s that would induce you to pass an opinion as to its practicability in spurs from the main lines in Victoria, say, 20, 30, and 40 miles ?-I think it would be the very thing.

1878. Could you give the Committee any idea what it cost Mr. RolJb to put down this 2-ft. gauge line with the 20-lb. rail ?-The rails and fastenings wouhl cost about £220 :1 mile.

1879. What would the 4ft. 6in. sleepers cost ?-Say, Is. each ; then there would be the fixing and laying. I should think the way we did it, similar to the dock, with light rails, it would cost £300; that is, without forming and without ballast-just laid on the surface.

1880. How did the surface al0ne work; had you many accidents ?-No, very few. The trucks somatimes came off, but that was more because they were triangular in shape and used to rock and topple off.

1881. How many men had you on the line altogether?-At the most 600. 1882. There must have been tremendous traffic on those rails?-Yes; we removed with those little

waggons 1,800,000 tons. 1883. At what cost per mile did you run that ?-It would not be fair to take our running as

compared with a running railway. Our longest run was about a mile; and if we had gone ten miles it would have been so mnch less.

1884. What would the haulage, with this kind of engine, amount to per cubic yard on ten miles?­My estimate down f,here was 2d. a mile ; fer ten miles it would be less in proportion per mile.

1885. Once loaded you can work with that engine at 2d. a mile on short distances?-Yes; with a long distance it would be somewhat less per mile.

1886. Supposing the Committee desire to construct a railway from Walhalla to Moondara-you have looked over that country with an eye to business ?-Yes.

1887. Could you, from the figures you have, form any estimate as to what you could run a 2-ft. railway for in such difficult country as that per mile-I mean the cost of construction?-! said about £3,000 a mile.

1888. Supposing that we have railway iron suitable-50-lb. rails ; what would a 2-ft. line cost in a comparatively level country to put down-the cheapest line in a comparatively level country, and without ballast ?-If the works were all surface forming and were under a foot, either the cuttinga or embanK~ ments--

1889. Give yourself 2 or 3 feet ?-Then you must exceed a foot the line itself would cost about l2s. 6d. a ditches; say l5s. a chain, which would cover the cost of that; and the sleepers.

into quantities. If the works did not forming, and you might require side

and then there would be the platelaying

1890. Say, sleepers l s. each ?-That would be £88 a mile for the sleepers ; laying the plates, Is. a yard, another £100 ; that would come to about £300 a mile ; but you would never get any length where the works would be under a foot. This is just the forming price as if you laid the rails down on the road.

1891. Do you know the Gembrook country ?-A little. 1892. If you constructed a twenty-mile line there, what kind would you make it ?-I think a

narrow gauge, with the traffic that wonld be likely to be at Gembrook. 1893. What kind of country did you survey for the 3ft. 6in. line ?-It was very mountainous,

as difficult as 'Valhalla-from Launceston to Scotsdale, Tasmania. The line is working there now ; the hills were not so steep as at W alhalla, but there were more short spurs-the sidling was quite as steep in places.

1894. How much a mile <lid that line cost ?-£7,000 a mile ; there were 5-chain curves and 1 in 40 gradients.

1895. Practically where there is not a great deal of traffic you would favour the narrow gauge-which of the narrow gauges do you ?-The 2ft. 6in. ; there is more room for passengers in the carriages and you could design better carriages for them.

1896. Can you suggest anything as to the difference between the two gauges-which do you think generally is the more economical for farming produce, say, and with a poor amount of trnflic on the lines?­I think the 2ft. 6in. would be the best; you can get a more powerful engine on that than you can on the 2-ft.

1897. In your experience in South Australia did you find any difficulty in loading and unloading on the different gauges ?-No; there was an outcry at first, but it is shifted very cheaply now, from 2d. to 6d. a ton, according to the class of goods. I have seen the grading machine;; ; they could be used in light works.

1898. What do you think the sidings can be done for per chain-can they be cheapened by aid of machinery ?-I do not quite understan<l.

1899. I mean making embankments ?-Prices vary according to the class of material to be dealt with. 1900. Is there any recent machinery that will cheapen that work ?-No, not for railway cuttings. 1901. If yon had a big cutting and a lead of a mile, would you put doiVn a 2-ft. gauge or a 2ft. 6in. ?

-I would adopt the 2-ft. gauge for all contractors' work. 1902. Say yon had plenty of potatoes and wheat to carry ?-Without passenger traffic the 2-ft.

gauge would do. H the grades were steep, I would rather have the witler. 1903. Is it practicable to go on a railway system of l in 20 with the 5ft. 3in. gauge ?-I should not

eare to do it. In Tasmania they are constructing a 3ft. 6in. line, the :Mount Lyell, with 1 in 20 and 1 in 16 gradients, and they are going to work this with a rack engine, so they do not think the ordinary adhesive locomotive would take their loadB up. I do not think the 1 in 20 gradients and 5-chain curves would work well at all, and with a 40-lb. rail I think the line would go to pieces in a few months in hilly country.

117 H. W. Ila.rgra:re, 14th August, 1896,

1904. By the Hon. E. Morey.-You say you were the engineer of that Launceston line?-Only on the survey.

1905. If that line had been a 2-ft. gauge what would it have cost ?-I should think about £3,000; but I do not lmow what the length would be. It might he a little longer on the 2-ft. gauge. The rate per mile would be about £3,000. The country is a little rougher than the W alhalla.

1906. You say with a narrow gauge and small truck you hauled for 2d. a mileP-Yes. 1907. You had also a wide gauge in those docks ?-Yes. 1908. What did it cost with the broad gauge ?-It cost about a ld., and then we hauled it double

the distance. 1909. Where was the difference in the wide gauge and the narrow ?-The difference would be in

the weight of the rails and the extra length of the sleepers. I think we had 50-lb. rails. The value of the material and layiug would be about double. We could not run at the same speed, and required two men on a small engine the same as a large one. We had the mateTial to carry, and in that case the broader gauge is the best to adopt.

1910. If it cost 2d. to haul it one mile, how much would it cost to haul it two miles ?-It would be less than 4d. for the two miles-perhaps 3d.

1911. By Jllr. Crat•en.-You just looked at this estimate of cost of l\Ir. Rennick's from lVIoondara to W alhall[t ? -Yes.

1912. That is given as costing £5,000 odd a mile, and you said that it ought to be made for £3,000? -That is what I thought when I was at Walhalla.

1913. I understood you to take Mr. Rennick's figures, and make out thnt it should be much cheaper?-Yes, so I think.

1914. In what particular is it too high ?-There are earthworks, 125,000 cubic yards at 1s. 3d., £7,812 10s. I do not see any reason for that large quantity of stuff, from my knowledge of the country. There are l8G,OOO cnbic yards on the broad gauge at 1 s. 3d. a yard. I think there should be a much greater difference between 2-chain curves and 5-chain curves as regards the earthworks.

1015. As a professional man, do you think it is right, in an off-hand way, without seeing the cross­sections and the longitudinal sections, to make a statement like that ?-I said I gave it as a travelling opinion.

1916. Should such an opinion be hazarded when Mr. Rennick has had a survey made, and has taken the absolute quantities ont ?--Yes, because, perhaps, the man he sent never had any experience in narrow-gauge railways.

1917. Suppose he has lmd experience-you do not know that he has not ?-I have had experience, and I think £5,000 a mile is excessive for that country.

1918. You say that as against the man who has had the estimates taken out ?-I give it as my opinion that that eight miles could be done for £:3,000 a and am prepared to stick to that.

191 9. Would r.here be any material difference hetweon making the line on the sidings as against down in the valleys amongst the rocks ?-I would prefer to be up on the siding.

192D. In that ease, what radius would you adopt for the narrow gauge ?-Two chains, if it were the 2-ft .. gauge.

Hl2l. In that case, you would probably go throngh spurs and cuttings ?-Yes; they would be very short, using 2-chain curves.

1922. You know the tramway there ?-Yes. 1923. Have you had experience of similar work to that where all your work is half cutting and

half embankment ?-Yea, a great deal. 1924. Do you think you could put your narrow··gauge rolling-stock on a line of that description?­

I did not examine the tramway very closely, hut if the top were kept up and the curves were true to radius we could.

Hl2;). Some of the curves are 50 feet-could you run your narrow-gauge stock round those with perfect ~t1fety ?-Yes, at slow speed; four miles an hour.

1926. Ha Ye you had any experience with the narrow gauge worked by horses ?-No. Hl27. What is the difference in cost between horses and a motor with a short lead ?-If the lead

were very short, of eourse horses would be better. By engine traction, taking a 40 hor~e-power locomotive, giving an average of 20 l1orse-power, there would be coals, 5s. 2d.; grease, oil and cleaning, 2s. 6d.; repair,, ::?s. ; wages, fireman and driver, 16s. ; interest and depreciation of euginn, 4s.; making, £1 9s. 8d. }'or horse traction there would be, f,lo•l fort wenty horses, £1 1 Os. ; hamess and shoeirtg, 6s. ; wages (I have put down five driYers for twenty horses), £1 10s.; stable hands, 8s.; interest and depreciation of horses, 5s.; £3 Is. as against £1 9s. 8d. per diem.

1928. You make the ntOtor much cheaper ?-So it is. 1929. I thought you said for !'hort lengths horse-powor would be cheaper ?-This would be for a

long length-for a mile line say. 1930. If the stuff to be shifted did not require the full power of a locomotive, then the horse-power

is cheaper ?-Yes. 1 D3l. l~y the Chaimwn.-Have yon had nny experience in feeding and keeping horses ?-A little. 1932. Can you keep a working hor;;e for ls. 6d. for 24 hours ?-I think you can nowadays. 19::33. By 1lf.r. Burton.-The only item you take exception to here is the earthworks ; and you take

exception to the sum of £9,R;i0 ?-Yes. 19:34. How mnch du you <leduct from that ?-I cannot say. I gave a travelling opinion that in

passing through that eo1mtry I thought a narrow-gauge line could be constructed for £3,000-the earth­works are the principal item in that.

193.3. How much will you deduct from that item ?-That I cannot say. 19.36. You are merely hazarding an opinion without data?-Yes. 1937. \V ould it lm a fair thing then to ask us to deduct 50 per cent. from that ?-From 30 to 50 per

cent. 1938. Do you think a charge for earthworb from 1 s. 6d., 1 s. 3d., and 9d. a yard is excessive for the

character of the country you observed there ?-No, I think the price is about right. 1939. Yon take :m per cent. off the earthworks; what other amount do you take off?-With the

narrow-gauge line, I have taken off the station. :Metalling and gravelling roads in station ground is a small

}L ,V. Hargrave, }4th August, 1895, 118

item, but that would depend on the traffic-I would take that oif. Bridges ·and trestles, I cannot say anything nLout that, hut the amount appears high, because it does n<)t require t;early so strong a bridge for a 10-ton locomotive as for n 40-ton. . 1!140. Unless you measure the bridges, could yon check this e~iimate at all as to the qnantity of

timber. ?-No. I see the length there which seems excessive, and the charge is £2 a foot--I think that is excesstvc for a railway to carry a little engine~

194-l. How much would yon think that is overestimated ?-I cannot state :my amonnt. 1942. By the Cliainnan.- What iB t be usual cost of bridges per running foot ?-They vary according

to span and height, from :303. to almost any price. . _1943. You have seen those gullies aml creeks-would the b1·idges require to be very high ?-They

m1ght m the steep places. I should think they might be constructed for 30s. a foot for the narrow gauge; that would rednce the cost to £13,000 odd from the .£15,000 odd.

1944. By illt·. Burton.-T!w next largest item is rails and fastenings ; do yon think they could be red need, bcaritlp: in mind the clmracter of the country?-Yes, to 20 lbs. or ;10 .lbs.-I do not see any necessitv :for ~ •10-lb. rail.

l94i). \Vonhl you pnt !hem on the snnJC quantity of Blcepers for the lighter rail ?-It would not make much diiTercnce in the sleeper:; ; you would not w:mt lo pnt more siEO]Wrs on than they have allowed there. I wonld not have an expensive station at \Vnlhalla for the narrow gange-I would put that up afterwanls. The station aecommodation is put at £1,000-station nccomrnodation should be charged on the whole line from terminus to termimls ; that should Le spret:d oYer the whole line to Moc, so you could take oiT two-thirds of tho thousand in that case.

1946. You would want an intermediate st::ttion at }f oondara?-Yes ; but a much chm1per one than at the terminals.

1947. Is there anything else you could save-are the l::Lst items for contingencies fair ?-Yes, I think those are fair.

1948. JJy the Ifon. E.11lorep.-\Vhat would he the di!Jerencc in cost ],etween a 20-lb. and a 50-lb. rail ?-It does not alt-ogether vary aeconling to the "'eight, because a 20-lb. is more per ton than a 50-lb.

1949. 'Vhat is the difference of the cost per mile ?-I wonld have to work ont the number of tons of the two.

1950. JJy the Chairman.- \V hat is the estimate per cubic yard of the cnttings there by the cugincer ? -Is. 3d.

1951. Is thateJCcessive?-I do not think it is for that class of country. It appeared to me to be mostly clay sbte.

1952. Do you know what was the cost in the railway in similar country bet\veen ,Jumbunna and Outtrim ?-No.

1953. If it is done there for 7d.?-Perhaps there is no rock there. 195J. JJy the Hou. D. Melm'7le.-Wl1st is the lowest price the contractors pay for that class of

work ; have you not clo1w it for le,;s ?-Y cs, from 3d. to 10s. a yard-it all varies. 19.j5. I saw a hoilcr of 8 tons with twelve horses go over a bridge 600 feet long and 50 feet high

for road purpo:ms above the water, :md the piles between the water aud the top 50 feet, and it was tr·ied when it was built with 1.50 bullocks, and there was no vibration, so it is a strong Lriuge ; how much per foot should a bridge of that description cost ?-I should !m Ye to know what strain it would l1ave to bear. Wh:tt spans hacl it ?

] 956. Seven spanR in the 650 feet; it i,; an ordinary pbin briUge with piles, and it can carry 25 tons ?-Those spans would be over 80 feet. That bridge, to carry a 10-ton locomotive, shonld cost £10 a foot; there rnnst be some mistnkc with 85 feet to a spnn. I could uot say >vhat the bridge would be worth without seeing the design.

1957. By 1'rfr. BHrton.-Would it be safe to run an engine and railway over ?-I would not care to take a locomotiYe over it, without seeing ~w]mt it is like.

1958. JJy the Chninnan.-Therc is a diiTerencc between £10 a foot and £2 ?-Of comse; but I take it a-; an 8;}-ft. spa'u properly trnss.ed.

19.'59. By i1h. c,·ru•en. Take a bridge 1,000 feet long and 40 feet high, on trestles, 18 feet stringers, what wonltl that cost ?-I would rather see the design.

1960. It is simply nestling; all the .epaiu; are 18 feet; it is simply on rough corbels, rough timber-wb11t would you ¥ive that ?-It would run from 30s. to .£3 a foot.

1961. To carry a 10-ton motor, wonld it be possible to build a bridge like that for 7s. 6d. a foot rnn ?-I should think it very cheap indeed.

1962. As to the qmtutity of bridging on tl:at Walhalla line-you know the rocky gulloy, leaving Walhalla ?-Not very well.

1963. It is precipitous rocks. ~Would you prefer to pnt a railway there partly on trestling or cut ont of rock ?-I think I wonld pnt it on 1restliug.

]964. On this estirrmte that Mr. Renniek gives here tho qnanti ty of bridges that you cannot nnder­staud is pnt in to save entting out of rocks-you approve of that method ?-Yes, where the rock would cost more.

1965. You say the ScotsJale line cost £7,000 a mile on the 3ft. Gin. gauge ?-Yes. 1966. vVlmt. would it have cost with a 5ft. 3in. gauge ?-They conld not ha Ye got through the

country at all with the 5ft. 3in. gauge.

DEAR Sm,

The w1:tness ~vithdrew.

[ T/;e witness supplemented kis evidence witlt the following letter;-J Wool Exchange Buildings,

Collins-strect, llolbourno, 23nl August, 1895.

Tlte Chainnan, Standing Committee on Railways, Parliament !louse, ilielbourne.

'Vith reference to the evidence tendered lJy me Lefore the Standing Committee on Raihn~oys, I desire to supplement the flame by a few details on the subject of narrow-gauge railways which may be u~efnl to your Committee, and also explain certain parts of my evi,lenco which may otherwise appear not quite clear.

119 H. W. Hargrave 14th August, 1895.

Advorting to the 2-ft. gauge railways used at the \Vest Melbourne Dock. The lines were first laid with l·i lbs. per rails, but, !o the noft nntmc of the rmHI-hcd in the rlock, lln<l the necessity for running over emhnukme1;! the sle1;pcrs ;;rmk into the ll!IH] unequally, 1:nd ~o cnns<]cl t.!Je crippling of tU;; scctio:J of mil. Hnbsoqueutl.'c' additiomil lines were bi<l with 20 lbs. per pnd rails, an<l this ~cetion qnite c<diieiolll, ];oti! in the ,Jock and upou tl10 recbnwtion banks, io withst.nnd the weight of the 13~-tou locomotin;;; me,.!. L~1.tterly, as the syste111 of liues cxtenr]e,\, 40 lbs. per yard rails were used, not IHlCau~e they were requiretl, but on account of the contractor having a s;uplus ~took of this class of rail on hnnd.

The sharpest curves, ns stated at 2 chains nulins, were adopted on what may be termed the "main lines" leading from the excavations, O\'er which the trains travelled at a speed varying from four to fifteen miles per honr, according to the load taken and the direction of the gradient. Curves, however, ranging from 40 to 70 feet existed in mally in the excanttions where the locomotives were not required to ruu at fn~t

The load taken l!J the 6~-!on engines np grndientR of l in 66~a minimum length of about trucks, wl.ich with their l0ading Cf[llalled (npproxima(eJy) a grOSS weight Ot 30 tons.

The average witlt thi~> loa<! on the np g;·:Hlir!nt >vas nlJout eight lllilcs per hour. The narrow-gauge lines provcrl spod:dly c:onveni('nt on neconnt of tho rapidity with which they

could be lai<ltlowu, the f:1cility of their· tmn~fercnce from place to place as the work progrc~sed, the smu.llueas of the curve mdins workable, the inexpunsil'e maintenance, the cl;eapness of the plant, ant! the snitability of the truck for quick unloading by side tipping.

For leads not exceeding 20 chains the cost of haulage was about equal to the broad gange (5ft. 3iu.) used for auother item of work; but beyond the distance named the haulage was cheaper on the broad gauge.

The material removed from the \Vest Mell>onrne Dock amounted in the aggregate to, approximately, 3,200,000 cubic yards, equal to abont 4,800,000 tons weight. Of this qnnntity ubout l ,800,00:) tons was removed by six 2-ft. gnnge engines nml trucks; the average run from pits to embankments was nearly iO chain~, awl the average train consisted of sixteen trucks, coutniniug about 1 cubic yard each. The six locomotives, therefore, combined, during the three years of their employment on the works, travelled \Yith full loads a tlistance of about 39,400 miles, or, including the return journeys with empty t,rucks, a total di~tancc of 78,800 miles.

The lines were not ballasted in any way ; the foundation of the roads was extremely treacherous, and the rails were always more or less greasy with mud. Notwithstanding all these adver:<e circnmstanees, the narrow-gauge lines conducted the enormous traffic above stated with surprising snccess.

PreviouR to my experience as contractor's engineer at the dock works, I was somewhat sceptical as to the capabilities of snch a narrow gauge as 2 feet for conducting an extensive trnffic, especially as regards the power of the locomotives; but my oh,;ervntions, extending over a perio<1 of three yean,, in eonnexion with the works convinced me l1eyond all doubt that iuRtearl of building expensive milways on the standard gauge of this colony to accommodate sparsely-populated districts, or to open up new country, narrow-gauge lines of 2 feet or 2ft. 6in. shouhl be constructed on account of the enormous saving in first cost, and especially where the traffic for a considerable time would be purely problematical.

With regard to the comparative estimates of the Engineer-in-Chief in connexion with the \Valhalla line, I am of opinion that a pioneer railway of 2 feet or 2ft. 6in. gauge conld be constrneteu between Moe nnd Walhalla for from £2,000 to £2,500 per mile, inclmling the necessar.IJ station buildings ami rolling-stook to work a udfic of JO:l tons per diem from either termini, and that the cost of the last eight miles need not exceeJ £3,000 per mile. I lo:)k at the qnestioll from n contractor's engineer's point of view in making a rot1gh estimate of the cost without survey. Contractors have, almost invariably, in large works to estimate for contingent works in eonnexion with their tenders upon the smallest data-generally an inspection of the locality is the only information to guide them as to the cost of lines to ballast pits which may be several miles from tho main works.

I consider that the estimates for the narrow-gauge lines are unnecessarily loaded, while considerable favour is bestowed on the quantities and prices for the broad-gauge.

A perusal of the letters prefacing the several estimates at once creates the impression that the worst case has been made out for the narrow-gauge, especially where an assnmed paying traffic for a broad-gauge line is taken for a comparison as to working expenses. Where there is sufficient traffic for a standard-gauge rail way, of cour8e it should be constructed, but I understand the question at issue to be broa<l 'L·ersus narrow gauge lines for districts where there would be but little or no traffic at the outset, and for some consider­able time to come.

The eleariug for pioneer narrow-gauge railways conld be limited to thut actually required for the necessary works, including the removal of undoubtedly dangerous trees, without clearing and grubbing an extensive width, a~ if for pnrpose of agriculture, and it might be found desirable to restrict the traffic to hours of daylight in thickly timbered districts. On many macadamised roads carrying an extensive coaching traffic the clearing is often but a chain wide, with immense trees on either side; nevertheless accidents from falling trees are extremely rare.

By a proper survey, on the contour principle, the earthworks can be reduced to a minimum, the introduction of curves of 2 chains radius enabling the works to be brought almost to the limits of those actually required for an ordinary cart-road.

Sleepers, quite suitable for the purpose, can be obtained, as a rule, along the route at a trifling cost. At the outset, ha!f-ronnd sleepers, split from saplings, would be sufficient.

The and culvert~ will, of conrse, vary in priee in proportion to their spnn and height. The rnle in economieal IJri<l)!C building i~ to so arrange the design that the cost of the snperstructnre of one span will not exeec(l the cost of' a pier. \Yheu tlw price of a pier, or trestle, will exceed that of a span, the latt.er shod,) be kllg;theued. The mn'll practice in Vieioria is to ballaot the bridges, which, of comse, involves deeking; nit hough prefemhle, it ls by no uwnnB necessary, and a large item of expen;;c in thi:> respect can bo avoided. For a preccuent we need not go fanlwr than South Australia, where, if uot by far the greater number· oE bl'idgo.s on the 3ft. Gin. gauge liHes are neither decked nor ballasted, tlte beincr secured tlirectly to the girders. "'

The necessary weight of rail for any line will depend on the weight of the locomotive employed.

H. W. Hargrave, 14th August, 1895. 120

It is good practice not to load light rails with more than 1 gross ton for each 10 pounds weight per lineal yard. Assuming the rail to be properly supported on cross sleepers in the usual manner, according to the above rule the various sections of rails mentioned helow will bear the engines as stated:-

Weight of Eng·inc in Tons~ Rail. If lour coupled If six coupled If eight coupled

wheels. \Vheela. wheels. 20 lbs. per lineal yard . . . 8 12 16 30 " " 12 18 24 40 " " 16 24 32 50 , " 20 36 40

It will he seen that an engine of 16 or 24 tons weight could he safely employed on rails of 20 or 30 lbs. per yard respectively. I see no reason a 40-lb. rail should he recommeiHled; if a,•ailable, howeve1·, as second-hand stock-or e>en a heavier is no objection to the extra weight; but the cost of' railway should not be debited with same.

Referrfng generally to the question of grtHlients an<l curves as affecting rhe haulage on any gauge, the effect of gravity is so much per ton, no matte1· what tl1e yauyc ma/1 be; that is to Hay, it woul(l require the. smne power to haul equal loads on similar oH any gange where the line is straight. The resistance offered to trains by curves is in proportion to the radius, and may he reduced or equated to represent gradients on a straight road as under :-·

Radius. Equate<l grade per mile. 1 chain 130 feet or 1 in 41 I~ chains 87 feet or I in 61 2 , 65 feet or 1 in 81 2! , 52 feet or 1 in 1!5 3 , 43 feet or 1 in I22 3§ , 37 feet or l in 142 4 •• 32 feet or 1 in 162 4! ., 29 feet or 1 in 183 5 , 26 feet or 1 in 203

It is obvious, therefore, that in locating a line it is desirable, whcro practicable, to reduce the gradients where sharp curves occur, and so avoid combinations of minimum curves with the mnximum

As an illustration of the combined effects of aml curves on trains, take an instance of a gradient of 1 in 33 occurring on a curve of 2 chains radim; :-

A gradient. of 1 in 33 equals a rise of 160 feet per mile. A cnrve of 2 chains radius is equivalent to a gradient of 65

" Total 225

" or combined, equal to a straight gradient of l in 23~.

The suitability of stock for traversing curves of small radius depends entirely on the length of wheel hase. Carriages and trncks provided with bogie frames and wheels can run round curve~ of a Yery small rauius, even upon the broad gauge, but to shorten the wheel base of 5ft. 3in. gauge locomotives to traverse similar enrves means a serious diminution of their power.

In comparing the resistance offered to trains on curves of similar radius on 2-ft. aml 5ft. 3in. gauge it would be mud1 greater on the broad gauge, by reason of the difference in width of gauge

the wheels of the 5ft. 3in. stock to slip or slide more in proportion than the narrow. With regard to the propo~al to construet 5ft. 3in. gauge lines, adopting 1 in 20 graJients and 5-chain

curves, I don't think the matter is to be taken soriou~ly, e~pecially the snggestell working of the lines with the present rolling-stock and engines. It appears very like the adaptation of the swiich-baek railway for locomotive traction. I may point out thut the Department is re-grading several lines on account of the objection to gradients not to be compared to those of 1 in 20 for steepnes>", and, moreover, 1dthont any such cur'l"es as 5 ehains radius. Let liS assume that a lino was eonstructed as proposed, and tlmt: at some future time the traflic ueeessitated an Improvement in the grading. Take a portion of line, for inotanee, crossing a saddle with an ascending gradient fi'Om either side of 1 in 20, and a length of, my, 40 chains for each gmdient. It is required to re-grade this mile, say, to a gradient of 1 in 66. vVe l1ave a total ritle on the 1 in 20 gradients of 132 feet, and on the 1 in 66 gradients of 40 feot, showing a Jitierence at the saddle of 92 feet, which must be met by either a tunnel from 25 to 30 ehai11s in hmgil1, or an open cutting l mile long and 92 feet deep in the miJdle.

As such an example would frequently orJcnr, it must he apparent that the cost of re-grnding this line would be prohibitive-it wonld mean a new therefore there would be nothing to be gained by constructing such a line on the standard gange-even if it was workable--with a view to subsequent improvement.

On the Great K orthem Rail way in South Australia, 3ft. 6in. gauge, t be portion between Saltia and Quorn is provided with guard rails to most of the sharp curves ( 5 chains). Shortly after the opening of this length for tmffic the locomotives on several occasions left the rails, and the laying of guard-rails was considered absolutely necessary, at the time, as a prevention ngainst accidents. Admitting tbat the Government engines were new, and consequently somewhat rigitl in their working as compared to the coutractors' ballaHt engines, they were specially designed for a gauge of 3ft. 6in., aud for curves of 5 chains radius, and did leave the rails. What may we expect, therefore, if the ordinary 5ft. 3ip. gauge engine, designed for traversing curves of 10 chains radius, is empluyed upon a mountnin railway with curves of half this radius, combined with such gradients as I in 20 ?

I am, yoms obediently, H. W. HAIWHAVE, A.:M.I.C.E.

James Styles, a i\:fember of the Legislative Assembly, sworn and examined. 1967 . ./Jy the Ohairma;n.-Are you an ?-Yes, a civil engineer. 1968. Have you ever had anything to with narrow-gaug·e lines of railway ?-I was on the

3ft. 6in. g~~uge railways in Queensland on two different lines; one rnnning from Ipswich to Toowoomba, and the other from Toowoomba to Dalby. The first is 78 miles long and the other is 51i miles. I was

121 Ju.mes St,yle8, 14th August, 1895.

connected with the construction of them both. I was first maler the contractors as a sub-contractor, and subr:eqnently T hall charge of a portion of the works fot· the contractors, Peto, Rrnsf'ey, and Co. It is very rough country; on the nwiu mnge there arc liiO curve:o; in uineteen mile~, with Reveu tunnel;<.

1969. What >Vas the co~t or tho,.;o lines ?-Thrcl line Ot 78 miles co~t ll million. bnt it i~ t.hrough exceptionally rough country. TlHlt is knowll ns the .Main lhmgc, and there is another range, the Liver­pool Range.

1970. Would it lmvc cost more if it had been a ;'ifr. 3iu. gauge ?--YeH, much more. l!l71. Aud less if it had been 2 feet ?--Y cs, mneh less. Millions of money have been wasted in

Australia, I may say, L!Hl nnd eareless surveying. As you know, it is the usnal thing to smul out yonng men smveying to hrenk thPtn i1t. Then they are taken to the construction work, which nowadays is only routine work. It should be ,iust the otl10r ''"ay Ro far as young engineers are conccmed. It was B1Jhsei]nently found tlmt that raihn1y I refer to, instead ol' being 78 miles, ought to have been G8-the gt':Hles would hnvc hcm1jnRt as easy, awl the cnrvo!' easier-it wonlrl lHlVC heen t.en miles shorter. Speak­ing from mc:mor~·, it cost £13,000 or .£[4,000 n mile. Coming tn :New South \Y<1les, overyLo<ly !mows the Zig- Za!!-it is now diseovercd that they eoul!l haxo got on:r tbo Blm: 'Mountains withoat any such thing. Go to Son1lt Yarm. Tho~· rnn the r~cihn1y into tlw wrong plaec tlwro, nml altererl it aftor. I ~vaK a witness in the nr1Jitration ease of tlte Bmnxholtw' m•tl Ca,dcrton Hailwnv. As it wn,; Jn:,l it was a botch, to some extent, and it was sub..,oquently alkn1<l. The contrnct0t' ma,]e n claim mt•l got it. I 'vas one of hi,; witne~ses. Thc:se me some instances that occnr to me right oif, and I am sure that millions of money have beon wa8te(l through not having the lines smveyed at firc't by men of expericn(•c;, Dam!enong brillge has been nltered twice or thrice because the wator-wav was not ~ui'!ic:ient. 1 have been at work on seven di11crent raih,·ays in Vietoria, one in Sontlt Amnmlia,'aml two in Qneen~laml. I know n cas'l in South Anstralia, wlwrc I n·aB reshlon t. engineer al!tl GoYermuent ngcut. I went over tLe line and ~:tw instances of improper snrveying. These thing~'> ~how where a lot of money is wa,;to!l and additionnl risk and expense rnn in constrnding milways on snrvey~ nmdc J,y incxpcrionecd mcll. A" to the other 50 miles of railwa.v in Quecn~lan'L the. contract price for thnt, originally was :£3,000 a mile, :Hr. Gin. g:nng-e ; hut that wa~ a(•ross t !te Darliug Downs-very level eonntry. 1 have no doubt the i'Htlle thing could be d"ne HO\\" for £2,000, and of course tlwso linos were nry heavily h:tllasted, and laid with -!0-lb. rnil>'. The bo:l!l of the rails on the J'viain Jbnge wns nenrly cnt oll' them on aet·omtt of the sharp cmn;s before the line wns opened. The rolling-stock is now improve,]. I snw a littlo diJicrenc!' in the rolliug-stock when I was j,!Jere seven rears ago; lnll not so nmch impron·meut as I thonght there would l>c. The en1-ducs \VCf•' hig;rer nw1 of ~ dilfrrent trpe; hut the cnrves wmo so sharp on the ":\!:ain ILmge that the l;catl of the rail was in ~some cases cut qnite vertical hy the contractor's eugine:; and trucks. Tlw curves were not 5 l'hainf:. l'eto, Tlrn;,~cy, alll1 Co. luul some men workiug for dwm who knew how to rowHl a ·l-cl:ain cnrvo when it was snppoo<:d to he a .S. A lot of the cmvos \Yct·e 110t ;) ehains.

W72. \Ye want to knO\\' the lliffi:rmwo betwe0n the coct of t.lte 5ft. 3in. gango and the Imow a lot of the ordinary ligl1t coHutry in Victorla-(lo yon know tlw lino from Tallnrook to Yea

1973. \Vhar, line (lo yon know '1-}ly firm constructed the li1te from Seymonr to llenalla, but that is a different eonntry altogether.

19H. Ha\"O von hecn to Walhalla ?-No. 1975. Have you been 011 the hnmchos in Gippslrtnd ?-No. 1976. Do vo;1 lo1ow the countrv from llm·wick to Gembrook ?-Yes, I was eontraetor'~ engi11eer

from Oakleigh to 'Bnnyip. • 1977. \Vhnt ,]jd the Oaldoigh to Btmyip line cost ?-I cannot sny right off-I think about £.5,000 a

mile. Mr. l\1cNcill was the origiual coutractor-it is a pretty heavy line. It is sevcutecn or eighteen years ngo ~hwo it was bnilt, mHll e:cnnot fix the exaet amonut.

HJ78. Ho"' much slwnhl that line cost if it wertl only a 2-ft. guuge 'I-I conltl not g:ve nu opinion that wa.v-no mau's opinim1 g·iven in t ltat w::y wonlrl be worth a rn~h.

HJ79. Do yon know anythiHg ahont the cost of tho narrow gauge ?~No. 1980. B!J the Jlon. D . . J'/eh;i//c.-Conhl you, from any or tho~e lines that you know, form an opinion

whnt reltttiv('h· wonl<l lmvu been tlte co~L on tho"c with the narrow gauge ?-I would uOl like to sav. I wrllrl,] not lii:e to t<:ll t.his Committee nnything I eonl1l Hot sulJStanti;;te. ,. Yon cpcak abonl the mtrro;y nnd broad grm:re. I have no donht knm1· tlmt we lmvo sufficient rolliug-stock to do 800 or 900 miles of railwa..1·--tlmt it; worth thinking we lta.nJ F~eore~ oi' engines 'lying idle :lll(l lots of trucks. If yon break the gauge you will have to have frr~it rolling-;;tock.

1981. By the Cl,ll'irman.-lht iu monntainoun country where there is a fnir pro:;pect of a clte;oply constrnoted railway earning mtoHgh to lt<•IY would yon get thoro if yon conld only fhiHk o[ going with a railway to cost £7,000 or £8,000 woalrl get there in that case with the narrow g:mge, becanse there 11rc places "·here t.ltat coul<l he wnrkod with adnmtago where the ln·uad gnuge conld not be laid, where t.ho cost would be certainly prohibitive for the :Ht. 3in.

1982. Yon do not, hesitate to say that it \\·o11ld be right to eonstrnct. n narrow-ga11ge line beca.use it could be com<tructecl at cm1sidcmhly less-we wnut to know the dii1'ermwo approximately ?-1 shonl.l uot like to sny witltont some information. Th('rc is no question tbat the IHllTO\Y gaugB could be con~tnwtetl considcra hi~· cheaper, cspo<cially i11 HIOllllhrinons ernmtry; lmt hcrCJ awl iu Pnrliameut 1 ohouU examine any proposition to C(mstrnct n mHTow-gnugc railway Yery clusel.v. There conl•l he nothing agaiust ih con;,trneted in nny place where it is ;;ul. lil;ely lobe connectp;[ with the present system. There are Gippslnnd to wl,ich that wonU npply, \\hero it woul<l be suita!J],; to r.pply the narrow gani.":C; bnt wlwlher a 2-ft. is the !Je:;t narrow gauge i~ qnite another question.

198:3. It wonlcl be no me at all nnless it connected with another miiwav ?-Might it noc be to an ontporr. There arc two g·ange~ in South Anstralia, awl they !tavo three iJ< Xew.Zcalan,i, with tlw ros;1lt that seventeen years :v~·o thoy al;andono<l the :ift. 0iu, auJ tho En,ginoer-in-Chiel' of Sontl1 Austmlin. went over an,l purchase<! the whole plnut for a mere song, beeanse hml n ;)[r. :Jin. in hi~ colony. l shonhl not, think it wonl<l cost above 6d. or Hd. a ton to transfer the from one trnck to mwthn. 'Wheat aml those things would not cost mueh 10 transfer.

1984. \Ye have had inl'ormation that it would eost 1.1. ton?-[ sav Gd. ri~rht bnt 1 llo llOt

know that I should care to lift a ton myself f(n· :lcl. or ltl. I thmk it conid ],c d.;no ehonpor than 5d. hy having tho smaller gauge railway l;ighor, and lc't the ;;tufi go in h} :~nvitatiou from the narrow to the broad gauge tmcks.

.James Scyles, 14th August, 1895. 122

1985. By the Hon. D . .hfelville.-They contract at Moreland for 10,000 bags of wheat, bringing it in from the Government" I" trucks; the men run it into the store and unload it at 4d. per ton, and when it goes out they re-load it for 4d.?-I think with any proclncc like that there woulrl not be auy great difficulty, because a very ~hort line of railway would make up for the dificrence-I think less than five miles would do that.

I 986. 'l'he Engineer-in-Chief represents that we can now construct 5ft. 3in. gauge railways running from £1,500 to £2,000 a mile in most places that haYe no railway~, and as we lmve surplus rolling-stock we do not think it advisable to recommend narrow-gange railways which will, they say, cost nearly the same amount. vVhat would you say about that-would you go with the Victorian engineer ?-I think I would. If it is a level country the break of gauge would be a groat nuisance. 'Vhere they have constructed at 7d. per yard for earthworks I wonder whether they debit themselves with r.ll the expenses, or whether we should find that they have done the work for what they state at all, all charges considered.

1987. If we have evidence that we can construct in ordinary country at from £600 to £1,000 a mile, and have to create new rolling-stock suitable to thar, what wonhl you say then ?~I should he inclined in that case to try the narrow gauge. There is one thing, no doubt, the Committee have borne in mind, that money is very cheap. You can borrow it now at :3& per cent. Suppose you save £1,000 a mile, that is only £35 a yenr on that mile. It is worth thinking over that one factor in the calculation of difference in the initial cost is not so important in many eases as the subsequent working expenses. Speaking· of the surveying, I have no douht there are cases where the whole length of a railway lw:; been spoilt by some little bit of Lad surveying. The weakest link of :1 chain regulates the strength of the whole chain. A bungle was made on the first section of the North-Eastern lino, the gradients were ] in 50; and the wcond 1 in 75. To show a bungle may Le made by a clever man-this end of lino where the hmn-y loads lmve to be hauled is where the heavy geadients are.

1988. By Mr. Craven.-IIave you had any experience of the lines to the sugar plantations in Queens­land ?-No.

1989. Although I agree with a great deal that you say about the buugling, you mentioned the South Yarm surveying bungle, hut if you knew as a fact t.hat the surveyor who did it complained about putting it where it was put in the first instance to the Minister, and was told if he did not pnt the line in that posi­tion he would have to make way for some one who would, what would you say ?-I should not blame him at all~I should get rid of the Minister in that case.

1990. As to the openings for water-way for bridges; as you know, the formula is so simple that a man making a bungle of that sort shows that l1e is not fit for his position ?-A man may be thoroughly conversant with all the formnlre and be a professor of engiueeriug and be utterly incompetent when he goes into the bush to design works to carry away water because he does not know the country-! mean men with scientific knowledge from the university and no knowledge of the bush.

1991. Is it not a fact that very often the surveyor is not allowed a free hand ?-No doubt. 1992. Where it costs money in paying wages of himself and men to explore the country thoroughly,

because he is not allowed to do that, he has to do the next best thing, and he gets all the blame ?-Those are not the cases I have in miud. I speak of eases where I know the snrveyor;; have bungled.

199:3. Do you know what gauge was introduced in New Zealand after they sold the 5ft. :>in. gauge plant ?-The 3ft. 6in.

1994. By t!te Chavrman.-I-Iad they as many miles of the 5ft. 3in. gauge in New Zealand as they had of the 3ft. 6in. ?-I cannot say. In South Australia there are about 500 miles of the 5ft. :3iu. gauge, and something like 1,100 miles of the 3ft. 6in.-tho latter is the Continental stamlard line. There is scarcely a position on the construction of rail ways that I have not held, from forenum of workmen for a little while up to being a contractor antl Government engineer-I held an appointment for a little while in Queensland. If anything farther occurs to you to ask me I shall be glad to answer it in writing.

Tlie witness withdretc.

Adjourned to to-morrow, at eleven o'clock.

THURSDAY, 15TH AUGUST, 1895.

1Wembers present:

MR. CAMERON, in the Chair; The Hon. D. Melville, M.L.C. Mr. Burton,

Mr. Craven, Mr. Harris, Mr. Trenwith, Mr. J. S. White.

J. W. Foster Rogers, a Member of the Legislative Assembly, sworn and examined.

The Witness.-I do not wish to detain yon long. At the same time, as you are well aware, you have before you a question involving the expenditure of millions of pound$, and if I give a little longer time to the matter than I desire or intended, I do not think it will be waste. I think, moreover, that when I have concluded yon will not want any more information, in fact, you have got information enough now to come to a rational conclusion. I may state, with regnrd to the valuable information you have received from the Agent-General, that it was a.t my suggestion that the previous Committee sent to the Agent­General to gnther that information. One or two members of the Committee seem to think that the infor­mation read out was news to me; it is not. I had the whole matter under consi!leration long ago, exeept one part. I wouicl point out to you that the difficulty you have to conten<l with is this: there is a struggle going on in this part of the world between the old school and the new in railway construction, and people may be

123 l!""oster Rogers, August, 1895.

biased according as they belong to 011<1 or other of those schools. Personally, I do not think I have any hias in the matter. I fd!Onhllikc to rcrmrk that I make <1· very clenr tlistinetion between opinions and fact. An opiniou may he ntlnahle or i!. may ~:uL l!ltt f:tct is i:1clispntable . .l make that observation in reganl to what I have see.ll reported in dtc Arflus. 1.'hcro is a ;,tate:nent there that I have not had time yet to read tlJrimgh, hut I mw one or rwo in it. Of eonree ihat statement has gone before t.he public ant! it is blinding the pnhlic. There is a staleu;ent nutde Loth in the Age nntl A1·gus that people commonly suppose that the relatiYo cost of the broad awln~.rmw gnnf!OS is ns te~1 io one. I have spoken to a great many people who know something ahollt tho 1Jro:Hi and narrow gtLnges, :wd I never hoard any human being express sneh all or)inion. That is utterly misleadiug; it want~ to make out that the supporters of the narrow are a set of noodles and do not know whnt they are talking about. I venture to say that yon have before you a gentleman, representing the 2-ft. gauge, who is in nll engineering requirements, r.s far as railwny moJters go, to any expert. that yon have iu the colony, wit.h perhaps one excep­tion, and dwt exception is not onr Eug:ueer-in-Chief it is :i\fr. i';Iai~. I hold him to be tile finest engiuem' we have eYer had in Victorin, thougll at the smne he knew little or nothing abont the 2-ft. lines, bnt with regard to the ~lft. Gin. n.llll tlw 5rt. 3in. !;nos lw is full master of them, and understands what he

There is nnotltm· point ilmt I nofice in :Hr. Hc!lnick's eslim::te; he estimates the lines for the light gauge, witb 40-lb. rails. I wonlt! point on;; thn.t the railway which you heard about this morning,

Bnd that (ms doao the paS:iCilt,;m" tra!fic CVCf done in the wodd, that is the milway in raris, hat! onlv a 20-lb. rail.

• 199.5. By 1111-. 'Trenu:illt.-It was only intended to last for six months ?-It would have lasted 30 or forty years in all probabilit.y. When that line was examinetl, after carrying 6,000,000 passengers and

for six: months ami clenriu~~ the rllte of £,10,000 per mile profit, it was fonnd tlmt even where thoro were sharp enryos and steep there was scarcely any wear nnd tear whatever, and that practi-cally it was as good as when it was llown. I think you will find in :Mr. Van do V elde's evidence a large tfnmber of facts which nre utterly imlepcntlent of opinions. It does not matter whM any man's opinion about t hosH things may be when there m·e fact~. The only mistake I know in his evidence wns one which I mpdf pointed out to him. Ilo made the E:~me mi,,tako that I did myself aG first., in mistaking a kilo­metro for a mile ; that wns in regard to the co:,t of the railway over the Himalayas to Darjeeling. The cost of that railway is in the evidence. I think the facts connected with that are so remarkable that no opinion of Jl.lr. Her~uick's ot· anybody el"c can affect them in the slightest degree, but some of the argu­ments haseclnpon those facts are used withont consideration. It is said that that 2-ft. line was eXJJCnsive, ami cost £6,000 a mile; so it did, bnt what snrt of eouutry was it! It was ovor sorneting like 7,000 or 9,000 feet in height, nnd through a o,,;mtry thnt our 5ft. 3in. line conhl not penetrate at all, except it were built ut a cost of something like £60,000 a mile. A firm who did not bnlld the line, but made calculations upon it-that i,; the DecanYille Company-1n·e engineers, and they have a dozen quite as expert us 1Ir. l{enniok, in their own employment, who made 11 calenlat.ion that the metre gange, that is the 3ft. 8!in., wonltl cost £32,000 per mile over those mountatns.

1996. Dcles not it strike yon that jnstify the comments that some people ~ay the narrow gauge cnn he mane at one-tenth the cost the broad gauge myself said in my first evidence that the cost waR from one-third to 011e-tcnth, acconling to the nature of the country; that is, given that the two lines "'ill last eqnnlly long, wllich is a point very mneh to consider. If yo>L build a shoddy line you may expect to get results acconlingly. Hero is another fact that is not an o;,iniolt-the Fn•11ch Government, which h?.s in its own lnwl of scvemJ thonsatHI miles oi' the 2·ft. lines. has decitled, or are reported to have done so by :m engineering pnpt>r which ought to he an authority on rho matter, to build a fmthcr l 0,000 mil0s of the 2-ft. lines in FntE(oO. :\rr. \V at con hn:J the paper, I think. There was nnothcr remark about the ripping np of the nnrrow gauges in Americn. lt is very true that in America there were some narrow-gauge lines taken np, a~ in South. Africa ant! )[ ew Ze:dand they had some broad lines taken In Ronth Afrie:t all ihe raibmys, with one exception, are on rho 3ft. 6in. gauge. They had some on 4ft. 8~in. or stnml:ml gauge, bnt they undid the:-c to hrwo an uniform gange. In New Zealand they take the 3ft. Gin. gauge. In ,Japan they build on the 3ft. 6in. gauge because it will do everything they want, and they are a pmctieal people. There is one point the Committee cannot pay too mnoh attention to-that is in regard to the durability of the sleepers employed in lines. In regard to this matter, yon valne the experienee of experts. There at·e diiferent ways of acquiring informntion-one is by experience, another i3 by other people's experience (which is sometimes better), and people may have nronrate, and certain, and fnll information abont mntters of which thoy ha Ye no experienre. For example, I have no of the North Pole, am! th•: nevertheless I know perfectly \YPll ecrtain sciemific facts nbont nml I lmve just as mneh ontainty abont the North Polo as if 1 had been there myself. I consider myself a Lit of an expert in regard to sleeper~, because I was brought up among them. :My father built three establi;;hments-two in Englan:l, aml one on the Continent-for the purpose of creosoting

Th(lro are, I suppose, ahout 20 N 25 pe>r cent. of the sleepers nsed thronghout the world in rail­ways at present made of metal, u(lt of wowl. Thoro \Yas a statement in the paper the other day in reference to the dnrability of a particular sleeper whieh JHr. Eddy took home with him as a sample, and presented to the International CongroHS sitting in London, and it made n great impression upon them. It had been in use for 27 years, nml was in n, good state of preservation. In Austra1ia we are pnrtienlarly fnvoured with wood suitable for the pnrpose of sleepers, yet we do not know everything about them. \Ve have had some Bleepers here which, after on the ground for four years only, have had to be replaced by others, aud this in a oonutry where we Jmye Yery material indeed. I have read and heard, though I do not know whctber it is correct, that something like 80,000 sleepers had to be taken up on the \V cstern line, which is 110t of ver·y formatio~L T!:ey might 1~ot have been down more than seven or eight years, and the liftiug of those nn<l the cxp:mse of replacing through their not having been preporly uttcnilefl to at is c:nor::10Us ; they eost, ()d. or 5s. eaeh, bec:ides the labour~ Tlwro nrc in Bcl~imn grcnml for ~::; years, nnuer very tmfiic, and are still inn state ol' prescrvatiou; a'Hl !nose arc :,oft wood sleepers, brought from Korway creosoted, wltich proeess gives tltC'lll r. life nf f:·om 15 to 20 ~venrs above the:r natnrallife.

19'11. If,! Jir. Harri.~.-'.\'!l:!~ doe., llmt c:''' '-I c:umut give the eost to vou. for yon: it is v~ry li£!ht. I have seen ship-load after ship-load. of Prussian •

I could it for denlt in that

way. To illu:ot:·ate the extent to which this process is used, I may say that either England or America

J. w. Foster Rogers, 15th August, 1895. 124

not only do they creosote sleepers, hnt they creosote the whole railway car bodily before the trimmings are put on. When the wood and iron structnre is complete they roll it on to a rail inside a boiler. They lock up the boiler, fill it with creosote, and boil it, and aftor the process every atom of that structure that is of wood is pcnneated with the creosote, mlfl t:he durability is wonderfully increased. It is possible that if those sleepers here that were taken np after fonr or five yE>ars had beeu crosoted they might ha,ve lasted 20 or 25 years. I alw w:mt to dntw your notice particularly to the kinfl of sleepers that l\Ir. Van de Veltle spoke of. It is a patent sleeper, and one that Mr. Rcnnick utterly ignore;;; it 1·equire5 but one-seventh of the ballast that onr sleepers require. If you have to take up the railway when the traffic gets too heavy for a 2-ft. line, if it is not worn out you can take it up in sections of Hi and remove it to another part of the country wherR it eau be nHed with ad nmtage. There is no comparison between t,he taking up of one of our railways and tLe t:~kiug np of that. Mr. Van 1le Velde';, evidence is so full of fact,s which are simply indis­puta blc th:tt I merely ch·aw spe~ial attention to them. Nllw I come to another phase of the matter. They Rpcak of: the cost of tmnsferriug ot' re-handling goods ; it is merely beating the air idly like children to talk of the tnm~fer of until you analyze the traffic-it is no u:<c saying it is 2d. a ton, or 6d., or anything else. Take coal and halJaqt, that is one particular kind of lading which can Le (lealt with, and is dealt with, in a pnrticn!ar way, hnt you mnst have particnlar mTnugements for it. One plan is to slide a load into another tmck-for that JOlt must have the two cars of the same size ; there are several other ways. There is another kind ol' traffie, ,:mch as b8.les of >vool, bagR of wheat, and Larrels of any substance. These can he dealt with as they arc 011 some I•'rench and German junctions, where the narrow aud broad gauges meet,

having a ramp to bring one carriag'e up to the level of the other, and then you can hand the things across, or you use a crane, or something of that kind, such as ~ailors unload vessels with. At the same time yon >vant a special arrangement for that also. Then thure is a totally different kind of traffic, such as milk, fruit, fhh, and butter; this kind ol' traliie you cannot transfer hy any of those means which have been mc::ntioned to-day, except putting them in Loxes and lifting thc•m box by box without injury, as they will Hot bear knocking aLout. If you take butter ou a hot clay out of one truck and put it into another, I would not give much for it by tile time it comes to market; the same with fish. All those perishable must be specially provided for. Then again, there is an item which promises to become most important, thnt frozen meat. You cannot i'cml frozen meat from p:.rts of Riverina to :1'1elhourue now, u,s mally of the squatters awl others desire, nor can you erect a refrigemting factory 50 miles the other side of Albmy, simply becauRc when you come to Albury there is a break of gnuge, and you cannot under the prm•ent system tran,<fPr; therefore that traffic does not exist. 'Yhat is done thnt from 50 or 60 miles the other side of Allmry cattle arc •lriven to aYoid trambipment, bccanso they say that from 10 to 15 per cent. of the value of those live ~tock is taken out of them by their having to be yarded and put into the train twice instead of once. The other day one of those herds of bullocks was run over by a train, and it was lnekv tlie train >ms not derailed. Thai: traffic is a deml loss to New l:;onth Wales and Victorian rail­ways, but the Commisf;ioners do !JOt care abont it; they would rather have the colonies separated than united. WhPn yon come to com,irler what that trn.tfie may be in the liltnre, it is very important. There is another kind of traffic that we do Hot hnve to carry, but we may have to do so some 1lny, that is, the r<rtillery, in case of war. vYe might luwe to send onr volunteers, with what little e(1nipments they have, to Sydney to help them, or vice ve1-sa, and if they have to tnkc their gnns with them you will have to make arragements for that. Yon cannot slluut them down into the other ears, nor can you comfortably lift them with a crane without injuring them. Then there nre passengers; of course they can get out. I mysel£ have travelled through .Albnry with some of my fnmily half-a-dozen time~, awl going to Sydney you could not go to sleep until you got past Alhnry; if you di<l, you had to get out and get up in the cold. I ::unsure every one of us would willingly have paid 5s. a head to avoid and I know that some people go to Sydney by sea to avoid it. Then eoming back you have to tnrn out in the early morning, whkh is very unpleasant. All this can he provided for i11 the transfer. The eo,;t of: goods is "tated in some eases at l~d.; some, 2~d.; some say 6(1. per ton; anti in some eYidenee given in Sonth Australia it was given at Is., while others say 1s. 6d.; hut if von take the live stock, ls. ()d. willuot do it. lt is very much more to yard nnd transfer the live stoek; ·'i r yon take 10 or 15 per cent. oH: their value, ls. 6d. a "ton il,; very mnci1 LeJmy the mark. It is only poF:'ib!e to mn ke a rough estimnle, taking the traffic all ronnel ; and certainly that rough estimate will he very moJeratcly estimated i!!!lec>(l if' it were pnt dowu at Gd. n ton, that i,; for one thing only, the la Lour; anll in giving these estimnteB il:nt is tho only item which those gentlemen have reckoned-they have left out the consideration o:' an importfmt itPm, that is, the time lost hoth ways in the delivery of goods, and the lo:'s throuuh the dc1ention or twcb. \Ylwn ;\!r. \Vootlroiie was here he to!J you that reckoned the detention ol' a ~rm·k at 10,;. a day. If you send goods throngh from Albury t~ :Mell.JO!Il'lllJ yon will certainly have a lot of goods detaino1l on both sides, going to and fro, and, reckoning them on that basis, it would Le a very con!li!lcmble lotiS. You may Hlirly double the estimate made for tho time lost, so that ls. a ton for all tmffic, which was sworn to in South Australia a~ the co:>t, is about as near as you C!lll reckon to the real cost of trnmdtipping traffic, all round.

1998. By the lion. D. Mdville.-That means only twelve miles n!lditional railway ?-Mr. Henniek says it is 4~) miles-other~ say it is tW<"Itty miles. I myself suggested it might be about twelve miles, but I think it is uearer twenty, and in perishahle goods you want a quick market. By wh!\t I consider t11e best means of transferring the trains, tile co~t, so far us J. mm reekon at present, would be reducell to ahont one­fiftieth of its pt'el'entiigurc, tlml i'-' to s:1y, ::.bout a fnrtJ1ing n ton-wlwnl say the lJest, I mean my own. I took out the patents for mine after J. h:1d seen all the other8; but there are other fitirly good plnns. Jt is estimated that at Albnry at on tlto twoHt_v-mile bnsil"i addltionnllongth there is a capital lo;;s of £300,GOU. Of course it iH very mnch to the ere,! it of om engineen', who seem to be :-mbjeet to very peculiar influences, that

should go awl make a break of gauge i11 a pJace where tlie common ~ense of everybody tells us lt never sboul,l !Jave ami tlteu, on tlte other hand, that they :d10nld tnrn round and say it is nece;,sary for the trnilie of t:lw country that we ;;houltlnot hnve any other hrcak. Tlwy put it in the wrong place, and will not have it in the right place-that is how they h1cve won for themselves un immortal name.

1999. By Jlr . .I. S. JYhite.-~Onr line was constructed first; why did not New South \V nle~ adopt our gauge ?-~For :c similar rea~on, pclri_H:ps, that o.nr (}overnnwnt llicl not do what the G_overument of New South vYalos fliLI when the lmnks bllod, that 1s, tlmt wh<tt one eolouy does the other will not do. I tlunk you will :dmi~ t.hnt. Won; it neecssnry to go into dRLails I could do so. 1 am going to look at this matter fi·om a statesman':; point of view. \Ve han; a big continent which has a future before it, and that continent

125 J. W. Foster Itogers, l51h AUi:,'1t-;t, 1~95.

has got to be traversed by railways, and I prophesy that the first milway th~Lt croRses it wilt be a 2-ft. line. These things lmvo u com:ne:eial value, and they arc ~nrc, ;,vOJier or later, to find their own lune>l; the expenditure on them i'l czo compnratin•!y light, and they call be nmlin ~o e:1ieient. f am ~me t;hnt if e1·er there is a federal government c~taidi;;l!Gd they will ~ee the !Hhi ability l1eg:inning i:1 a re:bllllable wav; they ran always replace tlwm whon the\' want to. Confiniug nttrst';''es to Victoda, ·se knc :),O~J() mil~,; in e;._istence at present. In the light. of tlw ild'onnatiou tluit we lm\·e abont the perfection lo 'lrhicl1 this 2-ft. lil1e has been carried in France, I Jeol convinced that we ~honhl have snve,l from £] 6,000,000 to £20,000,000 if we ha1l made about a thousand miles of' our on the samo ;.;·nuge as Now S.uath \V nles, and our other 2,000 miles hall been on the 2-ft. gauge. \V e have now ever so many non-paying lines that have cost an euormons amount, and if had boon !milt on a reasonable scale to Ruit ilw Lrdlic that there may be some day we should lmve saved much mom•y.

2000. By ilh. Cnmen.- \V onl(l it not have hee11 better not to have bnilt them at all ?-No. Taking the whole railway system it is one of the moHt valuable thing's we have; it cost ahont £f50,000,000, nml i1:' real value to the country is auont £200,000,000. Take away the railways and the wealth of Victoria would be one-third of what it is. If you construct a railway for a company yon must look to what it b going I c, pay directly; when you constntct one to develop a ycnmg country it i~ a differont matter, aml the imlirect gain to us is very much greater lhan the dircet could be.

200!. Do you make any distinction between leading linos and cockspurs ?-Leading liues should lllwc feeders (I do not call them cockspurs) running lOO miles if necessary. I consi<ler it likely that if this 2-ft. gauge is introduced, in the course of time, within a generation, if we continue to make the progress in the future which '1\'C have done in the pabi, and ·which we are not doing in the present, we shall have something like at least 50 of tho~e 2-ft. gange feeders at an average of 50 miles in lcngtb, that is, 2..')00 miles. That is the point which this Committee really baYe bel'ore them, awl il' they settle fol' the 2-fr. line, th0 probability is that iu the course of thirty ycnrs they >Yill have scllled for :2,Ei00 miles of railway, therefore I ~ny it is a hig question, nnd worth consideration. I should ~ny the ave!·age enpital cost, taking mountainous arHl flat e,>wttry together, would bo about one-third or onc-foldir of tlHc co~<l of the ;"ifL 3in. gauge built equally durably. If the,v arc not durable they are no good, nntl the eo~t of the transfer, according to my notion, which mny or may not Le erroneous, would make tlll addition of 25 miles to that 2,500, 1 reckon the cost of my break of transfer at lmlf-a-rnile f'or cotrstruetion aud eYerytUTrg ebe. I may point 011t tlro enormous advantage would be to fanner~ a;Hl :-<f]ll:ttters--you coni<! tak<: a mnrow-gauge Ji;:e wit,hin reasonable distance o£ many lmudreds of such properties, atHl then yon crmld ndopt t ],e continental Ry,;tem of 1mttiJrg down portable linos so that you could load yom wool or your or nuy-· thinp: else yun had to load in the middle or your own farm, and cend it to the ship'~ sic!o nt Port 11elhonrne or \Vi1liam~town without re-hnndling, so that yon wonld not wal!t ~'om· horsPs anl cnrt:l to drive to the station-you conld use them for oiher purpo:ms. You woul<l simply pnt this lino <lown aml take np yonr crops in the centre of your own farm :m<l send it to the ship, 200 or 300 mile~, withont any re-lmudling or waste or damage, all of which has to be considered when ypu nre tran~fening even iu the mo~' ~kiHul manner. I mn qnite sure yon have ample evidenee, if it is only digested and analyzeil, to show that there are absolutely indisputable frwts to prm·e th:: snperiority of n 2-ft. line over mly otlH'r gauge for onr uecds. I wns for a long time in favour oi' the 3-fL ganrre, awl nfterwanh; of the 2ft. 6in. gauge, but that w:ts only until I began to hmm more fully and realize better what I had myself seen. \Vhen .ron come to examine that 2-ft. lino yon flnd yon eau comfortaUy put on those cncrb, ;J() feet long anti (j feet wide, anything yon want ; they nro of capacity t!mu o;;r prellcnt and you can sc,rd thom through with the grer\tcst far:ility and greater snfety tlnw on our present imt8mneh as the centre of gravity is nearer the oarth, so that there is leRs linbility to npset. That w:Jo~ a point wlrieh Pro­fessor Kernot did not seem to be aware of, and although every engineer should know it, he imngiue<l that those little trains would be blown away by the wind.

2002. By the lion. D. Jl1elville.-How does it effect the height of the train ?-The train i.s not so high.

2003. How would it aifect the loading of chrdf?-You must not take it np too high. 2004. How would it afioct the narrow gnnges iu a gale ol' wind ?-The win<l may acquire force

enough to blow away any trains, and hns done so on the broad gauge, but I never beard of a mnn in Gippslaml being blown away in his buggy, and that uoes not enter into Olir calculations any more than the occurrence of an eartbrtuake would.

2005. JJy the Cha1irman.-Ii' it is necessary to c~rry chaff on the narrow truck,, they would have to be extraonlinarily long ?-20 feet long and 5 feet wide.

2006, How many tons wouhl you put on a tmck ?-Th:1t depends. \Vith the bogie stock yon can have a 3-ton truck to carry 10 tons load. That is a very imporJ,aut point in coHnexion with :Mr. l~ennick's evidence; he :1sscrts that the haulage on a 2-ft. line is more expensive than on a f5ft. :3in. line. On a 5ft. 3in. line wit:r onr stock it i'! very much dearer th::u a 2-ft,. line with the bogie stock. If we had the bogie stoek in nse there wonld he very much less dead weight. The haulage on our liues properly equipped woul(l be very nmch less than on (1ur lines as 1hey are. In America they make <t 10-ton trnck carry 40 tons on the bogie principle, but om stoek is 110t on the bogie principle for the most part. \Ve have a 6-ton truek to carry 10 tons, but on a 2-ft. line you can have a car weighing 3 tons and carrying 10 tons, and in India on 2ft. 6in. railways they have them less than that. Even in regard to passengers, you can have a passenger car that will weigh only :3 tons to carry 36 people eomf()rtably, whereas we have a 20-ton car to carry 72 people.

2007. By Jlf1·. J. ,S'. TVhite.-Yon think we should not cm:~trnct any more truck::~ on the present pattern? -No. That is one of the mistakes our engineers are rospousild.e for; even 'Mr. Tl1omas Higinbotlwm, who was a. British engineer, was nut acqnainted with some Amc•ricar1 idea~. I may ]JOiut out as to tho"e two enl!ineering invention:;, which are of the highest consequence to rnilwny nntttcrs, that the bogie was invented in vYale;', and the 2-ft. gan~~e liJle was estal,Ji~hed in -\Vales, yet the Briti~h engineers took 110

notice of them whatever, An Amcricnn saw the bo;,;ie, took Jt over to Amuricn, mill perfecr.ed it; a Frenchmen ~nw the 2-ft. lino and went l•aek allll perfected it, and made a fortnue out of it, Lnt many British enfrineerR are too ;;tiffneelwd ro lc:1rn anything out of tlH;ir beat.

2008. By the llou. D. Jld,ville.-Nlr. Hennick's estimates are iu to-Jay's Argus. It say;-" First-class light railways (on the present gauge), £1,525 per mile: second-clttss light milways ( ou the

,J. W. Foster Rogers, 15th August, 1895. 126

present gauge), £I ,265 ; third-cla~s, £965 ; 2-ft. gauge, £733. Mr. Hogers and his friends are decei vcd by their :tdvisers qnoting a cheaply-built nnrrow-gangc line ~~s a well-bu:it light line, but if li;.;ht rails :md light cfl.rthworks "''" nse<l in each insti1n(:e the tlit1'ercacc b eost. bq.;-ely dis:tppear.o." A'i~nming those figures arc correc:t, :u:d 1llat W•3 lnn·e an cnormons of w:n~el! nwl surplus rclling-stock; with all these dmwlmcks th:1.t von have 'Jointc.I o::t mornin'r, whid1 of the two railwavs \Yonld vou bniltl, the 2-ft. or the ln·;;,J ::·nug..J-whiclJ ~nmld be the more eco~omical ?-If that were c~rrect I tl{iuk I shonld be inclinetl to take the :l-ft., bocaasc yon \\'0:11d hrwa loss dcrul weight and you would Cl•us:mJG less coal, and there wonla bo less wear and tear of hoth locomotives and lines ; and it is better adapted to light traffic, because it is better to have small trains fre:1uently tbn to have heavy trains at long periotls for such traiJic.

2009. By the Chairman.-For the third-class lJroad-gange line ihe cost is £965 per mile, and for the 2-ft. gauge liue it is £7:33 per mile ?-What is the good of builuing a third-class line. :Mr. Reuniek gives three classes for the bro:td gauge and oae for the 11:1lTOW ; what does he mean?

2010. Taking a third-class line and a JlaJTO\Y-g-au;re, there would be a saving of 4s. 3cl. in every £1-why do they not con:-;trnct the thinl-class railways ?-They would be shoddy railways.

2011. Mr. Hc:mick's figures are £1,525 for a first-elass line, but that line is not half complete?­Just so; there is 1io eo m parirwn.

2012. He hns lately raisccl his price for those lines from .£1,800 to £2,300 per mile ?-I may tell you that there me experts in other countries besides this. There are experts in India, where they hnYe a much more varied experience in rail way construction than we have, and several of the Indian experts point out that whenever they have bnilt a llroad line cheap, that line has invariably proved a failure, because whenever the trafflc grew heavy it had practically to be rebuilt-it was a shoddy railway.

2013. :M:r. Rennick estimates the llroad·gauge at £1,525 and the 2-ft. line at £733, the latter being a little more than 50 per cent. clwaper than the other ?-Yes.

Tile witness withdrew.

Adjourned.

TUESDAY, 20TH AUGUST, 1895.

Llfembers present:

Mu. CA~rERON, in the Chair;

The Hon. J. Buchanan, 1\LL.C., :Mr. Burton, The Hon. D. Melville, :M.L.C.; Mr. Craven, The Hon. E. Morey, M.LC. Mr. Harris,

Mr. J. S. White.

Frederick Upham, sworn and exmnined.

2014. By the Chairman.-What are you ?-An engineer and designer of machinery, locomotives, nnd rolling-stock. I am an American, and in America I was cngagcrl for a number of years in loc01;notivc building :tnd construction, and I had a cousirlerablo share in the equipment of the elevated railways of New York. I have also a literary knowledge of' tl:c qno~tion of railways in other countries. I have traYcllcd throughout the United States and Ca:mdn extensively. As to narrow-gauge lines, I put great emphasis, first, on the question of abundant steam power; and socom!, groat tractive power. I emph:tsize also the value of having Ycry light rolling-stock, consistent with strength.

20!5. Is there :my distinction a~ to tlto cost of haulage between ln·oa1l and narrow lines ?-There is. The l'eason why lmul:tge can be m:ule much cheaper on narrow-gauge lines is almost~ntirely owing to one thing, viz., that grcrtt friction aHJ loss of tractive powe:· arc cmisoJ in going round curves, and, while the curves may be of the same radius in both g:mges, tllC slip of the oulsido rnil is greatly lessencLl in the narrow-gn.nge lines. As an illqstmtion: \Vith a 5-ft. gnuge line the uistanc'tl in a circle of the outside rail would be longer than the inside, and would be approxinudely 32 feet ; and in a 2-ft.. gnugc line the same slip of the wheels wonhl he only 3G por eent., or ,,omething like 12 feet. The slip is t!:c uetrimenktl force in the questiqn of haulage, nud. as usual it takes large engines to do a comparatively small amount of work. On tho other hand, in a narrow-gauge service the amount of steam applied is far less, because of the detrimental force heing leBs.

2016. What is .the percentage of the dead lwulagc ?-In the present system, speaking approximately, the dmtd haulage and the paying lutulage are about equal; 100 tons of rolling-stock carries lOO tons paying haulage. \Vith fairly w-elt designed 2-ft. gauge lines this cnn be pnt :v;; l to 2~ ia favour of narrow-gauge linos as compared with what can be done with the ~ame horse-power on broatl linos.

2017. Is it a fnct that narrow-gauge lines !•re unstable, and that the oscillation is dangerous in going round curves7_:_It is not a fact; the contrary is the truth. This question has been greatly dell<1tetl, and tho truth of the mntter can be explained very briefly. By the cm·ious law of oscillation t!10 nflrrow­gango carriugos and trucks tent! to roll over on to the tops of the rails instead of against the flanges. '.he rolling of wheelil ag:tinst the lianges is a dangerous factor, but in the oscillation of nmTow-gangc lines this roll is taken Ly the tops of the rails, where no injury can ensue, and recently the ::H1vance in thcl science of finding tho centre of gravity hns been of ~uch :1 e!mmcter that tho roll is now reduced to a minimum.

2018. You are a\Yarc that it is accepted as a fact that narrow-gango liaes eau only go at a very slow pace'?-Yes. 1 hnve gone to exceptional care to controvert that. In the London Engineer of Fobrnary, two yenrs ago, there is n earefnlly written report of an ul!ieial trip on the I'lcw Zonlnnd railways, whieh are :3ft. Gin. lines. A. trip wn,s malle of Gl miles an hour between .iVIinnwatu aml '1Yellingtou. \Vith loco­motives designed for the purpasc, speccl can be made upon narrow-gani!:c liues to any speed required, with safety strictly maintaineJ. Seven years ago I patented a locomotive in this colony specinlly designed for

127 Frederick l'phnm, 20th August, 1895.

high speeu for narrow-grruge linea. I have a dmwing of it here, but I have not done anything with it owin,!!; to t!1e inertia of tho Hnilwny Department at tllc t.ime. The pntent wa~ taken out without any spccin\ rc·forcnco to the gr.ngo. It merely iltYolYcLl tl1e ptiw:iplc, bnt the dr:twingo I b:tve HO\i'

were pn•pare•! for a 2-ft. ,-y,;tem. 2019. \Yhat woul<l it lmni on n, k:~·el ?-Tile may br·, on a 2-ft. lim', as as ·100 tous

with perfect ancl over ordinarily twt·J~. g'l'eatly fallB oH' :tci Oto gradients uro approaehotl. Of eourse, in practice, no roads are built. hanl::tge woulrl fall off in a gradient of 1 in 50 ouc-!wlf; tlutt woultl mnke it ~00 tono. That id speaking of the maximum power that conld be used on the 2-ft. lines.

2020. Hun: yon had any experience in the transhipping of cargo from oue gauge to another?-­Yes; while going through Canada antl the Unit.ed States I cxmniued the system there. There are 7,000,000 ton.'l of cargo earrioLI across on trucks, one truck on top or the other, oil the St. Clair Hiver, antl on ferry­boats, thro{tgh the break of gauge. This is a sketeh showing the i•lon-[Blwwing a sketch]. That hus been in tv'e er er Hi nee I wnR n boy. The l' nited States Govemment Aeml po.~tal matter d.ireet through from Nmv York over two different gauges, aiJtl it nms through to Bm1ton and the castem cities. The rcawn it was r.doptoil for that was on aceonllt of possiiJb theft not to lJrcak it. 1 ean remember its 11sed for 30 ycar~-[e.rplatnln.IJ on the ..

2021. Ry tlte Ilon. /J. ilfelville.--H,nv about g-etting umlcr bridges ?-The Harrow-gnngc t.rucb go on the otheri', and do uot go higher than 12 feet. The smoke-iit.aclc of an engine i~ 13 feet frequently. The first trud: is 4 feet anrl the next one 8 feet. That ta.kt!s tl10 ordinary size of narrow-g:wgc trnek­[sluncinp on t!w That is adapted more for fish and cold ~tomge, not for such as clmil' nnrl heavy

2022. B.'/ the Chairmrm.- \Yhat weight of raila are used ?-For any traflic that is lik•:ly to be used in Victoria I think 20-lb. rails arc amply suiJicieut. Of course, in making this recommemlation, 1make it upon the lm:-is of the ~y;;t.em of locomoth'o constrnetion >vhich I have <lrawn in which the driving wheels arc nmlt.iplie,J. 1 nse five driving wheels oa enclt side, and which the s:trne tractive force without the ,;trnin being concentrated. 1 nm snrc tlHtt 20-lb. rails woulrl be ,,uf!icicnt for :my trai!ic that woulrl arise. Incidentally, I mny sny, it will tlcsirablc to use Logic trucks on all nmTow-gauge lines, fill· the reason that arc lighter in haulage awl more pla;;tic iu turning short curn'B.

202:3. How would the he pbcGll ?~1,760 to the mile. The placing of the sleepers :tt wider clistanee~ than the usnal to elasticity, aatl in light lines thLs is brought ahont by the use of a largo nnmbcr of \\'heels. \Yith a smnll number of wheels the rails would be hammered and broken down.

2024. How long hr,vo vou l!cen in thi~ country ?-Twelve years. 202.5. Do you know th~ country pretty well ?-:__Yes ; I hn;e n very fair knowledge of this country,

excepting the \Vostem dbrriet. 202G. Do you know the Gipp;;bnrl country I know it well. I h:we been through that country.

The last time 1 was in that di"trict I \Vll:i at Glen Wills. 2027. Snpposing that yon wore asked to con~trnet :1 line of 40 or 50 miles tht·ough those mountains,

which gtmge would you adopt ?-I can speak nuhesitatingly-the narrow-gange would have very many adYantagcs.

2028. II~n·e yon m1y knowl(~dgo of the cost of construction ?-I could not spenk on that with tho accnral'y with which I haYe spoken oH the other questious, but it i8 snfe to say they ean he mnde for £1,000 a mile. That is cxclu~ive of equipment.

2029. \V!mt woul•l the othcL' r;augo co;;t in the country where .£1,0~)0 would L>c enough for the narrow gauge ?-1 do not think it i" p ls,-ible tlmt the cuttings could be mnclc fnr auythiug less tlmn £3,000 a mile.

2030. S11ppooiHg it. wcro not possible to do it for less than £f:,000, how mneh \Yonl•l the nnnow gauge cost ?-The proportion of ono to three hn taken liS a fait· estimato of the cost of the two lines.

20:!1. Do you eay that from kuowlcclge woul,lnot like to be po~itivc of that. I haye it a. good <!Pal of inve.5tigatio11. I cm1~trnetd 300 goods wap:gous for the Rnilway Department a few y.ears ago, mul in tllCl constmction or rolliug-~tock for Victoria 1 have a knowledge of cost:'!, hut as to the cost of the lines I cannot spPak with the nuthority I would like.

20:32. Ilnvo you any knowledge of con"truction in America ?-Yes. In Amcricn the hrond-gangc linos are coll~trudc(l very ciwnply, so tlt:tt the gain on tlEJ narrow-gauge lines is not so apparent.. They constfllct the wide·gango linos so cheaply that the distinction is not ::;o pronomH~e,J.

20:18. In a case, in level eonntl'y, where the hroad gange costs only £1,500 a mile, how nmeh less woulrl the narrow cost '1-In that <'a:'e the difference is not nearly so apparent. In speaking of the figmes, I had reference to the tli!Iorence of co"t in rails am] cuttings. 1 think in ttmt ease the difference would not be an1·thing like as much. Oue might be done for £1,500 anrl the othet· for .£800.

20:l4. \Youlrl there be only £:200 11ifferonce in eonstructiug a 11arrow gauge in rough country and level conntry; ;·m1 say in Gipp':!iand for £1,000 aml in the mallee you ~:cy £800 for the narrO\Y ga.ngc? - Yon are overlooking the imporlant fact that this snm I eslimate (:£800 per mile) is not all material. A large portion of it was for gmdiug. Ahont £·100 of that, one-half of that, is for gradin,g·. The material Rhould be got for £:300 or £400 a mile for the narrow gnn~c, o,;o I take into neeonnt the difference between the material and the estimate;] priee as graded, and if yon add £200 for the other lines I think the m:tinmte hohls goorl.

20:.l5. Are yon acquainted with the qnanlity of goods hauled over the American lines, broad and narrow?-Yes.

20:36. lf you had trunk !incH to construct in this country, 200 or 300 miles, would you recommend for ordinary traffic the narrow ga11ge instead of the lmmd ?-I should, unhesitatingly. I have already done so thro:1gb the pupors. :Niy reasotJS wonlJ he bttSe(l on the general evidence I have g'iven, coupled with the fact that with more roasonal>le con,.;truction of rolling·stock nnd engines we woulll be able to carry our

a]](l passengers so much cheaper. I am eolll·inced th:tt if one-fourth of the scicrwc wore put into tlto con;;trnction of rolling--stock that i~ pnt into the con>4ruction of a hieycle wo eonld carry onr goods four miles for 1d. n ton. There lms heen r.!most r.n absence of scientifle eonf>truction of locomotives in this colony. 1 may mention one thiHg that ha-; tended t:o cost in hanlage; the introdnction of eomcr buffers lws made it imperative to make all the under-frames and works of the carriages so very strot<g to

Frederick Upbam, 20th Angusr_., 1395. 128

stand corner strains that the dead haulage is something enormous. On the other lmud, with narrow-gauge haulage the stn,ins are concontmted through the ecntrcs of the carriages, and all the rost of the vehicle is comparatively light. That reduces the tkntl haulage to a minimum. Tbc modem scl1ool ol' engineering believe in making things !iglu ami cJtron!r, in .listinotion to heavy and strong, awl are going in for hollow stool axles. They are being introduced iato Germany nn(l .France. By the~ introduetiou of those lighter principles in designing you might be easily able with a narrow-gauge lino to carry l ton four miles for a penny.

20:37. Would those hollow :txles be safe ?-I am certain that they are safer than the others, because in the working of all materin.l tlw inner portion bcJeomc:s crystalline and of no strength, so that a tube is much stronger than the rolled ::md test.~ in Germany luwe proved t.hat they are far stronger than the solid ones.

2038. Have you any experience of rough vehicles, such as waggons or drays; you know the axles they nsc ?-Yes.

20;39. \Vonld a great thick piece of iron, say, 6 x 3 be stronger tlmu lmlf that ~izc encased in wood ?-Special conditions govom it. If it wore circular the strength would be very great. Or.her shapes might be corrcspondiugly weak, bnt bronJly speaking, whore the metal is (lisplnccd in t.ho eentre it is \"cry much stronger in proportion to its weight. Tltis fnct is brought out in a pronounced nmnner in bicycles. They are muuufactur0•l hollow, alHI :strength is phenomenal.

2040. By the Hon. D .. Melville.~Wlmt are the: trucks without thnt ?-The trucks weigh 6 tons to 10 ton haulage. In reality the deml haulage is more like lf>O to lOO payable; I was gidng them the advantage of that.

2041. You say you can construct at a proportion of 1 to 2~; is that including engines ?-Yes. 2042. Then ;dmt will a truck weigh to enrry 10 tom ?-.Not to exceed 2 tons. 2043. Yon say yon c:tn coustruct a certain truck on the narrow gauge ; could you do that on the

broad gauge ?-The circumstances twe not oo tiJvornble. 20,!4. You tell the Committee that. on tlw uarrow gauge your truck to carry 10 tons \l'ill only weigh

2 tons ?-Yes. 2045. At preFent H weighs Gt tons '1-Yes. 2046. 5tons 4±cwt. carrio8 8 tons ; 5tons l9cwt. 3qrs. carries 10 r.ons ?-Yes. 2047. You say on a nalTtJW gauge yon can have 2-ton trncks to carry 10 tons ?-Yo>. 2048. Couhl you, if you were enllsd on to design, make wlwt you say ?-1 conlLl. 2049. Aml produce a truck, such as the "I'' truck, whose weigi1t would be loss thnn 2 tons. You

woultl gunmntee that it would bear all the wcr~r nnd tear and strain and carry 10 tous ?-I will enter into an agreement the moment you are ready.

20i>O. If your trnck weighed 4 tons ?-The principle wou Id hold good in the bogie tr,Jck, that it would carry 20 tons, ami the larger the bulk there is a! ways 10 chance to lessen the weights of pnrts that is not npparent in small haulage.

2051. By the Hon. E. ilfore:'!·-It is not nsnal to ittkc 20 tons in a truck ?-Only in fi lo11g gondola truck. I was saying that our Victori:m rail ways carry approximately ton for ton, including engine, whereas a bicycle will carry nine times it~ own weight, showing what the new school of engineering will do. Thnt shows a deficieney of skill some\dwre.

2052. Can you giYo the weight of :tn ordinary bicycle ? Twenty-eight pounds, and they have been tested to carry nine times theit· weight. Thc,v are made much lighter than that ; that is the average.

2053. A bicycle does not get so much rough usage?-Yes; a bicycle goes over a rough road, and the other goes on a rail ; you are reversing the fact~.

2054. By the Hon. D. Jlfeh•ille.-You say there is no science introduced in our rolling-stock?­I speak in a compr1rativo sense; little or none, I would ~ay.

2055. What is the cost of an ordinary " '" trnck ?-I have tendered for the construction of trucks with a part of the fitting::; furnished. I do not know the cost of those fittings. I think I am snfe in saying they cost £140 completed.

2056. It will carry what ?-Ten ton~. 2057. Your truck will carry 10 tons, weighing 2 tons; what would it be made of ?-Steel and

wood. 2058. What would the cost· to the Department be?-Would you include, as iu the other, the

Westinghousc bmke? 20;)9. Yes, everything necessary to the proper working and equipment ?-Not to exceed £10 per

ton, or £100 for 10 tolls; awl in that answer the cost of the lVestiughouse brake woulLl t•emain the same in both cases. It coultl be made .for less now.

2060. Then, as far as ihe rolling-stock goes, there would not, be much difference. Have yon seem this class of rolling-stock working anywhere?-Yes, many times. The Atldnson, Topeka, aml Santa. l<'e Itailway line in America, 1,300 miles long, is operated entirely by trucks of this clmmeter.

2061. Have yon any standard in the 2-ft. gauge showing what tlw floor ~pace of the truck wonid be ?-The snme as the other, but merely carried on more wheel~.

2062. What width would it be ?-I shonld reduce it from 7!; to 61, reet, uot more than that. 2063. Is there any scientific rule abont that ?-1'\o. The later investigation as to the centre of

gravity permits almost as n"ille trucks to Le used ao formerly; iu fact, there is no diffienlty in nsing them jnst as wide. A ne\\' principle of mechanics ha;; grown up ~ince the knowleclge of the cantaliver bridges, so that the danger is only apparent., not real, in running very wiclo trnck8 on the narrow-gauge lines.

2064. Do you know any line that you can point to that irwreases the width of the floor more than Victoritt does proportionately ?--Ye;;. Proportionately, all the mtrrow-gange liues have floor widths f:tr in exeess. It is qnito common for the floor wi,lth to be three times the gnuge. The milwa_v 1 mentioned has tlmt. The lmli:tll 2-rt. railways all have a floor space three times the gunge. ~\lost of the gauge in India is ;39 inches, and tho floor qmeo is two and a lmir times that.

2065. If we applied tlmt to the 5-ft. we wonltl have l 5 feet tloor ?-I know that. 2066. ls there any ru:c ?~Apparently not. The engineers have taken adyantage of the new know~

ledge ou the subject, and made them as wide as they can, consistent, with safety.

129 Frederick Upham, 2oth August, 1895.

2067. Is there anv rule about height on the narrow-gauge lines ?-No, apparently not. I may tell you in connexion with that that the mctacentre is constantly rising in all rolling-stock. On the 4ft. 8,!in. gauge, the most used in America, the height of the centre of gravity of locomotives in the early days ot my experience was 6 feet from the top of the rails. It has been progressively raised to 9ft. 3in., showing that this principle of the high oscillation is not looked on as at all dangerous. As I have said, the oscillation tends to throw on to the top of the rails and not on the flanges.

2068. What is the weight of your engine ?-I would recommend, in order to get adequate steam power, to have the engines very heavy, with the load taken over a large number of driving wheels, so that the load per wheel is not excessive.

2069. How will that stand with the very light rails ?-The load is taken over a large number of wheels, and is not unduly concentrated anywhere.

2070. By the Hon. E. Morey.-What weight of truck and carrying capacity would you recommend on the narrow gauge ?-I would adhere to the 10 tons per truck, but carry on eight wheels instead of four.

2071. We have some 6 tons ?-They are comparatively obsolete ; they are being done away 'vith in favour of the tens.

2072. What would be the grade of the line ?-The advantage of the narrow-gange line is that it can be adapted to any reasonable grade. I have seen locomotives go up as high as 1 in 10 with only a short pinch.

2073. I speak of carrying a fair load ?-Less than I in 20 is not practicable. 2074. Is that perfectly safe in coming down ?-Yes. 2075. With a heavy load ?-Yes. 2076. "What do you consider an engine ought to carry up a grade like that ?-An engine on a

narrow gauge, lOO tons up the steepest grade. 2077. Would it be safe coming down with a load of lOO tons, including the trucks as well, with a

grade of 1 in 20?-Yes, perfectly s!tfe. The Mount Washington Hailway is two or three times as steep as that without mek gearing. I have often recommended larger rails in the curves. I would recommend 36-lb. in that case. It is very desirable to strengthen the curves.

2078. By Mr. Craven.-What would be the weight of your engine ?-It is desirable to run a loco­motive weighing 28 tons. The principle is that the engine should be able to take up the load from a stand on a hill.

2079. That is 28 tons loaded ?-Yes. 2080. You have taken out a patent for your engine? -Yes. 2081. Have you tried to introduce it in any other parts of the world ?-No. I hope the principle

will not be impeached in the slightest when I give the particulars. I took the patent out seven years ago and I let it lapse, and it is common property now, to use as you like.

2082. The Darjeeling line in India, do you know about that?-Yes, it is a 2ft. 6in. line. 2083. All our information is that it is 2 feet ?-It is 2ft. 6in. Your returns are all wrong. 2084. Have you been there ?-No. 208.5. Do you say that in spite of the official papers ?-It is not a point worth fighting over. 2086. Their steepest gradient is l in 28, and the gross load of waggons and freight hauled up an

incline is 3.5 tons?-Yes, it is only a four-wheeled locomotive. 2087. Yours will pull lOO tons up l in 20?-Yes, I am sure of that. The diameter of the cylinder

is 20 inches, the stroke 24 inches, with five coupled axles, rigid wheel base 11ft. 8in., diameter of wheels 30 inches. The heating surface is not yet computed.

2088. By the Hon. E. Llforey.-What power is your engine ?-About 300 horse-power. 2089. By Mr. Craven.-You say you make your rolling-stock 6~ feet in width ?-Yes, approximately. 2090. Where do you get your savings in cuttings and embankments in the narrow gauge in

comparison with the 5ft. 3in. ?-The savings in cuttings are never considered except from the point of gradient. You can run up over hills and down hills. The mere witlth of cutting is not to be considered. We save in being able to take steeper grades.

2091. You spoke about the Glen Wills country; in country like that would you recommend going up the valley on as easy a grade ns you can and then putting in the rack arrangement to go on the Dividing Range, or take an easier grade all round ?-The special instance you note is merely a pocket in the ground. I should recommend zig zagging by 2-ft. gauge in preference to putting in a rack railway.

2092. You spoke about central buffers; do you know in New Zealand and South Australia, where they have them on the 3ft. 6in. gauge, what advantage it gives over the broader gauge ?-The advantnge from a commercial point of view of the central buffer system is that the rolling-stock can be made materially lighter, and in connecting the shunting is far less dangerous to the slnmters.

2093. If we can get all those advantages, light rolling-stock to carry a tremendous load, and also locomotives to pull the big load you speak of, why wonld not the same thing apply as well to the 5ft. 3in. gauge as to the 2-ft. ?-Because of the longer rail going round a curve; inclines are comparatively simple to deal with ; but in broad-gauge curves the outer rail being so much longer it sets up n slip of the inside wheel and lessens the action of the outside wheel. If you build a railwav round in a circle, the outside rail measures 32 feet longer than the other. This slipping over the 32 feet is the detrimental force in cal­cnlatiog, and is also the limiting force, in traction. You can produce traction up to the point where it slips, so the curves are the chief thing to deal with in a railwny. Too mnch emphasis cannot be put on this, because this is the corner stone of the whole question. If you lay the two rails together you lessen that slip just in proportion to the distance between them. The 2-ft. gauge has 36 per cent. of the detrimental force that the .5ft. 3in. gauge has.

2094. You can haul more stuff than on the narrow-gauge ?-U ndonbtedly. 2095. If we had put nll our lines first of all on the 2-ft. gauge it would have been better for us?­

Undoubtedly. You could carry the same quantity of stuff at a far eh ea per rate. 2096. And could develop a far greater traffic than at present ?-Unquestionably. 2097. What is the sharpest radius you would give ?-Minimum, 66 feet. Dealing with the point

of curves, there has been an import,ant thing discovered iu New Zealand which is being adopted in America. They discovered there if, instead of making curves, the lines were made parabolic, that the load entered on the curve more easily nml left it more easily, and the traction was greatly enhanced. That may not bear on the question, but it shows that we have the men here if we only bring them out.

NA.RROW GA.UGE, K

Frederick Upham, 20th August, 1895. 130

2098. Is it not a fact that the I<'estiniog line, built in 1834, was laid out with parabolic curves ?­I am not certain about that.

2099. By the lion. E. Jforey.-What speed do you propose to run ? -The question of speed always excites suspicion, but I may state that Gl miles has been done in New Zealand, and as the curves are easier on narrow gauge than the wide, I see no reason to limit the speed. Y on can run any speed you like with perfect safety.

2100. And going round curves with a 36-lb. rail ?-I fully indorse the idea about strengthening the curves ; it is always desirable.

2101. By .ll:fr . ..flarris.-It is stated on the one hand that the narrow-gauge lines are being taken up, both in India and America, in order to giYe place to broad-gauge lines. Some writers of eminence deny this as a fact ?-It would l111Ye to be a mere matter of opinion, gathered from a literary source. My knowledge of the matter is that, through the timber regions of America, the narrow gauge is entirely superseding the broad gauge.

2102. Here is the statement, and it is of a very valuable character: "In India, 2ft. 6in. gauge open waggon, built of steel, weighing 3! tons, can carry 18! tons of goods, against 11 tons of goods now carried by the 5ft. 6in. broad gauge, in iron or steel waggons, weighing 6± tons'' ?-That is merely going one better. I look on that as a splendid indorsement of what I have said.

2103. There is another point: "A covered waggon, on the 2ft. 6in. gauge, which weighs 4 tons, carries 12~ tons of goods against lOjf tons now carried in standard-gauge waggons, weighing 7 tons." Do you indorse that ?-Yes.

2104. "'Vith a goods train weight of 217 tons, a 2ft. 6in. gauge locomotive can haul, in open waggons, a maximum paying load of 170 tons, or 78 per cent. of the iotal weight, the deaLl weight or tare beiug only 47 tons, or 22 per cent. only. The broad-gauge locomotive hauls, in open waggons, a maximum paying load of 263 tons, equal to 63 per cent. of the gross weight, while the tare weight amounts to 164 tons, or 38 per cent."? -Those items are gathered from traffic that is heavier than is likely to be in Victoria. The estimates I have given have been carefully thought out on the basis of the probable traffic in this colony. Your figures score one better than the estimates that I have given.

2105. "It may be taken as an established fact that, with the same outlay, and exclusive of rolling­stock, 400 to .''iOO miles of feeder lines can be laid on the 2ft. Gin. gauge for every lOO miles of branch lines constructed on the broad gauge" ?-I have always considered that this question of considering railways and equipment in globo is altogether wrong, because a rail way 20 miles long and one 10 miles may have precisely the same equipment, therefore the question must be disct1ssed independently of equipment. I entirely indorse that.

2106. Do you indorse this: "And it furthermore, a well-known fact amongst locomotive engineers that on the same grndients a narrow-gauge locomotive, in proportion to its weight, can haul a. larger load than a broad-gauge locomotive" ?-All of those things are an indorsement of my own evidence.

2107. By tl<e Chairrnan.-'Ne haye an eminent engineer who is considered the highest authority in engineering in Victoria, and he said the man who made that statement was simply mad ; you do not agree with that ?-I once said that the age and position of a witness sometimes giYe importance to his words apart from their worth.

2108. By 1J1r. Harris.-Have you read carefully the evidence given by the Engineer-in-Chief lately ?-Yes.

2109. Arc there any ·points in connexion with it that you desire to traverse?-Yes. I have some pleasure in correcting a statement nmde here last week by Mr. Norman, that an inner guard rail would be an advantage in rounding curves. This, I may say, is contrary to all known principles of railway construction in curves, and is absolutely detrimental to the conditions necessary to be dealt with ; in other words, it would produce derailment.

2110. I asked, have you read the evidence given by the Engineer-in·Chief last, and do you want to traverse anything ?-I do not think it is necessary. I think my evidence has been sufficient as it stands.

2111. By the Chairman.-There is one question that stares us in the face-that there is a great difference in the opinions of practical engineers with reference to the haulage up a steep grade as between the narrow and the broad gauge. Can you explain to us how it is that some engineers who ought to know say that the broad gauge will haul up four times the quantity of the narrow ?-I will explain that to you how it is that that idea is erroneously held. In the ordinary construction of the narrow-gauge lines smaller rails should be used. Those lines are narrower on the face, and do not give so much tractive force per wheel, and thus cannot be worked with an ordinary locomotive. With my locomotive it becomes the same as with a big locomotive with the other wheels. There is 15 inches in contact with the ordinary locomotive on our rails with six drivers. If you take a narrower top rail, in order to get the same tractive force you have to have more wheels ; that is the whole thing. If the conditions in the locomotive designing are correct, I will pull the same load on the 2-ft. line as on the 5ft. 3in. It is because they have run wrongly-constructed locomotives on the narrow gauge, and expected them to produce right results.

2112. By the lion. D. Melville.-What is the cost of your engine approximately, if constructed to-day ?-The cost of the engines will not be materially less than the others.

2113. What is the cost, about ?-I can only speak approximately, between £2,000 and £3,000. 2114. If smaller trucks were more convenient in those ranges, to carry 5 tons for instance, would

your price be reduced in ratio ?-No; because there are many things that are the same on a small truck as on a big one. For instance, the "\'V estinghouse brake. That is why I recommend not to cut down the haulage weights ; carry the same as you do now on more wheels.

2115. What is the cost per mile of the rolling-stock you have in view for the Gippsland ranges?­You have a sliding schedule again, and it depends on the length of the line.

2116. The ordinary railways run from 20 to 35 miles ; what would be the cost of the rolling-stock per mile for the ordinary work in those ranges ?-About £500 a mile.

2117. Even this superior rolling-stock?-Yes. There is no enhanced stock. It is only because of the lighter materials; you make more trucks for the money.

2118. Can the hollow axles be made here ?-Undoubtedly. They are almost in universal use in Germany.

131 Frederick l:pham, 2oth August, 1895.

2119. At what rate per lb. would they be supplied from Germany ?-They are comparatively cheap. Roughly speaking, I suppose .£3 per cwt., and they arc very light.

2120. By the Chairman.-There is a difference of opinion as to the working expenses on the narrow gauge and the broad gauge as to the cost of the coke and oil and all that sort of thing ; which would use the most, hauling the same quantity ?-I will put the answer this way: your saving in the narrow-gauge railways comes in largely from the saving in the first cost and in running up hill and down. You will necessarily run as much steam as on lovellines, therefore the actual cost of steam power would not be much less, only to this extent, that you pull very much less dead haulage. What you use sLeam for you paid for ; that is the key-note of the whole thing. The saving in proportion to the deaJ haulage would I should say, 2 to l.

2121. By .ft'Ir. Craven.-Why is not this locomotive of yours taken in hand by ~ome firm if it is so wonderfully good ?-I am not praising it as wonderfully good. I am merely saying that it is a drawing of a locomotive that I have patented that involves certain principles I believe to Le correct. I have not asked any one to take it up.

2122. I would like to call your attention to a paper written by :VIr. E. Dobson, M.I.C.E., of New Zealand, read before the Australasian Association for the Advancement of Science, 1891. He says-" It is unnecessary here to argue that the reduction of the gauge to 3ft. 6iu. cripples the engine power, or, that, on the other hand, the increase to 5ft. 3in. gives facilities for the use of inside cylinders of larger diameter than can be used on the standard gauge without altering the usual position of the valve chest and intro­ducing a consequent elaboration of the valve gear. Whilst, therefore, in level country, light lines, whether of standard or smaller gauges, may bring good results, the crippling of the engine power by reducing the gauge will ofton, in the case of mountain roads, cause a great increase in constructive cost to obtain grades that can he worked with paying loads by the lighter class of engines." Then as to engineB, he says­" K ow what are the characteristics of engiues on the uroad and narrow gauges as compared with those built for the standard gangc? Tho 5ft. 3in. gauge affords room for the simplest ammgcment of the valve gear with inside cylinders of large diameter, and of large outside cylinders without objectionable overhang, whilst with tank engines the extra 6 inches adus a considerable mileage to the length of the trip between the visits to the water tanks. The 3ft. 6in. gauge involves reduced area of fire grate and diametc; of boiler, whilst the diminished width of base involves reduction of stability, the bringing down of the centt·e of gravity of the engine, and consequently the reduction of the size of the driviug wheels, and as a necessary result, howevei' great the haulage power of these engines, there is a serious diminution of speed as compared with engines built for wider gauges." Do yon agree with that '?-In my opinion the writer of that ought to stick a knife into himself to sec if he would bleed or not ; he does not know whether he is alive or not. The paper is inconsistent with known facts.

2123. By the Chavrman.-How can you account for a notable engineer giving that paper; is he only a theorist ?-What I mean is, it is inconsistent with the up-to-date facts.

2124. By 1'{r. Craven.-This is only 1891 ?-The ideas expressed there are not of the urew of 1891. You will find them in a book written the year I was horn.

2125. Here is another paragraph :-"It therefore appears to the author a matter for the doepest regret that, after the Kew Zealand railways had been started on a gauge which, without an appreciable increase of oonstructive cost, not only gives great stability at high speeJs, but affords ample facilities for the development of locomotive power, this gauge should have been abandoned for a narrow-gauge fad, which, without seriously diminishing constructive cost, has crippled the efficiency of our railway system. One single example of the mischief done may suffice; bad the South lino been constructed to the 5ft. 3in. gauge, on which it was originally commenced, the Dunedin express would have been run in five hours, and passengers leaving Christchurch at 8 a.m. might have had two hours in Dunedin for the transaction of business and have returned by the afternoon express, reaching Christchnrch at 8 p.m. Comment is needless. Is it yet too late to retrace our steps and to reform our railway system so as to make it efficient and self­supporting, if not a valuable source of revenue'?" Have you any remark to make on that ?-I do not think that calls for any answer. Of course I tlo not agree with it. It is no use asking me a question contradictory to all the evidence I have given, to show that the narrow-gauge system, to my mind, is in every way superior. I surely need not be asked over and over again do I agree with somebody who says tho broad gauge is the only right one.

Tlt!J witness witlul!rew.

J. W. Foster Rogers, M.L.A., recalled and further examined.

2126. By the Cltairman.-You gave an exhaustive explanation of your ideas the other day. Mr. Craven was not here, aml he now desires to ask you a few questions ?-1 shall be glad to answer any questions.

2127. 2128.

reasoning.

By M'l'. Cmven.-Have you had any experience at all in the construction of railways ?-No. Your knowledge is obtained from reading, priucipally ?-Chiefly by observation, inquiry, and

2129. Have you seen any of the narrow-gauge lines at work ?-Plenty. 2130. The 2-ft. ?-One. 2131.· You know we have a lot of districts in this colony where they are anxious to get railway

eommunication, and we know it is impossible at the present rates of cost of our lines to give them railway communication. Those places run generally about twenty miles in from the main trunk line. What revenue do you think a place should be able to give to justify a railway being built to it, a narrow-gauge railway ?-Really, I could not say.

2132. \Ve have seven or eight lines now on which the total revenue from all sources is not more than £600 or £700 a year. Of course we lose money on working those. What I want to get at is how to give such places railway communication on the narrow-gauge. \Vhat should be the minimum amount of revenue to justify the Committee in recommending lines to them ?-I cannot say. I look at it this way: if you want a railway to pay for itself, I should not have built one of those lines; hut if you want to open up the country, as in a new country is essential, then I should have built them, but not on our present scale, but on the 2-ft. scale, at one-third or one-fourth of what they have cost us.

KZ

J. W. Foster Rogers, 20t)l Aug~~~~t, 1895. 132

2133. If you could get a tramway built for that money, and could not get a narrow-gauge railway built for it, would you in that case favour a tramway ?-I have my owu notion about it. We have one com­plete system as it is at present on one gauge; that should not be varied more than is essential, and I should not tolerate from a State point of view any State-owned railway of other dimensions than it and the 2-ft. You could do a great deal of traffic on a 12-in. gauge, but I would not recommend that. J,et private people go in for that; but when you come to the 5ft. 3in, terminus and want twenty miles, I would have po shoddy railways, but a proper 2-ft. railway.

2134. Should the State take the responsibility of those narrow-gauge lines, or would you favour a loeal body or trnst beiug formed to take charge of them ?-I think it would be infinitely more advantageous for the State to do it, because they would have to be done on a system. Uniformity is not system. In an army you have artillery, infantry, and cavalry; so iu a railway system it is not necessarily unifor!Ility you want. You want that as much as you can as to the gauge, but you do not want to mix it and have half-a­dozen or so, as they have in India. The tendency of all the evidence is to show that there are only two kinds of gauge which the world really wants, speaking broadly ; one is the standard gauge, 4ft. B!in,, antl the other is the 2-ft.

2135. If a district could not have a 2-ft. gauge railway, would you favour local bodies running tramways to develop the place till the State could put down railways ?-I would, but it is very hard to distinguish between a railway tj.nd a tramway. I know a road on which I have travelled, the Neerim-road, which has been constructed and partially metalled at £1,040 a mile. If 1 am not mistaken you could get a good narrow-gauge 2-ft. line at a price not exceeding that, and you would also give the farmers and residents of those localities the advantage of their own horses and carts for their other work, For the cost and maintenance of that road you could build and maintain a good 2-ft. line. The cost varies according to the strength of the line. You can make shoddy lines, either broad or narrow ; but I would not recom­mend either to be made except to give good u!!e for, say, 30 years.

2136. In the malice country what would such a line cost ?-In the mallee country nature has done half the work of construction, and you surely would be able to put a 2-ft. line there at least at half the cost of a 5ft. 3in., and you would have it stronger, being narrower.

2137. The lines that have cost £1,500 you would lay down with what weight of rail ?-It depends on the traffic and the nature of the country. Generally 20 lbs. would be ample for a 2-ft, line. That is· proved in France.

2138. You say that Van de Velde's steel sleeper takes one-seventh of the ballast tha.t our ordinary sleeper does ?-Yes, that is comparing it with the 5ft. 3in. gauge.

2139. But supposing it is laid on a wooden sleeper, what saving would there be in using the patent steel sleeper ?-If you had good wood at hand you might get a wooden one cheaper, but not to last half as long. I should have no hesitation in sending for the best man from France to build a Decauville railway, and then get another line built by an English or local firm with the wooden sleepers; then you could see which is the better of the two.

2140. You spoke last week about the cattle traffic ; why is it that cattle are now being driven to Wodonga and not sent by rail ?-I am informed that the reason of that is that the owners of those cattle do not like to have the trouble of shipping them into the train twice. This causes great loss.

2141. With the same gauge in our own country, why are cattle driven parallel to the railway line instead of being put in at Tallangatta ?-I suppose it is because they find it cheaper.

:.!142. Then if we had the narrow gauge there instead of the wide, could we carry cattle cheaper?­I think so. One reason is, that you would have so little capital cost originally, and cheaper maintenance also.

2143. You have a patent for transferring; how would you transfer cattle ?-The whole train. You lift up the whole body of the train on bogie stock, and roll away the wheels and put another set of wheels under, at a cost of 7:t;d. per ton.

'/!M ~- witluifr6W.

Adjourned till to-morrow, at half-past twelve o'clock.

WEDNESDAY, 21sT AUGUST, 1895.

Members present:

MR. CAMERON, in the Chair ; The Hon. J. Buchanan, M.L.C., The Hen. D. Melville, M.L.C., The Hon. E. Morey, M.L.C.

Mr. Burton, Mr. Craven, Mr. Harris, Mr. Trenwith, Mr. J. S. White.

John Robb, sworn and examined.

2144. By the Hon. D. Melville.-What are you ?-I have been a railway contractor. 2145. For how many years ?-About 30. 2146. In all the colonies ?-Excepting New Zealand. 2147. You know what is known in the colonies as the narrow gauge ?-Yes, 2148. We are now discussing the question of the 5ft. 3in. gauge as against anything under 3 feet ;

what gauge did you use in the excavation at the West Melbourne Dock ? -Both the 2-ft. gauge and the 5ft. 3in. That was my son's contr:tct.

2149. Why did he use the 2-ft. gauge ?-It was the most suitable for the work. 2150. What was the extent of the work ?-I forget the number of yards shifted; it was a big piece

of earthwork.

133 John Robb, 21st August, 1895.

2151. If you had the same work to do again would you do it in the same way ?-Not exactly. I would use all 2-ft. gauge, no 5ft. 3in. at all ; at least I would advise my son to do so.

2152. What branch or cockspur railways have you constructed in the colony ?-I made two in Gippsland, the }foe to Korumburra line, and the one from Morwell to North l\Iirboo.

2153. Taking the line from )forwell to North :Mirboo, do you remember the cost ?-No. 2154. The return says £153,700 for twenty miles; the loss on working is £1,131, and the loss on

interest and working is £7,000; do you think, if yon had had the sole direction of that railway, you would have constructed it in that way for the traffic there ?-I do not know that I would have made a break of gauge in that place. I think in these broad-gauge railways they might have cheapened the construction by making sharper curves and getting engines suitable for them. I do not consider the engines they run on the .'ift. 3in. gauge suitable at all. The reason the 5ft. 3in. gauge was adopted in the dock was that it would suit the Government to have the same gauge ~r trucks for what they took out of the dock. I advised my son to get two Yankee engines ; he could not keep the English engines on the road, but the Yankee engines never ran off the road onee. I made the Cairns Railway on the 3ft. 6in. gauge. I was working with a Yankee engine, and when the Government came to work the same traffic with the same trucks, the English engines would pull only half the load, though they were heavier. They expended a lot of their power in getting round the cnrves, but the Yankee engines ran round quite easily, and could take half as much again more load. The lines in Gippsland might have been made cheaper. I reckon on the 5ft. 3in. gauge you could work with 5-ehain curves if you had a Yankee engine made specially to work them, but 8-chain curves is as much as yon ought to go with the 5ft. 3iu. gauge and the present engines.

215•3. Do you think the State could have constructed a 2-ft. gauge at that particular part ?-They could have done it very much cheaper quite easily-perhaps at half the cost.

2156. A solid 2-ft. railwav conld have been constructed there ?-Yes. 2157. Would it have beer{ half the cost ?-I think it could have been done for half the cost; it

depends a guod deal upon the country. 2158. The gross revenue from that line is £2,046 per year; the narrow gauge would have done five

times the actual work there is there?-Yes. 2159. Have you had any difficulty in working the narrow gauge in such country ?-I have had a

lot to do with 2-ft. lines. On the sugar plantations I had ten miles of narrow gauge (2 feet), and it answered the purpose remarkably well for carrying traffic; but, if you carry passengers, the trouble is that the trucks capsize. Cane trucks are very high. \Ve capsized a good many trucks down here at the dock, and I do not know that I would like to recommen(] the Government to make a 2-ft. railway to carry passengers. The lft. Sin. railway is 30 years old, and is suitable for such work as the dock; but I do not know that I would like to ride on it in mountainous country.

2160. What is the grade of the narrow railway ?-That makes no difference. I think we had about 1 in 30 or 1 in 40 at the dock.

2161. When loaded with cane the trucks are liable to capsize; what turns them over ?-The top weight.

2162. How high do you load them with cane ?-I forget. I suppose as high as the men can lift the cane upon the trucks-perhaps 7 feet high.

2Hi3. Was it any fault in the construction of the line that they capsized there ?-No, the line was well constructed; there was a top on it equal to anything in Australia, and the top was kept perfectly smooth.

2164. Was it a Deeauville railway ?-I had about ten miles of the ordinary 2-ft. line laid with sleepers and ballasted like an ordinary railway, and I had four or five miles of the Decauville. We never dared run an engine on the Decauville; it was loose. Two men could lift a piece up and put it any­where to suit the loading of the cane in the field, then horses worked the trucks; and we had convenient points where we put the trucks on to the main line which was permanently laid. We dare not put an engine on the Decauville line except for temporary purposes, taking it from oue place to another.

2Hi5. ·what was the weight of the rail ?-Ahont 25 lbs. They were not so heavy on the Deeau­ville; I think they were about :20 lbs. Onr own construction was quite solid, but the Decauville was temporary ; the rails were riveted together with plates or sleeperR.

21GG. vVhat sized truck did yon have ?-'lJ1e truck was about .'J feet long, I forget the wiifth ; it was made proportionately in every way.

2Hi7. You make a distinction; you approve of the 2-ft. gauge for goods, but you are not so sure about il,; ~afety for passengers ?-No; you can carry any amount of goods on a 2-ft. railway; there is no difficulty.

2168. What can be drawn on a 2-ft. gauge with your engines?-We used to bring about ten trucks of cane Hp this grade. I forgot how many they took out of the dock; they had a good stiff grade there.

2169. Have you those engines yet ?-No, J think my son has sold them. 2170. What do you suppose we could haul with one of those American engines on such a railway

as yon constructed with a grade of 1 in 40 ?-I never made a calculation. 2171. Do you think we could take lOO to us with that engine on the 2-ft. gauge?--~ o, the American

engine was a 3ft. 6in. engine. The engines in the dock were not American engines ; they were made on the Continent.

2172. Ilow did they work ?-Very well indeed. 2173. Could that engine go away with 100 tons ?-No, it might take 30 tons. 2174. What is the cDst of snch an engine ?-It would cost less than £1,000; I think it was about

£800. There was a duty of 2.5 per cent. 2175. Could they he made herc?-Yes, but not so ensily as the others, because there is a lot of cast

steel about them. 2176. There would be no difficulty in using that engine on the ~:t:irhoo line ?-No, not if you put

down another rail. 2177. You made no other spurs iu the colony ?-No. 2178. The l\foe to Thorpdale is IO~ miles in leng-th, and cost £116,000. It loses £5,400 a year in

interest and working; would there have been any difficulty in putting a 2-ft. gauge line there ?-I do not

John Rohb, 21st August, 1895. 134

think there would have been any difficulty in making it 2 feet. It would have carried all the coal that was wanted, and you could have tipped the coal into the big trucks on the 5ft. 3in. gauge without any harm.

2179. With your experience, in the face of these serious losses on the short lines, would you recommend the 2-ft. gauge ?-Yes, but it depends upon what you are going to carry. I would not recommend it for passengers. If you are going to make it for carrying traffic, you can carry any quantity of traffic, an<l it is cheap to construct.

2180. Can you give any idea what the cost of carrying per ton per mile was on the 2-ft. gauges?­If you have the traffic for the 5ft. 3iu. gauge, there is no doubt at all it is the cheapest way of hauling, but if you have not the traffic, the other is the cheapest-you cannot compare the two things. If I had to make a railway to carry that quantity of coal from Moe I would make it a. 2-ft. line, and I would not handle the coal at all. I would put in a raised platform and tip the coal into the broad-gauge trucks.

2181. You know the cost of construction generally of the railways during the last ten years; how much cheaper could you construct similar railways. They are fairly sub:Stantial ?-Yes, very good railways.

2182. How much cheaper could we construct fairly substantial Mt. 3in. railways to-day ?-I think, to construct them on the same plan, using goOtl timber, there woul<l be only the difference in the wages. I believe 6s. a day, instead of 7s. 6d. and Ss., represents the difference.

2183. Would the price of materials not have changed a little ?-If you use redgum and box, it has como down a little, but not in proportion. In rough country I do not think you will do it much cheaper ; I know I lost money on ihose two lines.

2184. Can we construct lines one-third cheaper ?-I do not think so, to make the railways the same way-to do exactly the same work; there 1s not one-third difference in wages.

2185. Would you say one-fourth?-You might say one-fourth. 2186. Then the same class of railway can be produced to-day one-fourth cheaper?-Yes. 2187. Taking inferior oft. 3in. railways, what is your idea as to that ?-The only alteration I would

make in constructing lines in rough country would be to sharpen the curves, and put Yankee engines on the lines.

2188. What difference would that make ?-The sharper the curves, the greater the saving ; but if you keep on using the English engines the curves are sharp enough at present. You can run an engine round a 2-chain curve, but it will take all her power to run herself if it is up a grade.

2189. Why do they use those English engines ?-That is a matter for the authorities to say. I would not use them.

2190. Could you alter the grades with advantage in any way ?-It depends upon the traffic. If you have a heavy traffic, never make a steep grade. Some of the grades are too steep at present, where you have the wheat traffic between Wimmera and Melbourne, and it destroys the earning power of the locorno~ tive.

2191. You would not have a 1 in 20 grade ?-If you had very little to do you might; but it all depends upon the circumstances of the country aml the amonnt of traffic to carry.

2192. Would it cheapen the construction~-Yes. It would increase the cost of working, but it would cheapen the construction.

2193. Any proposition to cheapen it in that way would not have your approval ?-No. 2194. Is there any practical working of 1 in 30 and 1 in 25 grades that you have heard of ?-No.

You can work them, but it is a question of cost. 2195. There would be no advantage in cheapening the railway in that direction ?-No. 2196. Do you know any other direction in which you could cheapen a 5ft. 3in. line ?-No, except

sharpening the curves and putting on engines suitable to work them. 2197. Which would be best, a 1 in 30 grade on a 2-ft. gauge or a 1 in 20 on a 5ft. 3in. gauge?-

1 think they would be both wrong. It applies to one equally as to the other. You have to go 11p a hill, and you have to give the coal and wood to pull it. The 5ft. 3in. would have rather the best of it if there were no curves, but you cannot always get up a hill without curves.

2198. The summary of your evidence is that you are favorable to a change in these feeders ?­Yes, for goods only. I think if you arc going to break the gauge for passengers I would not advise going under 3ft. 6in.

2199. Ry ltfr. Trenwith.-You say the trucks capsizeLl in Queensland. I assume the loading was very much above the top of the truck ?-The trucks themselves are very low. \Ve put iron stanchions on the sides and filled the cane in between, and tied a chain over the top. The cane is about 5 feet long. I think the trucks were about 4 feet wide. The loa(l would be about 5 feet above the bottom of the truck, and the truck is about 2 feet from the rail.

2200. Is it not possible to constmct trucks in which the centre of gravity would be better adjusted? -No; that has been studied out on those trucks. No matter how you make the truck you have got only a 2-ft. base ; you cannot alter that without ulteriug the gauge.

2201. By lowering it you could alter it ?-The truck is made as low as it can possibly be worked. The wheels are only 18 inches high, as well as I remember.

2202. Is cane a more unsatisfactory load from that point of view than most goods that would be curried on the rail way?--Yes, far more ; it is far from being a fair example.

2203. What speed would they be travelling at when they capsized ?-They never ran very fast except when coming to a hill ; they would go about ten miles an hour sometimes.

2204. You had trucks capsize at the dock ?-Y os, I believe so. 2205. Was that from top heaviness ?-You could see them shaking about; the roads were not good

there, you could not keep them good there. They were very !2:ood on the sugar plantation. 2206. There are 2-ft. railways to carry passengers?-Yes, they carried them in Paris at the

Exhibition, but that was a real railroad, that would not be a Decauville, and then it woul<l be straight running and no rough country. In that case, I dare say you conhl earry passengers as well as anything else.

2207. ·when a truck capsized with goods, was there not danger to the driver and stoker ?-No; they were all iron trucks ; they used to bend the iron work sometimes, and we had to get them repaired.

135 JohnRobb, 21st August, 18g5,

2208. By 111r. J. S. lVltite.-Is the 2-ft. line in Queensland made substantially ?-Yes, it is made exactly as you make the 6ft. 3in., that is the one I made in the sugar plantation. The sleepers wore only 18 inches apart, and it was !m !lasted and packed the same as yours.

2209. Were the curves sharp ?-Y os. 2210. \Yas it at the curves where they capsized ? -No, they used to ea psize at every place in some

unaccountable way. Making the road good would not interfere with that because the road could not be better.

2211. You know there is country here where the roads are impassable ; the sl1ires have no money to make roads, and there is no communication between the district and the present terminus. \Vould you suggest a l in 40 grade if there were only about 15 tons to pull once a day ?-No, but I would not make the grade so heaYy ; you spoil the usefulness of the road.

2212. Unless we could make a line cheaply, and make it pay, how will you opon up that country?­I do not think there is anything better for opening up country than the 2-ft. railway; that is to say, you can go from 1ft. 8in. to 3ft. 6in.

2213. By jlfr. Trenwith.-Is there much difference in the cost of a 2-ft. and a 3ft. 6in. line?­y os, r. groat difference. Y on can go round almost anything with a 2-ft. gauge.

2214. The gauges appear to be 2-ft. and 3ft. Gin. ; there appear'S to be nothing between the two? -The reason I advocate the 2-ft. line is becanso there are plenty of engines and trucks thnt yon can get ; you lmve only to wire for them. I do not think they could make them here at anything like the cost, because they are nearly all of east steel. You can get engines that will pull a good load and work "·ell on the 2-ft. gauge, but you want to keep a top on the road, the same as with other railways. vVith those lines you can rnu round n big stump almost.

2215. By 11'fr. J. S. White.-In making those branch feeders to the main lines, from your knowledge of the country generally, do you think the State should take charge of the working, or should a trust be formed in the locality that would have to take charge of the whole thing ?-I think the Government are the best to work it. I think when they arc spending State money the Government should spend it them­selves, and have control over it-there should be a board ap11ointed by the Government to work it.

2216. JJy .il-11-. llarris.-You expressed some doubt as to the safety of carrying passengers on the 2-ft. gauge. Have yon read various statements made by different engineers in various parts of the world showing the experience of actual practical working of the 2-ft. gauge, and less gauges, in different parts of the world with steep gradients and sharp cmves, carrying enormous passenger traffic ?-1 have not roacl about the enormous passenger traffic. I know they have had lft. 8in. railways for 30 years carrying goods. The rolling-st0ck that I had experience of was made by the best makers in Europe. There is no place in the world where more value is set on lnunan life than here, and if you killed a man, even on the narr0w gauge, you would have to pay. At the same time, if yon travel at a very slow rate of speed and keep a good top on the road, it might he all right. In tl1e first place the trucks are very light and are easily knocked off the road ; every 6 inches you have in gauge makes it harder to knock the truck off the road.

2217. ·what were the curves on the line that you used in the sugar plantation ?-I forget. I think it was any curve to suit the ground; you can run easily round a 1-chain curve. Iu the dock it was more a tramway than a railway, but on the plantation it was a good first-class railway. The grades were steep in some places, but short, It "·ould not do to carry human beings in the way we used to rnn there.

2218. Does not your experience lead you to think that with some alterations ir: the rolling-stock that you hat! and the mode o£ procedure in using them, it would be perfectly safe to carry passengers?­You would require to have proper carriages; mine wore only cane trucks. No doubt you could put a carriage on the railwny and it might run for a long time, but it must be easier to knock off a 2-ft. truck than a 3ft. 6in. or 5ft. 3in. one. I never saw a 3ft. 6in. or 5ft. 3in. truck upset in the same wuy.

2219. By 1lfr. J. S. f'Vltite.~In America, do they insure the lives of passengers as cheaply on the 2-ft. gauge as on the 5ft. 3in. ?-I do not know. As to a break of gauge, if you break the gauge at all, I would recommend the 2-ft. gauge.

2220 . .By the Hon. J. Bucluman.-In giving the prices of the construction of the Moo Hailway, were the rails cheaper then than they are now ?-I think so. I do not remember the price then.

2221. What did you pay for sleepers ?-I used ironbark sleepers. I paid 4s. each to the people for delivering them at the station. That was thought a reasonable price, because there is no end to their life.

2222. You did not use any rerlgum ?-No. On that line it was all ironbark. ·we got it out of the Gippsland forest ; they cost a little more at first, but they pay the country better in the long run.

2223. Did yon use iron or steel rails ?-There is a gre<~.t fall in the price of steel rails now, more than in anything else used on a railway in proportion. I believe they are about half the money now.

2224. By 111r. Burton.-Y uu said there were two different classes of rolling-stock and rails on the railway in Queensland; what was the other portion that was not Demmville?-The ordinary 2-ft. railway made in the ordinary way. You would not know the difference between them and the railways you run on here except for the width.

2225. What make of engine did you use ?-I had one called the Fowler engine, and I had another the same as was used in the dock down here.

2226. What weight of cane did you carry on the line?-We carried 25 or 30 tons of cane. 2227. \Vhat was the maximum distance you had to haul it ?-About five miles. 222!:5. What was the cost of the construction of that line ?-It was done very cheaply-! cannot

:::ay from memory. 2229. What "'as the Decauville railway used for ?~For taking through the cane fields and taking

the cane; when you had done one portion yon carried the railway through the middle of it to another place.

2230. What sleepers did you have ?-Steel sleepers. 2231. Did they keep the rails in position ?-That was only a temporary road; we did not run an

engine on it. 2232. Would you recommend the Decauville railway for carrying goods and pa.ssengers ?-No. I

would have a railway with ballast and sleepers, the same as we have now.

John Robb, 21st August, 1895. 136

2233. Had you an engine supplied by the agent of the Decauville Company on that railway ?-No, I would not run an engine on that road unless I was running across a level plain. It might run across river flats, but, for a permanent thing, the ordinary lines are far the best.

2234. 'Vould you advocate breaking the gauge for small traffic on branches away from the main line ?-It might be a matter of necessity to break the gauge-you might be able to make a 2-ft. railway and not be able to make a 5ft. 3in. one at all.

2235. Take the l\Iirboo to North Mirboo line; knowing all the circum~tances surrounding that country, would you make a permanent 2-ft. line there ?-No, I would not, before the railway was made. With the information yon have now that the traffic is so disappointing, I think you would be justified in making a 2-ft. line.

2236. Would £100 per mile of revenue justify the construction of a narrow-gauge line ?-I do not know·.

2237. By Jfr. Craven.-If we had to do our work over again, what gauge would you advocate?-4ft. 8~in. for the whole of the railways in this colony. The greatest mistake ever made was when :Mr. Elsdon returned from America and advocated making a 4ft. 8Fn. line to Beechworth, and no notice was taken of it at all. If that had been done the trucks would have been running with produce all over New South "\Vales. That gauge is becoming the universal gauge of the world where the traffic warrants it.

2238. Taking the line from Morwell to North Mirboo, what could that line be made for on the 2-ft. gauge ?-It would be only guesswork to say.

2239. If the Engineer-in-Chief gave us the result of the survey and the quantity of earthworks, would you place confidence in what he said?-Yes, I would. I know he is a most reliable man.

2240, Would the narrow-gauge line be suitable for carrying live stock ?-No, I do not think so; there would be the same fault as with the cane. You might carry sheep, but cattle would be very dangerous.

2241. You know the ,J umbunna to Outtrim line which is now being constructed, would you have put a 2-ft. gauge down there in preference to the 5ft. 3in.?-I think so. It is astonishing the amount of work you can do with the 2-ft. gauge. If it were easy country with easy grades, I would not break the gauge.

2242. The grade is 1 in 40; to run the narrow gauge on a grade of 1 in 100 increased the length two miles; which would you prefer then ?-I think they have not done the thing fairly. One in lOO is no grade at all-it is too easy. One in 80 is an easy grade ; you could run 1 in 50 easily enough. One in 40 would be very little longer with the narrow gauge, because you would only run round the hills instead of through them.

2243. vVould you prefer the narrow gauge with a grade of l in 80, and increasing the length one and a half miles, to a 1 in 40 grnde with the present gauge ?-I would take the 1 in 40 if you had a lot of work to be done ; but I do not like a 1 in 40 grade, it is so expensive to work.

2244. Do you know the tramways at Walhalla ?-No. 2245. You know some of the timber tramways iu Gippsland ?-Yes. 2246. What do you think of a system of tramway that ean be made for £200 or £300 a mile to

drag goods out of a district ?-I would not have them at all-I would not have wooden lines at all. 2247. Suppose you have iron rails ?-Then you have a 2-ft. railway at once. 2248. You would wind about to keep an easy grade?-Yes. 2249. Provided those tramways were limited to carrying goods only, you would favour them?-Yes. 2250. Have you had any experience of the cost of haulage on tramways with horses ?-No, I have

had tramways on railway cuttings, but that is not a fair comparison. 2251. Do you think it is a good plan in the end to cut down your weight of rail in the first

instance?-No, I do not think it is. 1 think it is the reverse; steel rails are so cheap now, and they save sleepers.

2252. It is false economy to use anything but a good heavy rail?-Yes. Of course it depends upon the gauge what the weight of rail would be. If you take the 70-lb. steel rail as heavy enough for ordinary lines, you must take into consideration what that has to carry and what the narrow gauge has to carry, and you will find the rail may be very light.

2253. Suppose you put a lighter engine on the 5ft. 3in. gauge; could you not have as light a rail on the 5ft. 3in. gauge as on the 2ft.?-~o, you cannot. The axles must be stronger, and everything in proportion-there is a wider base.

2254. If your locomotive were spread over a greater number of thiving wheels, so as to distribute the weight, could you not have a lighter rail ?-I believe that is one thing we have failed in here, and they have not given the 5ft. 3in. gauge a fair chance by having engines suitable to work them.

2255. If you are making a line to a gravel pit you make it roughly?-Yes. 22f:i6. What would that cost per mile 7-You can make it from £1,000 to £3,000 a mile. 2257. On ordinary undulating country can you make a line to a gravel pit for £1,000 per mile?

-Yes. 2258. Would not such a line be suitable for those outlying districts?-Yes, but where I made it for

£1,000 per mile it wa8 all level country, with no sleepers, ballast, or anything. 2259. By Mr. Harrz's.-That was a good time ago?-~ o, it was the last railway I made in Victoria;

that was the W arracknabealline. 2260. Would it cost considerably less now ?-There would be a little difference in the labour. 2261. By ilfr. Cmven.-Suppose you put in much sharper curves on a 5ft. 3in. gauge ; say they

were 2 chains radius, and you hall the stock to go round those curves, could you alter the cost of the construction Yery much ?-Yes, tremendously ; but I would not recommend altering the curves to that extent. I think 5 chains is as much as you can go with a 5ft. 3in. gauge.

2262. In America they are running round curves of lOO feet with the 4ft. 8~in. gauge but the extra inches makes a tremenclous difference. In America they design the engine to suit the road; they do not buy any sort of engine, as is done here ; when they make a road there they make an engine to suit it.

2263. Do you think if we adopted the system now adopted in many places with a reasonable traffic of making a lighter road and putting in sharper curves and introducing the proper kind of stock, that would

137 JobnBobb, 2lat August, 1895.

be a distinct advantage ?-It would if you had any traffic to warrant it at all. I do not believe in breaking gauges.

2264. Could we reduce the cost of the 5ft. 3in. gauge to nearly half?-No; the rails, sleepers, and ballast are the same.

2265. Could you reduce the earthworks by half?-No, I do not think so. If you had a fair average country with a prospect of fair traffic I would never break the gauge.

2266. By the Hon. E. ftlorey.-You would rather make the narrow gauge to J umbunna, than the wide gauge ; what would be the difference in the haulage of the stuff ?-I never went into the calculations.

2267. What was the difference in the docks between the wide and the narrow gauge over the same distance ?-The wide gauge is the cheapest-! cannot say how much ; we had a suitable engine for t.he work, and you can carry cheaper if you have work enough to do; but where it is a matter of having a cheap line or doing without a railway at all I would recommend the narrow gauge. Coal is a thing I would not handle at all in the shipment, even if you have a break of gauge-you could have raised platforms and tip it from one truck to the other.

2268. What percentage of loss would there be by that system ?-I do not think there would be much loss. I know of coal mines in Victoria where the traffic is very little indeed, but if you have the traffic you can carry the stuff cheaper on the wide gauge.

2269. By the Ilon. J. Buchanan.-In speaking of your Queensland line, you said it was better than ours ; what was the difference ?-I put my sleepers closer. One fault in our Victorian construction is that they do not put the sleepers close enough.

2270. Were the sleepers split ?-They were squared, which is better than sawn. 2271. Wore they full size ?-For the 2-ft. road my sleepers were 7 inches wide, 3! inches thick

and 18 inches from centre to centre. 2272. By 111-r. Trenwith.-Is it better to have smaller sleepers closer together ?-I would not put in

a sleeper less than 4~ inches thick for the 5ft. 3in. gauge. The 2-ft. gauge is the thing where you are not warranted in making the 6ft. 3in. gauge.

2273. By the Hon. J. Buchanan.-Is the timber better and cheaper there than it is here ?-It was cheaper ; the rails were 25 lbs.

2274. The improvement was only by putting in more sleepers ?-That was all. 2275. By the Hon. D. 1Welville.-Have you seen in Queensland or anywhere else such specimens

of carriage-building as exist in Victoria. such as a truck that weighs 19 tons 3 cwt., or a passenger carriage weighing 20 tons sometimes carrying three passengers ? -No, I do not know that I have. There are bogie carriages made purposly for running round sharp curves, and if you have sharp curves I would recommend that all the carriages should be bogie carriages.

2276. Can our railways ever be profitable with the traffic that we have with such big carriages ?­A lot of railways have been made that will not pay in my lifetime. If you have traffic for those carriages they can be used ; if you have sharp curves you must have bogie carriages.

2277. By 1rfr. Craven.-Suppose we were to extend railways in the mallee, would you recommend breaking the gauge in that country ?-No, I would not.

The witness withdrew.

Sir William Austin Zeal, K.C.M.G., President of the Legislative Council, sworn and examined. 2278. By the Chairman.-Probably you are aware why we have asked you to come here to-day. The

question of the narrow gauge vers1M broad gauge for future use in Victoria is now before us, and we were very anxious to get the opinion of gentlemen who we thought were capable of giving good opinions ; will you make any statement you like on the subject ?-I read in the newspapers a proposal on the part of the Engineer-in-Chief to construct railways on a certain plan, which I thought would be a great misfortune for the colony, that is, introducing unusually steep gradients and sharp curves. I notice that rather than advocate the construction of narrow-gauge railways, he would adopt 5-chain curves and grades of 1 in 20. I say that such a line would be practically useless. You could not carry any paying load upon it. I have looked over the evidence given before you by engineers, and I find that Mr. Mais advocates on narrow­gauge lines gradients of not less than 1 in ,)0, with minimum curves of 7 chains radius. I entirely agree with him. It is most undesirable to introduce dilliculties on railways which would prevent their ever being economically worked. A great deal has been said about this break of gauge, and the inconvenience passengers would suffer through it. I say that on every line in Victoria where a branch line joins it, passengers now have to encounter a break of gauge. They have to get out of the trains aml go from the branch to the main line, so that whatever gauge is determined on it would not affect the convenience of passengers in any shape or form. The only inconvenience would be the transhipping of goods. If a line of railway is made from the main line to a point which can only be reached hy a. coekspur, it seems to me that the question of gauge is a secondary matter, and the line should be made on such a gauge and on such a basis as would make it a profitable venture to the St,ate. I submit it is the duty of the Committee to consider whether the narrow gauge would be better in that case than the broad gauge. I will take two simple cases. There is the case of the proposed line to Gembrook, where you go from the plains into mountain country; and there is the other case of what is known as the Lorne line, that is Dean's Marsh to Lornc. The Department made a deviation at Lorne of about a mile and a half, which has addded to the cost of tht~t line some £40,000. If the original line had been taken, and a branch narrow-gauge line made from De:tn's Marsh into Lorue, this branch line could have been made by the State at a less cost than the present deviation ; in other words, the direct line plus the branch line would not have cost as much as the present deviation, and in all probability it would have made some beneficial return to the State. Then take Gembrook. If you attempt to introduce the broad gauge into such a district with a gauge of 5ft. 3in. and grades of l in 20 the line will never pay under any circumstances, but if you could get a line on the narrow gauge built and equipped under £2,000 a mile, with light rolling-stock and inexpensive plant, no doubt it would be remnuerative. A great deal has been said by the officers of the Department as to the introduction of steep grades ami sharp curves, but I point out to the Committee that the whole object and aim of the Railway Department

Sir W. A. Zeal, K.O.M.G., 21st August, 1895. 138

has been to cut down grades, and the first instance that comes to my memory is that on the line near to Korumburra. The Traffic Department stated that the grades were so steep they could not advantageously carry coal between Korumbuna and Melbourne, and they set about to reduce the l in 40 grade to the usual standard. Again, on the North-Eastern line, which is a well laid out lino with flat curves, the grades are now being reduced from l in 40 to something approaohiug l in 50 or l in 60, and I think the last work the Department has undertaken has been reducing the l in 40 grades between Ararat and Stawell. If that is the ense, if it is found absolutely necessary on a mnin line with all the necessary appliances at command, and with superior works and rolling-stock, it seems to me little short of madness to propose constructing lines with l in 20 grades and 5-chain curves in the mountains. Any man of ordinary intelligence must see that such a thing can only end in financial failure. With reference to the cost of the two gauges you have thoroughly exhausted the evidence. It would be no use for me to repeat what other witnesses have said, but I p0int out that my statement is corroborated by 1\fr. Mais, who has had as large an experience in the construction of lines as any man in the colonies. I also point out that you have a return of seven railways which have been constructed in Victoria at a cost of £Fi79,67l, the gross revenue of which is only £10,993 per annum. The working expenses are £16,564, and the interest on the total capital £35,187, showing a nett loss of £40,758 per annum. I think you must see that if ventures of this kind are continued the colony will never get out of its difficulties. Railways, therefore, instead of being a blessing to the country, which they would be if economically constructed and worked, will increase our losses. As against those seven Victorian railways mentioned I instance the Darjeeling Railway, which has been constructed in India and which rises to a height of 9,000 feet above its starting point, that is about 4,000 feet higher than Mount Feathertop; it has grades of l in 28 and curves of 60 feet radius. This railway, under economical management, pays the State 8·17 per cent. on its gross cost. That shows that necessary ordinary economy in railway construction would result, even in sparsely-populated Victoria, in great advantage to t.he State.

2279. By Mr. Burton.-How are the rates and charges comparatively ?-The rates and charges are more than double on the Indian line, but the result is so disproportionate that it completely overrides the question ol' charges. In one case the line has been paying 8·17 per cent., while in Victoria the lines referred to create a loss of £40,000 a year. I suggest that as the Department seems to be quite antago­nistic to the consuuction of cheap railways, it is practically useless asking them to give an opinion about it. Their officers' minds seem made up, therefore the only way for the Committee is to ask the Govern­ment to call for tenders from reliable substantial promoters, and empower them to make and fully equip a line on a certain basis, then let that line be worked for one or two years by the parties making it, and then handed over to the State. That would be a test of bona fides, and remove all objections. It seems to me when the Engineer-in-Chief comes before you and recommends the Department to construct railways with I in 20 grades over 5-chain curves he is asking you to adopt a plan that must prove unremunerative.

2280. You have no objection to the grade being much steeper and the curves sharper ?-1 would not go below a grade of l in 50 on light narrow-gauge lines worked on the adhesive principle.

2281. Even in the narrow gauge?-Yes. I would prefer to have the line a little longer for the sake of economical working.

2282. The Darjeeling line has grades of 1 in 28 and sharper curves ?-Yes, but the nearer you get to a level line the cheaper it will be worked. Those narrow-gauge lines can be made and fully equipped for £2,000 a mile, so that a mile or two extra in length would be quite a secondary consideration com­pared with the disadvantages of steeper grades and sharp curves. A steep grade is a constant expense and limits the power of the locomotive.

2283. Have you considered the difference between the cost of the present gauge and the 2-ft. gauge in similar country ?-I am certain the narrow gauge would, under favorable conditions, not cost more than one-half of the 5ft. 3in. gauge.

2284. The Engineer-in-Chief in estimating the Jumbunna to Outtrim line estimates the cost of the present gauge at £20,000, and of the narrow gauge at £12,000 for four miles extra, that is considerably cheaper per mile ?-Yes; if you have to provide for a very large traffic, and an enormous amount of goods and passengers, I would recommend the broad gauge, bnt it you go to a district where there are practically neither goods nor passengers it is absurd to provide a waggon to do that which a wheelbarrow will do.

2285. As and old rail way engineer, do you think the broad gauge should be constructed very much cheaper than we have had it constructed here lately ?-Under private management no doubt it could be. All our new railways, such as those to the mallee, are only in a make-shift state. If they are to be com­pleted, very much larger expense will have to be added to what bas already been expenJ.ed. There are no stations or any provision for them, so, though they are apparently very cheap, such a conclusion is very misleading and does not show what will be their ultimate cost.

2286. You do not think it would be giving the narrow gauge 11 fair chance if constructed by the present engineers ?-No, I do not. The Engineer-in-Chief recommends broad-gauge lines, having .I in 20 grades and 5-chain curves, should be constructed instead of the narrow gauge. That proposal can only end in disaster.

2287. By having accidents ?-No, from a commercial point of view. 2288. By Mr. Trenwith.-You say that you think the grades should be lighter and the curves

flatter because of the extra cost of management incidental to heavy grades and sharp curves. Have you considered the question of interest on capital; may it not be possible to have a comparatively expensive yearly average of working expenses, and yet save money by the immense saving in first cost?-Yes, if you had the population; hut in this country we have a small population, so that if you condensed your traffic you could take on almost any branch line a week's traffic in one train in any one day.

2289. Supposing making the curves and grades that you think desirable entailed a cost for interest on capital of say £lOO extra, and the extra cost of working were £90 there wou!J. be a saving of £10 a year ; is not it possible, in connexion with rail ways into country where the traffic is not very great, to make a railway with heavy grades and sharp curves, thereby entailing additional expense in working, but more than saving that additional expense in the interest ou first cost ?-No, l)eeause the conditions of working here are so adverse. Everything in the shape of labour and material is most expensive. 1 think the great aim of the State should be to keep its labour well in hand, not to have any surplus material or labour, in short, to provide only what is absolutely necessary.

139 Sir W. A. Zeal, K.O.M.G., 21st August, 1895.

2290. By Mr. Burton.-Ha.ve you in your experience had any experience of working narrow-gauge railways ?-No.

2291. Have you travelled over narrow-gauge railways?-Yes, but not on the 2-ft. gauge; I have been over a great number of lines on the 3ft. 6in. gauge, and have carefully studied their capabilities and working.

2292. By 1Jfr. Cra1•en.-Would it not have been very much better if the Dean's Marsh line had started from somewhere near Mount Moriac ?-Probably so. I was not criticising the plan of the line, but pointing out what has been done.

2293. Has not that muddle at Dean's Marsh been caused more through selecting the wrong route than the bungle at the station itself ?-Partly so, but the principal outlay has been mmsed by deviating the line a mile and a half out of its route for a subordinate object.

2294. Were those narrow-gauge lines that Mr. Mais speaks of 3ft. 6in. lines ?-Yes; I believe he refers to the South Australian lines.

2295. Do you think that if we constructed li'nes on the 2-ft. gauge they should be under State control, or nnder the control of a local body or trust ?-There is no doubt the State cannot manage rail­ways as economically as a private body or a trnst, but it seems to be the settled policy of the country that the State shall manage the railways, therefore all possible means should be taken to minimise the loss.

22ll6. Coulll they construct the lines as cheaply as private bodies ?-No. 2297. In the evidence that we had yesterday from 1\'Ir. Upham in favour of the narrow gauge his

principal point was that the ad vantage of it over the wide gauge was that it enabled sharp curves to be made, and also to go up hill and down dales on very steep grades ; you do not agree with that ?-I say a railway might be worked on a 1 in 20 grade, but it would be better to increase its length and obtain grades of 1 in 50.

2298. In this return there are 23 lines, starting with the one from Burnley to 'Vaverley-road and going down to Hamilton and Coleraine ; the total loss in the working of these lines is, on the Burnley to Wavcrley-road £3,186 per annum, and it comes down to a loss on working of £513. The approximate loss per annum of interest on capital is, on the Burnley to \Vaverley-road, £6,868, and it comes down on the Hamilton and Coleraine line to £4,407. In those instances we wonld lose even with the narrow gauge a great deal of money ; what would you do in country of that sort ?-I will explain why those branch lines do not pay. Take the Burnley to Waverley-road; there is an engine running on that line that weighs approximately 25 tons. There are two carriages that weigh nearly 20 tons each, and a carriage that weighs from 12 to 13 tons ; that is about 78 tons. For three-quarters of the day that mass of dead weight is being dragged along from one end of the line to the other to carry five or six pasRengers per train ; that is more than 10 tons of dead weight to each passenger. It is impossible that any line can pay under those conditions. I will tell you what came under my notice a fortnight ago. I was at Ascotvale station and had to wait a little time. I noticed the trains passing the station, and I reckoned up the number of passen­gers in those trains. I found that the dead weight in those trains was frequently much more than 10 tons per passenger. It is impossible that any system can pay under those conditions. On the Burnley line near where I am living a siding had been laid down to allo\Y goods trains to pass when the line is occupied by a passenger train. The departmental idea of economy has been recently illustrated by pulling up two points and crossings, and to leave the siding n wreek. If a second train now comes on that line at this point it is impossible to pa8B the station. The Department must have spent £20 in providing and cutting new rails and laying them merely to abolish this siding, and it is noteworthy that the material and points and crossings have been thrown in the water tables to rust and be destroyed.

2299. Do you think we have gone nearly far enough with rail way construction in this colony for a few years to come ?-I think so, under the present management. Such a course will only add to our difficulties. Unless a more economical system is observed it is impossible to construct railways to advantage.

2300. By the Hon. E. 111m·ey.-Would it not be much cheaper to make those narrow-gauge lines than to make roads in the country?-Yes, I think so, and they would be much more economically worked. The settlers \Yould get the advantage of being able to take their produce into market at all times and seasons of the year, which they cannot do with a road.

2301. By Mr. Harris.-You know the Morwell to North :i\::Iirboo line ?-Yes. 2302. It cost £153,700 exclusive of rolling-stock. If you had to construct that line in that class of

country do you think the narrow gauge would be suitn.ble for it?-Yes, I think it would be adequately served by the narrow gauge.

2303. If you had had the construction of that line from Morwell to North Mirboo you would have built it on the narrow gauge and not on the broad gange on a narrow gauge. The traffic in the neighbourhood is insufficient to pay for an expensive line.

2304. That is the opinion you would have formed at the time of its inception ?-Yes. 2305. With regard to the line from Moe to 'fhorpdale, similar remarks would apply?-Yes. Ithink

all those short branch lines are built on R most unnecessarily extravagant scale. 2306. The statement has been made that it was better to make a railway of the standard gauge with

a 40-lb. rail than to break the gauge ?-So it would be if you were connecting two main lines, but where you require only a cockspur to an outlying or mountainous tlistriet, I say a break of gauge is not objection­able when the great advantages which would accrue to the State in the less cost of construction are fully consiueretl.

2307. By 1Jfr. Burton.-This return shows that the whole traffic over this line to North :Mirboo is equal to £100 per mile. Could any railway of any kind pay through that country if that was the maximum earnings ?-It depends upon working expen~es and the cost of the railway per mile. At present, I do not think any railway could be made to pay on that basis.

2308. These conclusions must be fallacious because they are not based upon the arcrument of a line which ought to be constructed at all to that extent only. The State determines a "'railway shall be made in a certain direction ; if that is observed, the duty of all connected with the State is to make a line that will minimise the loss, so that, in the case of the North Mirboo Railway, if the line had been made on an economical scale the loss would have been reducetl one-half; snch a loss would therefore be a matter of very little concern ; as it is now it is a very serious matter.

The witness withdrew. Adjourned,

Sm,

140

[Letter forwarded by the Engineer-in- Chi if. J Board of Land aml Works (Railways Construction Branch),

Engineer-in-Chief's Ofiice, Melbourne, 24th August, 1895.

Tlte Secretary, Parliamentary Standin,q Gommitte~ on Railways.

Referring to the evidence tendered lately before your Committee on the subject of steep grades and sharp cmves in railway construetion, I desire to point out that the law so confidently formulated by some of the recent expert witnesses-that no steeper grade than 1 in 50 is justifiable for an adhesion railway under any circumstances-is contrary to good railway practice in every mountainous country in the world.

As to sharp curves, for the last 40 years or more railways on the 4ft. B~in. and 5ft.. Sin. gauges have been made and worked successfully in America and elsewhere, with numerous curves of 5 chains radius or less. To take a recent example, Engineering of ,Tuly 12th, 1895, illustrates and describes six powerful engines of the consolidation pattern, constructed by the Baldwin Locomotive W orb, Philadel­phia (the largest locomotive builders in the world), and weighing 74 tons each, for the Cia. Panlista Railway, Brazil, 5ft. Sin. gauge, curves SOO feet radius. Such an engine would haul up a 1 in 20 gmde a train of 165 tons. The maximum train loads that might reasonably be expected on country branches in hilly and mountainous parts of Victoria would not be much more than half this weight, and the engines needed for these trains 40 tons weight at the most. The ruling grades and curves of a railway ~hould be and are generally decided by the physical features of the country and the magnitude of business or tmffic expected, the problem being to make a railway on which the annual working expenses and interest on capital will be a minimum. This, which is or should be the determining factor in nearly all cases, has led to the adoption of both steep grades and sharp curves in railway construction for hilly and mountainous country nearly everywhere. Most railway engineers will admit the principle, and the best railway practice for the last 40 years is full of such examples.

No two engineers perhaps would apply the principle in exactly the same way to a specific case, but the variation would not be great if dealt with by engineers familiar with mountain railway construction and working.

The application of this principle to Victoria justifies for difficult country ruling grades of from 1 in 50 to 1 in 20, according to the prospects of traffic and development. For unimportant branch lines in such country, and it is only for snch that I recommend a ruling grade of 1 in 20, steep can be worked both economically and safely. As a matter of fact there are scores of lines in various parts of the world, some most important ones, on both narrow and broad gauges, with such steep grades.

Let us see what a ruling grade of 1 in 20 would mean on our 5ft. Sin. o:r:. on the 4ft. 8~in. gauge. Our present most powerful engine could take 100 tons, exclusive of own weight, up such a grade,

and an engine of the American consolidation type, such as they use on their· steep grades in New South Wales, could haul 150 tons.

Where is there in the hilly and mountainous parts of Victoria any proposed railway that such a method of construction wouhl not suit no·w, or for the next SO years ? I do not know of one such case.

No economical railways of any considerable length, no matter what the gauge may be, can be made in such country without both steep grades and sharp curves.

But poor hilly and alpine Victoria! you arc to be denied railways until you can afford the cost of lines with easy curves and grades not steeper than 1 in 50 !

Let us see wha.t has been d!tne in Australia. and elsewhere in constructing steep grade lines. Mr. John 1Vhitton 35 years ago projected and built trunk railw::~,ys across the Blue Mountains in

New South Wales with a ruling grade of 1 in SO and 8-chain curves which have admirably served the requirements to date and now carry n very heavy goods traffic.

The late :M:r. Thomas Higinbotham adopted the same ruling grade for the Beechworth line here, and the late .\1 r. Robert W atson adopted the same ruling grade for its extension to Y ackandandah. These gentlemen saved in so doing thousands of pounds per mile on what the cost would have been with a grade of 1 in 50, and events have fully justified them. There are scores of such cases in all quarters of the world, some of which arc quoted in the list given herewith.

On the Canadian Pacific Railway, a most important trunk line, a grndc of 1 in 22 has been worked for years. In South America and Mexico such graLles are worked for scores of miles, combined with curves of 4 and 5 chains radius, on costly and important trunk railways. Instances might be multiplied.

Is it to be saitl that in this country grades of 1 in 25 or 1 in 20 would be UilWOrknble or uneco­nomical for lines such as Moe to Walhalla, Bruthen to Omeo, Mansfield to Glenrowan, Healesville to Alcxandra, or cockspurs in Gippsland, in eastern ami north-eastern Vietoria, and through the Cape Otway country? The is preposterous. On the Darjeeling 2-ft. gauge railway, at the foot of the Himalays, in India, a 14-ton locomotive hauls 3.5 tons of \Yaggons and freight up a grade of 1 in 28, and this is pointed to as a triumph of narrow-gauge working, wlwreas a load of lOO or 150 tons, hauled up 1 in 20 on our Victorian gauge, would be counted uneconomical by staunch narrow-gauge advocates. Where is the consistency in such a contention as this? The practice now obtaining of re-grading (flattening grades) on existing trunk lines in New South "\Vales and here is pointed to as a proof that the original steep grarles adopted were a mistake. It proves nothing of the sort. It only shows that after 20 or 30 yea1·s, or to meet an immediate development of traffic that could not have been anticipated, it is expedient now to flatten some of the steeper grades at a moderate outlay. Ap<trt from the few obvious blunders originally made, the practice here of adopting 1 in 30 and 1 in 40 grades on unimportant lines has been fully justified by the first saving in cost ami interest extending over 20 or 30 years ; and the high cost of construction of some of our branch lines is largely due to the fact that this method of reducing cost by steep grades and sharp curves was not carried out to the extent WltiTanted by the small traffic prospects of these branches.

The policy of carrying out railway construction and equipment cheaply in a skeleton form snfficient for proximate requir·ements and improving, enlarging and adding to the property as development extends, is a sound o11e. The other policy of waiting till a district can pay for the construction of a costly railway with easy grades and curves through a hilly or mountainous country is fatal to progress.

141

There is not the slightest doubt that the adoption of steep grades and sharp curves in railway con-struction through difficult couut.ry leads to an economy of thousands of pounds per mile in first cost. Let us see then how the principle affects the working expenses.

A 60-ton locomotive will do as much work on a ruling grade of l in 20 as a 35-ton locomotive on a ruling grade of l in 40, and the trains being of the same capacity, the working expenses will differ by at most 5d. per train mile against the larger engine, or, say, 3s. lOd. as against 3s. 5<1. per train mile. For three trains each way per day, equal to a much l:nger traffic than many Victorian cockspur lines can reach for 30 years, this would amonnt to about £40 per mile per annum, which would balance a saving of only £1,000 per mile in first cost, against some thousands of pounds which would be effected by the adoption of the steeper ruling grade. In some few cases in difficult country a l iu 50 ruling grade might be got by, say, doubling the length of the line without materially inereasing cost of construction ; but the working expenses would be nearly doubled, and higher rates must be charged for carriage. What chance would such a line have of competing against carriers' waggons or traction engines on made roads, which must sooner or la.ter be con­structed in all developed districts. Tl1e practice of the railway world and any rational consideration of the subject will show that, for cheap construction and economical working combined, steep grades and sharp curves are essential for small traffic lines in diffienlt country.

I attach a list of a few out of many railways, amongst them some very important trunk lines, that have been made with steep grades, and successfully and economically worked in various countriefl.

I have the honour to be, Sir, Your obedient servant,

F. RENNICK, Engineer-in-Chief.

STEEPEST GRADIENTS ON ADHESIVE RAILWAYS.

England, Cornwall Mineral Canada, Canadian Pacific United States, A., Pennsylvania Railways .•.

, , " " Mexieo, all lines

Utah Indiana Salt Lake City, Park City Deuver-Rio Grande

, Vera Cruz to Mexico Venezuela, La Quaira-Caracas Peru, Lima-Oroya

, Arequipa-Cuzco Chili, Buenos Ayres-Valparaiso Brazil Alpine Railways, St. Gotthard

, Brenner , Semmering , Mont Cenis , Arlberg , Simplon ,, Zurieh-Uetli ...

:::lwitzerland, other lines ... Austria-Hungary, main and branch lines

, Lambach-Gmunden Italy, Genoa-Aleseandria

, Florence-Bologna Prussia, all lines Bavaria, all lines Saxony, all lines Wurttemberg, all lines Baden, all lines Alsace-Loraine All other German States, all lines Russia, Tifiis-Poli, Caueasus Norway, all narrow-gauge lines Greece, Diakophto Railway Cape Colony . .. • •• Mauritius India, Darjeeling Railway

, State lines .•. , Ghato , Ceylon

Japan , other lines

Tasmania, all lines Queensland, Central

,, Northern South Australia

Victoria New South Wale• New Zeala.nd

Gau!{e. ft. !n.

4 8~· 4 st 2 6 4 Bi 3 0

4 S! 3 0 4 8l

2

4 St 4 8~ 4 st 4 Sl

2 4 st 4 SJ

?,

Various ,

Narrow 4 8! 4 8!

Steepest Grade, lln-

34 22

16·6 9t and 12

16i 16·6 30 25 29 27 2.5 25 40 14! 38! 40 40 33

32i

Various ...

40 14 20 40 29 29 39 25 30 30 25 33 29 23 22 42 28 40 27 28

, , , , " "

Narrow ...

3 6

2 0 5 6

. 6 i)

3 6 3 6 3 6 5 3

6 3 4 8~ 3 6

40 and 15~ ••. 37 42 16 38! 40 30 25 45

30 30 35

Length of Steepest Grade.

o miles continuous. 11 miles. 3 miles. 11 miles continuous. 14 miles. 5! miles continuoue. 23 miles. 48 miles continuous. 27 miles continuous.

5~ miles. 17 miles continuous. 24 miles continuous. 13 miles continuous. 14 miles continuous.

4 miles continuous.

4 miles continuous. 16 miles continuous.

o miles continuous.

131 miles.

116 miles continuous.

34 miles.

25 miles, and 6 miles continuous.

7! miles. 3 miles continuous.

Sm,

142

[Letter jorwa1·ded by Sir William Zeal, K. C.M. G., President oftlte Legislative Council.]

William-street, Melbourne, 28th August, 1895.

To the Chai1·man of the Railways Standing Committee, .Melbourne.

I have read in this day's Age a statement laid before your Committee by the Engineer-in-Chief in support of his proposal to introduce 1 in 20 grades and 5-chain curves in the construction of railways pro­posed for mountainous districts sparsely populated.

It appears to me the Engineer-in-Chief is singularly unfortnnate in his illustrations, and the two remarkable examples he quotes in support of hiB proposal are, I ventnre to affirm, most striking instances of unremunerative enterprise. I refer to the Beechworth ::md Yackandandah lines, on which grades of only 1 in 30 are to be found. Your Committee need only visit these lines and then examine the returns supplied by the Hailway Department to be convinced of this.

No engineer disputes that grades of l in 20 over curves of 5 chains radius can be worked. Tbe question is, can these grades be avoided by adopting a more simple plan for lines such as have been sub­mitted for the consideration of your Committee.

I contend, nay, am certain, this can be successfully accomplished. The Engineer-in-Chief instances what has been done elsewhere than in Victoria, and cites the

"Blue Mountain" range in New South \Vales in support of his contention. Here his illustrations, how­ever, will serve him no better than his Yackandandah precedent.

The "Blue Mountain" range crosses the railway from Sydney to Bonrke, a main line into the interior of New South Wales of 503 miles in length. This line has an altitude at its apex of 3,658 feet near Lithgow (88 miles from Sydney), and 3,176 feet at Huntley (186 miles from Sydney), but has long lengths of compensating grades. This railway is the chief artery of tm!Iic in New South Wales, and over it is carried a large coal supply, and a very considerable proportion of the wheat and wool trade of the mother colony. On this railway the heaviest engines used in Australia are placed; they weigh, with the tender, upwards of 80 tons. The steepest grade on the Blue Mountain incline is, I believe, 1 in 33, and the curves are not, I think, so sharp as the Engineer-in-Chief states.

No doubt if your Committee are prepared to recommend the use of 80-ton engines and very heavy rails, small loads on 1 in 20 grades with 5-chaln curves can be moved on such lines, of which the line to Gembrook is a type.

But I respectfully submit that the class of lines which your Committee have to report upon are those required in mountain districts, where there is now, ancl probably will be for many years, a very small population.

It appears to me a waste of time to even consider the proposals of the Engineer-in-Chief, and I warn the Committee that if they adopt the recommendations of that officer, the results will be even more disastrous than those attending the present system of railway construction and working.

I have the honour to be, Sir, Your obedient servant,

W. A. ZEAL.

By Authority: RoBT. S. BRAIN, Government Printer, Melbourne,

..